3rd Sem All Pgdib

  • November 2019
  • PDF

This document was uploaded by user and they confirmed that they have the permission to share it. If you are author or own the copyright of this book, please report to us by using this DMCA report form. Report DMCA


Overview

Download & View 3rd Sem All Pgdib as PDF for free.

More details

  • Words: 82,636
  • Pages:
LIST OF ATTEMPTED QUESTIONS AND ANSWERS Multiple Choice Single Answer Question Commercial Papers are issued by :Correct Answer Corporates Your Answer

RBI

Select The Blank Question The loans under________ are based on Poverty Reduction Strategy Paper. Correct Answer PRGF Your Answer

PRGF

Multiple Choice Single Answer Question Treasury Bills are issued by the :Correct Answer RBI Your Answer

RBI

Multiple Choice Multiple Answer Question IFCI raises its funds by way of :Correct Answer Issuing bonds , Borrowing from RBI , Borrowing from IDBI Your Answer Issuing bonds , Borrowing from RBI , Issuing shares True/False Question

NABARD does not provide Refinance for stocking & distribution of Chemical Fertilisers. Correct Answer False Your Answer

True

True/False Question

Non bank financial institutions undertake fund and non fund based activities. Correct Answer True Your Answer

False

Multiple Choice Multiple Answer

Question

DICGC does not insure the deposits :-

Correct Answer Of foreign Governments , Of State Cooperative Banks , Of the Central Government Your Answer Of foreign Governments , Of State Cooperative Banks , Of the Central Government Select The Blank Question Development financial institutions have been established to cater to ________ of the industrial sector. Correct Answer Long term financial needs Your Answer

Long term financial needs

Multiple Choice Multiple Answer Question The unorganised financial system comprises of :Correct Answer Money lenders , Indigenous bankers , Lending pawn brokers Your Answer Money lenders , Indigenous bankers , Lending pawn brokers Multiple Choice Single Answer Question Which of the following institution was set up as a subsidy of RBI in 1964? Correct Answer IDBI Your Answer

IDBI

Multiple Choice Multiple Answer Question Under Project Finance Scheme IDBI provides assistance for :Correct Answer Modernisation , New Projects , Renovation Your Answer

New Projects , Renovation , Modernisation

Select The Blank Question One of the objectives of NHB is to augment the ________ resources for the housing sector. Correct Answer Financial Your Answer

Financial

Match The Following

Question

Correct Answer

Your Answer

Small Savings

Chit Funds

Chit Funds

Gen.Insurance

NICo of India Ltd.

NICo of India Ltd.

Life Insurance

LIC of India Ltd

LIC of India Ltd

Mutual fund

UTI

UTI

Multiple Choice Single Answer Question Which one of the following is a Cooperative Bank? Correct Answer COSMOS BANK Your Answer

COSMOS BANK

True/False Question

Bank Guarantee can be Performance of financial Guarantee. Correct Answer False Your Answer

True

Multiple Choice Multiple Answer Question Credit Information report is also known as :Correct Answer Credit report , Banker's report , Confidential report Your Answer

Credit report , Banker's report , Confidential report

True/False Question

Most of the Indian PS Banks practice retail banking.

Correct Answer True Your Answer

False

Multiple Choice Multiple Answer Question Prominent Institutional Investors are :Correct Answer Foreign Investors(FIIs) , Corporate Treasuries , Mutual Funds Your Answer Foreign Investors(FIIs) , Corporate Treasuries , Mutual Funds

Select The Blank Question The price in financial markets is known as ________. Correct Answer Rate of interest Your Answer

Rate of interest

Select The Blank Question Financial markets are the centres that provide facilities for buying and selling of ________. Correct Answer Financial claims and services Your Answer

Financial claims and services

Multiple Choice Single Answer Question Treasury Bills are actually a class of :Correct Answer Govt.Securities Your Answer

Govt.Securities

True/False Question

IMF discourages excessive use of its resources by imposing a surcharge on large loans. Correct Answer True Your Answer

True

Multiple Choice Single Answer Question Initial Quantum of resources of IMF were contributed by members according to :Correct Answer Quotas fixed for members Your Answer

Quotas fixed for members

Select The Blank Question The extent of powers enjoyed by a branch depends on the size of its ________. Correct Answer Business Your Answer

Deposits

Multiple Choice Single Answer

Question

The unorganised financial system includes :-

Correct Answer Indigenous bankers Your Answer

Indigenous bankers

Multiple Choice Single Answer Question Development Financial institutions include :Correct Answer State financial institutions Your Answer

State financial institutions

Multiple Choice Single Answer Question Deposits per depositor per bank are secured by DICGC up to a sum of :Correct Answer Rs. one lac. Your Answer

Rs. one lac.

Multiple Choice Multiple Answer Question The general insurance business can be :Correct Answer Marine insurance , Fire insurance , Miscellaneous Your Answer

Marine insurance , Fire insurance , Miscellaneous

Multiple Choice Multiple Answer Question The objective NEF Scheme of SIDBI is to provide equity support to :Correct Answer New Units in tiny Sector , New Units in Small Sector , Existing units in tiny & small scale sector Your Answer New Units in tiny Sector , New Units in Small Sector , Existing units in tiny & small scale sector Match The Following Question Correct Answer

Your Answer

CRR

Primary reserve requirement

Primary reserve requirement

Bankers Bank

Controlling the reserves of commercial banks FEMA

Controlling the reserves of commercial banks FEMA

Exchange control

OMO

Government securities Government securities

Multiple Choice Single Answer Question Financial institutions deal in :Correct Answer Financial assets Your Answer

Financial assets

Multiple Choice Multiple Answer Question Treasuries could be either those of the :Correct Answer PSUs , Private Sector Cos. , Govt.bodies Your Answer

PSUs , Private Sector Cos. , Govt.bodies

Multiple Choice Single Answer Question Long term debt instruments have a maturity of :Correct Answer Over one year Your Answer

Over one year

Select The Blank Question The Shipping Credit and Investment Company of India Ltd. has changed its name to ________ effective from October 1992. Correct Answer SCICI Ltd Your Answer

SCICI Ltd

True/False Question

A Bill of Exchange is negotiable.

Correct Answer True Your Answer

True

Multiple Choice Multiple Answer Question IDA lends to poor countries :Correct Answer By borrowing from IBRD , By lending at 0% interest. , For periods up to ten years Your Answer By lending at 0% interest. , For periods up to ten years

Multiple Choice Single Answer Question The organised financial system includes :Correct Answer Commercial banks Your Answer

Commercial banks

Multiple Choice Single Answer Question Liabilities of Insurance Cos are mostly :Correct Answer Long term Your Answer

Long term

Select The Blank Question SCICI provides domestic & foreign loans for capital equipment and________ services. Correct Answer Technical Your Answer

Technical

Multiple Choice Single Answer Question Development Financial Institutions provide finance to the needy Correct Answer Corporates & govt. institutions Your Answer

Corporates & govt. institutions

Select The Blank Question GIC raises funds to meet claims from ________. Correct Answer Premium collected Your Answer

Premium collected

True/False Question

The currency notes issued by RBI are legal tender everywhere in India without any limit. Correct Answer True Your Answer

True/False

True

Question

Issuance of CD attracts reserve requirements.

Correct Answer False Your Answer

True

True/False Question

IDA does not lend to countries that are not in a position to borrow from IBRD. Correct Answer False Your Answer

False

Multiple Choice Multiple Answer Question Mutual fund has main entities like :Correct Answer Sponsor , Trust , Asset Management company Your Answer

Sponsor , Trust , Investment bank

Multiple Choice Multiple Answer Question R B I issues currency notes against the security of:Correct Answer Gold bullion , Foreign securities , GOI securities Your Answer

Gold bullion , Foreign securities , GOI securities

True/False Question

The SFCs also borrow from SIDBI & IDBI.

Correct Answer False Your Answer

True

Financial Institutions and Banking_9 Select The Blank Question Kisan Vikas Patra are issued through ________. Correct Answer Post Offices Your Answer Post Offices Multiple Choice Single Answer Question For a focus on the SSI Sector in 1986 IDBI created :Correct Answer SIDF Your Answer SIDF Match The Following Question Correct Answer Your Answer Corporates RIL, HPCL RIL, HPCL NGOs CASP, CRY, PLAN CASP, CRY, PLAN DFI IDBI, IFCI, ICICI IDBI, IFCI, ICICI PSUs NTPC, NHPC, Coal India NTPC, NHPC, Coal India Multiple Choice Multiple Answer Question DFHI was set up jointly by :Correct Answer RBI , Banks , All India Fis Your Answer RBI , Banks , All India Fis , SEBI Select The Blank Question In addition to IFCI and ________ the Soft Loan Scheme for modernisation is also undertaken by IDBI. Correct Answer ICICI Your Answer ICICI Multiple Choice Single Answer Question The unorganised financial system includes :Correct Answer Indigenous bankers Your Answer Indigenous bankers Multiple Choice Single Answer Question Investment on a monthly basis and repayment on due date is made in :Correct Answer Recurring deposit Page 1

Financial Institutions and Banking_9 Your Answer Term deposit account Multiple Choice Multiple Answer Question Tourism promotes :Correct Answer Credit Card Business , International remittances , Travellers' Cheques Your Answer Credit Card Business , International remittances , Travellers' Cheques Multiple Choice Single Answer Question A company issues CP to save on :Correct Answer Interest cost Your Answer Taxes Multiple Choice Single Answer Question A current account where the customer can withdraw more than the balance is :Correct Answer Overdraft Your Answer Overdraft Select The Blank Question AFC's membership consists of commercial, cooperative banks and DFIs including ________. Correct Answer EXIM bank Your Answer EXIM bank Multiple Choice Multiple Answer Question Functions of R.O. do not cover :Correct Answer SLR/CRR maintenance , Liaison with RBI/IBA , Issuing Lending Policy Your Answer Liaison with RBI/IBA , Issuing Lending Policy Select The Blank Question The ________ of NABARD covers release of refinance without prior sanction for refinance limit. Correct Answer ARS Your Answer CFSF Select The Blank Question The ________ regulation makes the banks the largest investor in the GOI securities. Correct Answer SLR Page 2

Financial Institutions and Banking_9 Your Answer SLR Multiple Choice Multiple Answer Question Post Offices are popular for mobilizing small savings of the public due to :Correct Answer Wide network , Tax concessions , Saftey of funds Your Answer Wide network , Tax concessions , Saftey of funds True/False Question The SFCs also borrow from SIDBI & IDBI. Correct Answer False Your Answer True Multiple Choice Single Answer Question SIDBI commenced its operations from :Correct Answer April 2,1990 Your Answer April 2,1990 Multiple Choice Single Answer Question The ultimate source of money in India is:Correct Answer RBI Your Answer RBI Multiple Choice Single Answer Question The organised financial system includes :Correct Answer Commercial banks Your Answer Financial companies Multiple Choice Multiple Answer Question IDA lends to poor countries :Correct Answer By borrowing from IBRD , By lending at 0% interest. , For periods up to ten years Your Answer By borrowing from IBRD , By lending at 0% interest. , For periods up to ten years Multiple Choice Multiple Answer Question The prominent Bond issuing PSU are :Correct Answer MTNL , NTPC , SAIL Your Answer MTNL , NTPC , SAIL Match The Following Question Correct Answer Your Answer Page 3

Financial Institutions and Banking_9 Hill ,Tribal Area development Area Development Area Development Aquaculture ,Brackish Water Fisheries Fisheries Ploughing, weeding, transplantation Seasonal Agricultural Operations Irrigation & Drainage Market Yards, Storage & Warehousing Agricultural Marketing Agricultural Marketing True/False Question Small Savings are source of Capital Receipts for the Government. Correct Answer True Your Answer False Multiple Choice Multiple Answer Question The financial Assistance provided by the IMF enables member countries to :Correct Answer Rebuild resources , Stabilise currencies , Pay for imports Your Answer Rebuild resources , Stabilise currencies , Undertake specific projects Multiple Choice Single Answer Question Cetificate of deposit are issued by :Correct Answer Banks Your Answer Banks Multiple Choice Single Answer Question IDBI has helped in the establishment of :Correct Answer EXIM Bank Your Answer ICICI Select The Blank Question NHB at present has a ________ Capital of Rs. 350 crores. Correct Answer Paid-up Your Answer Paid-up True/False Page 4

Financial Institutions and Banking_9 Question The Capital of NABARD is subscribed by World Bank. Correct Answer False Your Answer False Select The Blank Question In a Bank, the investment decisions are taken at ________ level. Correct Answer Head Office Your Answer Head Office Multiple Choice Single Answer Question RBI is also referred to as :Correct Answer Apex Bank Your Answer Apex Bank Multiple Choice Multiple Answer Question EXIM Bank can raise resources by:Correct Answer Issuing bonds , Borrowing from RBI , Borrowing from International Financial Institutions Your Answer Borrowing from RBI , Borrowing from International Financial Institutions Multiple Choice Single Answer Question PFC is committed to the integrated development of :Correct Answer Power Your Answer Power True/False Question PFC has been persuading State Governments restructuring of their power sector to make them viable. Correct Answer True Your Answer True Multiple Choice Multiple Answer Question SIDBI provides :Correct Answer Refinance , Rediscounting of Bills , Financial support to SSIDC Your Answer Refinance , Financial support to SSIDC , Cash Credit facilities True/False Question Under MIGA the World bank offers various forms of Page 5

Financial Institutions and Banking_9 Political Insurance. Correct Answer True Your Answer True Multiple Choice Multiple Answer Question Other than Tourism, International funds remittances comprise of :Correct Answer Donations/Charity , Payments of Salaries , Payments of Royalty Your Answer Payments of Royalty , Travel related payments True/False Question SCICI is a member of the OTCEI. Correct Answer True Your Answer False Select The Blank Question HDFCs loans were linked up with________. Correct Answer Planned Savings Your Answer Planned Savings True/False Question A Bill of Exchange is negotiable. Correct Answer True Your Answer True True/False Question HDFC was set up by NHB. Correct Answer False Your Answer False Select The Blank Question Mutual Funds provide the benefits of ________. Correct Answer Portfolio management Your Answer Portfolio management True/False Question In case of T-Bills potential investors have to put in competitive bids. Correct Answer True Your Answer True Page 6

Financial Institutions and Banking_9 True/False Question Non bank financial institutions undertake fund and non fund based activities. Correct Answer True Your Answer True Multiple Choice Multiple Answer Question Functions of R.O. of a bank include :Correct Answer Inspection/audit , Control on branches , Grant loans/advances Your Answer Inspection/audit , Control on branches , Grant loans/advances Multiple Choice Multiple Answer Question Duties of an International Banking unit cover :Correct Answer Handling foreign currency , Handling Doc. credits , Granting lines of Credit Your Answer Maintaining SLR/CRR , Handling foreign currency , Handling Doc. credits , Granting lines of Credit Select The Blank Question Secured Premium Notes are ________. Correct Answer Debentures Your Answer Debentures Select The Blank Question Syndication involves ________ sanction. Correct Answer In principle Your Answer Funded

Page 7

LIST OF ATTEMPTED QUESTIONS AND ANSWERS Select The Blank Question The Shipping Credit and Investment Company of India Ltd. has changed its name to ________ effective from October 1992. Correct Answer SCICI Ltd Your Answer SCICI Ltd Select The Blank Question AFC's membership consists of commercial, cooperative banks and DFIs including ________. Correct Answer EXIM bank Your Answer EXIM bank Multiple Choice Single Answer Question Financial institutions deal in :Correct Answer Financial assets Your Answer Financial assets Multiple Choice Single Answer Question The lender of the last resort to the market! is the :Correct Answer RBI Your Answer RBI Select The Blank Question One of the objectives of NHB is to augment the ________ resources for the housing sector. Correct Answer Financial Your Answer Financial Multiple Choice Multiple Answer Question IDA lends to poor countries :Correct Answer By borrowing from IBRD , By lending at 0% interest. , For periods up to ten years Your Answer By borrowing from IBRD , By lending at 0% interest. , For periods up to ten years Multiple Choice Single Answer Question A company issues CP to save on :Correct Answer Interest cost Your Answer Interest cost True/False Question PFC has been persuading State Governments restructuring of their power sector to make them viable.

Correct Answer True Your Answer True Multiple Choice Single Answer Question The ultimate source of money in India is:Correct Answer RBI Your Answer RBI True/False Question The SFCs also borrow from SIDBI & IDBI. Correct Answer False Your Answer True Multiple Choice Single Answer Question RBI was established under the:Correct Answer RBI Act Your Answer RBI Act Multiple Choice Multiple Answer Question The major area of AFC's specialisation included Crop Production as well as:Correct Answer Crop Planning , Plant Protection , Farm Management Your Answer Crop Planning , Plant Protection , Farm Management Multiple Choice Multiple Answer Question A member's IMF Quota determines its :Correct Answer Financial commitment , Voting power , Drawing power Your Answer Financial commitment , Voting power , Drawing power Select The Blank Question Period of Holiday Insurance is ________. Correct Answer Six months or less Your Answer Three years Multiple Choice Single Answer Question The underdeveloped markets are characterised by :Correct Answer Government regulation and control Your Answer Government reg! ulation and control Multiple Choice Single Answer Question Cetificate of deposit are issued by :Correct Answer Banks Your Answer Banks

True/False Question The bids for T-Bills are on price/interest basis. Correct Answer True Your Answer True Multiple Choice Single Answer Question The unorganised financial system includes :Correct Answer Indigenous bankers Your Answer Indigenous bankers Multiple Choice Single Answer Question Long term debt instruments have a maturity of :Correct Answer Over one year Your Answer Over one year Multiple Choice Multiple Answer Question The general insurance business can be :Correct Answer Marine insurance , Fire insurance , Miscellaneous Your Answer Fire insurance , Miscellaneous , Marine insurance True/False Question NABARD does not provide Refinance for stocking & distribution of Chemical Fertilisers. Correct Answer False Your Answer False Multiple Choice Multiple Answer Question The unorganised financial system comprises of :Correct Answer Money lenders , Indigenous bankers , Lending pawn brokers Your Answer Money lenders , Indigenous bankers , Lending pawn brokers True/False Question NABARD is an apex institution in the field of Integrated Rural Development. &nbs! p; Correct Answer True Your Answer True Select The Blank Question 55% of HUDCO loans are earmarked for________& low income groups. Correct Answer Weaker sections Your Answer Weaker sections

Multiple Choice Single Answer Question Development Financial institutions include :Correct Answer State financial institutions Your Answer State financial institutions Multiple Choice Single Answer Question Money Markets Instruments have a maturity of less than :Correct Answer One year Your Answer One year Select The Blank Question The price in financial markets is known as ________. Correct Answer &n! bsp; Rate of interest Your Answer Rate of interest Match The Following Question

Correct Answer

Development financial institutions

IFCI

Organised sector

Banking System

Unorganised sector

Indigenous banks

Mutual funds

UTI

Select The Blank Question The HDFC was set up in 1977 by the ________. Correct Answer ICICI Your Answer ICICI Multiple Choice Single Answer Question Certificate of deposit is a :Correct Answer Usance promissory note Your Answer Term Deposit Receipt True/False Question A Bill of Exchange is negotiable. Correct Answer True Your Answer True Multiple Choice Single Answer Question Commercial Papers are issued by :-

Your Answer IFCI Banking System Indigenous banks UTI

Correct Answer Corporates Your Answer Corporates Multiple Choice Single Answer Question A current account where the customer can withdraw more than the balance is :Correct Answer Overdraft Your Answer Overdraft Multiple Choice Multiple Answer Question The loans of finance companies are generally :Correct Answer At high interest , Unsecured , Based on borrowers worth Your Answer At high interest , Unsecured , Based on borrowers worth Multiple Choice Single Answer Question For a focus on the SSI Sector in 1986 IDBI created :Correct Answer SIDF Your Answer SIDF True/False Question Issuance of CD attracts reserve requirements. Correct Answer False Your Answer False True/False Question Banks issue Guarantees on behalf of their clients. Correct Answer True Your Answer True Match The Following Question

Correct Answer

NABARD

Rural Development

AFC Ltd

Consultancy

HUDCO

Housing

NCDC

Your Answer Rural Development Consultancy Housing

Development of Cooperatives Development of Cooperatives

True/False Question Under MIGA the World bank offers various forms of Political Insurance. Correct Answer True Your Answer True

Multiple Choice Multiple Answer Question In case of a Life policy it is possible to withdraw from the obligation to contribute further premium by :Correct Answer Surrendering the policy , Converting it into paid up policy , Assigning it in the open market Your Answer Surrendering the policy , Converting it into paid up policy , Assigning it in the open market Multiple Choice Single Answer Question The organised financial system includes :Correct Answer Commercial banks Your Answer Commercial banks Select The Blank Question The term ________ refers to the act of a bank extending finance to the seller against a letter of credit. Correct Answer Negotiation Your Answer Negotiation Multiple Choice Multiple Answer Question Functions of a Bank's H.O. include:Correct Answer Liaison with RBI , Evolve Policies , Vigilance & Control Your Answer Liaison with RBI , Evolve Policies , Vigilance & Control Multiple Choice Multiple Answer Question Bonds issued by institutions through SPVs to execute infrastructure projects are :Correct Answer MSR Dev.Corporation , MKV Dev. Corporation. , SSN Nigam Ltd Your Answer MSR Dev.Corporation , MKV Dev. Corporation. , SSN Nigam Ltd Select The Blank Question GIC raises funds to meet claims from ________. Correct Answer Premium collected Your Answer Premium collected ! Multiple Choice Multiple Answer Question Treasuries could be either those of the :Correct Answer PSUs , Private Sector Cos. , Govt.bodies Your Answer PSUs , Private Sector Cos. , Govt.bodies Multiple Choice Multiple Answer Question Under Project Finance Scheme IDBI provides assistance for :-

Correct Answer New Projects , Renovation , Modernisation Your Answer New Projects , Renovation , Modernisation Select The Blank Question The Shipping Credit and Investment Company of India Ltd. has changed its name to ________ effective from October 1992. Correct Answer SCICI Ltd Your Answer SCICI Ltd Select The Blank Question AFC's membership consists of commercial, cooperative banks and DFIs including ________. Correct Answer EXIM bank Your Answer EXIM bank Multiple Choice Single Answer Question Financial institutions deal in :Correct Answer Financial assets Your Answer Financial assets Multiple Choice Single Answer Question The lender of the last resort to the market is the :Correct Answer RBI Your Answer RBI Select The Blank Question One of the objectives of NHB is to augment the ________ resources for the housing sector. Correct Answer Financial Your Answer Financial Multiple Choice Multiple Answer Question IDA lends to poor countries :Correct Answer By borrowing from IBRD , By lending at 0% interest. , For periods up to ten years Your Answer By borrowing from IBRD , By lending at 0% interest. , For periods up to ten years Multiple Choice Single Answer Question A company issues CP to save on :Correct Answer Interest cost Your Answer Interest cost

True/False Question PFC has been persuading State Governments restructuring of their power sector to make them viable. Correct Answer True Your Answer True Multiple Choice Single Answer Question The ultimate source of money in India is:Correct Answer RBI Your Answer RBI True/False Question The SFCs also borrow from SIDBI & IDBI. Correct Answer False Your Answer True Multiple Choice Single Answer Question RBI was established under the:Correct Answer RBI Act Your Answer RBI Act Multiple Choice Multiple Answer Question The major area of AFC's specialisation included Crop Production as well as:Correct Answer Crop Planning , Plant Protection , Farm Management Your Answer Crop Planning , Plant Protection , Farm Management Multiple Choice Multiple Answer Question A member's IMF Quota determines its :Correct Answer Financial commitment , Voting power , Drawing power Your Answer Financial commitment , Voting power , Drawing power Select The Blank Question Period of Holiday Insurance is ________. Correct Answer Six months or less Your Answer Three years Multiple Choice Single Answer Question The underdeveloped markets are characterised by :Correct Answer Government regulation and control Your Answer Government regulation and control

Multiple Choice Single Answer Question Cetificate of deposit are issued by :Correct Answer Banks Your Answer Banks True/False Question The bids for T-Bills are on price/interest basis. Correct Answer True Your Answer True Multiple Choice Single Answer Question The unorganised financial system includes :Correct Answer Indigenous bankers Your Answer Indigenous bankers Multiple Choice Single Answer Question Long term debt instruments have a maturity of :Correct Answer Over one year Your Answer Over one year Multiple Choice Multiple Answer Question The general insurance business can be :Correct Answer Marine insurance , Fire insurance , Miscellaneous Your Answer Fire insurance , Miscellaneous , Marine insurance True/False Question NABARD does not provide Refinance for stocking & distribution of Chemical Fertilisers. Correct Answer False Your Answer False Multiple Choice Multiple Answer Question The unorganised financial system comprises of :Correct Answer Money lenders , Indigenous bankers , Lending pawn brokers Your Answer Money lenders , Indigenous bankers , Lending pawn brokers True/False Question NABARD is an apex institution in the field of Integrated Rural Development. Correct Answer True Your Answer True Select The Blank Question 55% of HUDCO loans are earmarked for________& low income groups.

Correct Answer Weaker sections Your Answer Weaker sections Multiple Choice Single Answer Question Development Financial institutions include :Correct Answer State financial institutions Your Answer State financial institutions Multiple Choice Single Answer Question Money Markets Instruments have a maturity of less than :Correct Answer One year Your Answer One year Select The Blank Question The price in financial markets is known as ________. Correct Answer Rate of interest Your Answer Rate of interest Match The Following Question

Correct Answer

Development financial institutions

IFCI

Your Answer IFCI

Organised sector

Banking System

Banking System

Unorganised sector

Indigenous banks

Indigenous banks

UTI

UTI

Mutual funds

Select The Blank Question The HDFC was set up in 1977 by the ________. Correct Answer ICICI Your Answer ICICI Multiple Choice Single Answer Question Certificate of deposit is a :Correct Answer Usance promissory note Your Answer Term Deposit Receipt True/False Question A Bill of Exchange is negotiable. Correct Answer True Your Answer True

Multiple Choice Single Answer Question Commercial Papers are issued by :Correct Answer Corporates Your Answer Corporates Multiple Choice Single Answer Question A current account where the customer can withdraw more than the balance is :Correct Answer Overdraft Your Answer Overdraft Multiple Choice Multiple Answer Question The loans of finance companies are generally :Correct Answer At high interest , Unsecured , Based on borrowers worth Your Answer At high interest , Unsecured , Based on borrowers worth Multiple Choice Single Answer Question For a focus on the SSI Sector in 1986 IDBI created :Correct Answer SIDF Your Answer SIDF True/False Question Issuance of CD attracts reserve requirements. Correct Answer False Your Answer False True/False Question Banks issue Guarantees on behalf of their clients. Correct Answer True Your Answer True True/False Question Under MIGA the World bank offers various forms of Political Insurance. Correct Answer True Your Answer True Multiple Choice Multiple Answer Question In case of a Life policy it is possible to withdraw from the obligation to contribute further premium by :-

Correct Answer Surrendering the policy , Converting it into paid up policy , Assigning it in the open market Your Answer Surrendering the policy , Converting it into paid up policy , Assigning it in the open market Multiple Choice Single Answer Question The organised financial system includes :Correct Answer Commercial banks Your Answer Commercial banks Select The Blank Question The term ________ refers to the act of a bank extending finance to the seller against a letter of credit. Correct Answer Negotiation Your Answer Negotiation Multiple Choice Multiple Answer Question Functions of a Bank's H.O. include:Correct Answer Liaison with RBI , Evolve Policies , Vigilance & Control Your Answer Liaison with RBI , Evolve Policies , Vigilance & Control Multiple Choice Multiple Answer Question Bonds issued by institutions through SPVs to execute infrastructure projects are :Correct Answer MSR Dev.Corporation , MKV Dev. Corporation. , SSN Nigam Ltd Your Answer MSR Dev.Corporation , MKV Dev. Corporation. , SSN Nigam Ltd Select The Blank Question GIC raises funds to meet claims from ________. Correct Answer Premium collected Your Answer Premium collected Multiple Choice Multiple Answer Question Treasuries could be either those of the :Correct Answer PSUs , Private Sector Cos. , Govt.bodies Your Answer PSUs , Private Sector Cos. , Govt.bodies Multiple Choice Multiple Answer Question Under Project Finance Scheme IDBI provides assistance for :Correct Answer New Projects , Renovation , Modernisation Your Answer New Projects , Renovation , Modernisation Select The Blank Question Kisan Vikas Patra are issued through ________.

Correct Answer Post Offices Your Answer Post Offices Multiple Choice Single Answer Question For a focus on the SSI Sector in 1986 IDBI created :Correct Answer SIDF Your Answer SIDF Match The Following Question

Correct Answer

Corporates NGOs DFI PSUs

RIL, HPCL RIL, HPCL CSP, CRY, PLAN CSP, CRY, PLAN IDBI, IFCI, ICICI IDBI, IFCI, ICICI NTPC, NHPC, Coal India NTPC, NHPC, Coal India

Your Answer

Multiple Choice Multiple Answer Question DFHI was set up jointly by :Correct Answer RBI , Banks , All India Fis Your Answer RBI , Banks , All India Fis , SEBI Select The Blank Question In addition to IFCI and ________ the Soft Loan Scheme for modernisation is also undertaken by IDBI. Correct Answer ICICI Your Answer ICICI Multiple Choice Single Answer Question The unorganised financial system includes :Correct Answer Indigenous bankers Your Answer Indigenous bankers Multiple Choice Single Answer Question Investment on a monthly basis and repayment on due date is made in :Correct Answer Recurring deposit Your Answer Term deposit account Multiple Choice Multiple Answer Question Tourism promotes :Correct Answer Credit Card Business , International remittances , Travellers' Cheques Your Answer Credit Card Business , International remittances , Travellers' Cheques

Multiple Choice Single Answer Question A company issues CP to save on :Correct Answer Interest cost Your Answer Taxes Multiple Choice Single Answer Question A current account where the customer can withdraw more than the balance is :Correct Answer Overdraft Your Answer Overdraft Select The Blank Question AFC's membership consists of commercial, cooperative banks and DFIs including ________. Correct Answer EXIM bank Your Answer EXIM bank Multiple Choice Multiple Answer Question Functions of R.O. do not cover :Correct Answer SLR/CRR maintenance , Liaison with RBI/IBA , Issuing Lending Policy Your Answer Liaison with RBI/IBA , Issuing Lending Policy Select The Blank Question The ________ of NABARD covers release of refinance without prior sanction for refinance limit. Correct Answer ARS Your Answer CFSF Select The Blank Question The ________ regulation makes the banks the largest investor in the GOI securities. Correct Answer SLR Your Answer SLR Multiple Choice Multiple Answer Question Post Offices are popular for mobilizing small savings of the public due to :Correct Answer Wide network , Tax concessions , Saftey of funds Your Answer Wide network , Tax concessions , Saftey of funds True/False Question The SFCs also borrow from SIDBI & IDBI.

Correct Answer False Your Answer True Multiple Choice Single Answer Question SIDBI commenced its operations from :Correct Answer April 2,1990 Your Answer April 2,1990 Multiple Choice Single Answer Question The ultimate source of money in India is:Correct Answer RBI Your Answer RBI Multiple Choice Single Answer Question The organised financial system includes :Correct Answer Commercial banks Your Answer Financial companies Multiple Choice Multiple Answer Question IDA lends to poor countries :Correct Answer By borrowing from IBRD , By lending at 0% interest. , For periods up to ten years Your Answer By borrowing from IBRD , By lending at 0% interest. , For periods up to ten years Multiple Choice Multiple Answer Question The prominent Bond issuing PSU are :Correct Answer MTNL , NTPC , SAIL Your Answer MTNL , NTPC , SAIL Match The Following Question

Correct Answer

Your Answer

Hill ,Tribal Area development

Area Development

Area Development

Brackish Water

Fisheries Fisheries

Aquaculture ,

Ploughing, weeding, transplantation Seasonal Agricultural Operations Market Yards, Storage & Warehousing

Agricultural Marketing

Irrigation & Drainage Agricultural Marketing

True/False Question Small Savings are source of Capital Receipts for the Government.

Correct Answer True Your Answer False Multiple Choice Multiple Answer Question The financial Assistance provided by the IMF enables member countries to :Correct Answer Rebuild resources , Stabilise currencies , Pay for imports Your Answer Rebuild resources , Stabilise currencies , Undertake specific projects Multiple Choice Single Answer Question Cetificate of deposit are issued by :Correct Answer Banks Your Answer Banks Multiple Choice Single Answer Question IDBI has helped in the establishment of :Correct Answer EXIM Bank Your Answer ICICI Select The Blank Question NHB at present has a ________ Capital of Rs. 350 crores. Correct Answer Paid-up Your Answer Paid-up True/False Question The Capital of NABARD is subscribed by World Bank. Correct Answer False Your Answer False Select The Blank Question In a Bank, the investment decisions are taken at ________ level. Correct Answer Head Office Your Answer Head Office Multiple Choice Single Answer Question RBI is also referred to as :Correct Answer Apex Bank Your Answer Apex Bank Multiple Choice Multiple Answer Question EXIM Bank can raise resources by:Correct Answer Issuing bonds , Borrowing from RBI , Borrowing from International Financial Institutions

Your Answer Borrowing from RBI , Borrowing from International Financial Institutions Multiple Choice Single Answer Question PFC is committed to the integrated development of :Correct Answer Power Your Answer Power True/False Question PFC has been persuading State Governments restructuring of their power sector to make them viable. Correct Answer True Your Answer True Multiple Choice Multiple Answer Question SIDBI provides :Correct Answer Refinance , Rediscounting of Bills , Financial support to SSIDC Your Answer Refinance , Financial support to SSIDC , Cash Credit facilities True/False Question Under MIGA the World bank offers various forms of Political Insurance. Correct Answer True Your Answer True Multiple Choice Multiple Answer Question Other than Tourism, International funds remittances comprise of :Correct Answer Donations/Charity , Payments of Salaries , Payments of Royalty Your Answer Payments of Royalty , Travel related payments True/False Question SCICI is a member of the OTCEI. Correct Answer True Your Answer False Select The Blank Question HDFCs loans were linked up with________. Correct Answer Planned Savings Your Answer Planned Savings True/False Question A Bill of Exchange is negotiable. Correct Answer True Your Answer True

True/False Question HDFC was set up by NHB. Correct Answer False Your Answer False Select The Blank Question Mutual Funds provide the benefits of ________. Correct Answer Portfolio management Your Answer Portfolio management True/False Question In case of T-Bills potential investors have to put in competitive bids. Correct Answer True Your Answer True True/False Question Non bank financial institutions undertake fund and non fund based activities. Correct Answer True Your Answer True Multiple Choice Multiple Answer Question Functions of R.O. of a bank include :Correct Answer Inspection/audit , Control on branches , Grant loans/advances Your Answer Inspection/audit , Control on branches , Grant loans/advances Multiple Choice Multiple Answer Question Duties of an International Banking unit cover :Correct Answer Handling foreign currency , Handling Doc. credits , Granting lines of Credit Your Answer Maintaining SLR/CRR , Handling foreign currency , Handling Doc. credits , Granting lines of Credit Select The Blank Question Secured Premium Notes are ________. Correct Answer Debentures Your Answer Debentures Select The Blank Question Syndication involves ________ sanction. Correct Answer In principle Your Answer Funded

FI & B Binu -LIST OF ATTEMPTED QUESTIONS AND ANSWERS Multiple Choice Single Answer Question The type of banking involving a small number of Large Customers viz.Corporates & multinationals is :Correct Answer Wholesale Banking Your Answer Wholesale Banking Multiple Choice Single Answer Question PFC is committed to the integrated development of :Correct Answer Power Your Answer Power Multiple Choice Single Answer Question IDBI has helped in the establishment of :Correct Answer EXIM Bank Your Answer EXIM Bank Select The Blank Question NABARD took over the functions of ACD, RPCC of RBI & ________. Correct Answer ARDC Your Answer ARDC Multiple Choice Multiple Answer Question The major area of AFC's specialisation included Crop Production as well as:Correct Answer Crop Planning , Plant Protection , Farm Management Your Answer Crop Planning , Plant Protection , Farm Management True/False Question Banks issue Guarantees on behalf of their clients. Correct Answer True Your Answer False Select The Blank Question The HDFC was set up in 1977 by the ________. Correct Answer ICICI Your Answer ICICI Multiple Choice Multiple Answer Question The main objective of ICICI was :Correct Answer To augment export finance in India , To channelise World Bank's funds to industries in India , To help in building up a Capital Market in India Your Answer To augment export finance in India , To channelise World Bank's funds to industries in India , To help in building up a Capital Market in India Multiple Choice Multiple Answer Question ECGC Board comprises of Directors representing :Correct Answer Government , Banking/Insurance , Trade/Industry Your Answer Government , Banking/Insurance , Trade/Industry True/False Question Bank Guarantee can be Performance of financial Guarantee. Correct Answer False Your Answer False Multiple Choice Single Answer Question Which of the following institution was set up as a subsidy of RBI in 1964? Correct Answer IDBI

Your Answer

IDBI

Match The Following Question Correct Answer Your Answer Advances

Loans/cash credits

Cash withdrawal facility Remittances

ATM

Loans/cash credits

ATM

MT/DD/TT/EFT MT/DD/TT/EFT

Government business

Collection of taxes

Collection of taxes

Multiple Choice Multiple Answer Question The objective NEF Scheme of SIDBI is to provide equity support to :Correct Answer New Units in tiny Sector , New Units in Small Sector , Existing units in tiny & small scale sector Your Answer New Units in tiny Sector , New Units in Small Sector , Existing units in tiny & small scale sector Select The Blank Question 55% of HUDCO loans are earmarked for________& low income groups. Correct Answer Weaker sections Your Answer Agriculturists Select The Blank Question The ________ of NABARD covers release of refinance without prior sanction for refinance limit. Correct Answer ARS Your Answer CFSF Multiple Choice Multiple Answer Question Mutual fund has main entities like :Correct Answer Sponsor , Trust , Asset Management company Your Answer Sponsor , Trust , Asset Management company True/False Question EXIM Bank provides Lines of Credit to other nations for Indian exports. Correct Answer True Your Answer True Select The Blank Question Legally speaking Pass Through Certificates are ________ and are tradeable freely. Correct Answer Promissory notes Your Answer Promissory notes Multiple Choice Single Answer Question A record of the holdings of GOI dated securities of each holder is contained in :Correct Answer SGL pass book Your Answer SGL pass book Multiple Choice Single Answer Question The underdeveloped markets are characterised by :Correct Answer Government regulation and control Your Answer Government regulation and control Multiple Choice Multiple Answer Question Functions of R.O. of a bank include :Correct Answer Inspection/audit , Control on branches , Grant loans/advances Your Answer Inspection/audit , Control on branches , Grant loans/advances

Multiple Choice Single Answer Question Liquid assets to be maintained to a stipulated ratio under Section 24 of BR Act is called :Correct Answer Statutory Liquidity Ratio Your Answer Cash Reserves Ratio Select The Blank Question In a Bank, the investment decisions are taken at ________ level. Correct Answer Head Office Your Answer Head Office Multiple Choice Multiple Answer Question RBI sells securities through :Correct Answer Auction program , Open market operations , Private placements Your Answer Auction program , Open market operations , Private placements Multiple Choice Multiple Answer Question R B I conducts monetary control by way of O M O in :Correct Answer Treasury bills , State /central Government Securities Your Answer Treasury bills , State /central Government Securities True/False Question The SFCs also borrow from SIDBI & IDBI. Correct Answer False Your Answer False True/False Question No TDS is deducted on Interest paid on CDs. Correct Answer True Your Answer True Multiple Choice Single Answer Question Corporate Debentures are :Correct Answer Long term debt instruments Your Answer Short term deposit receipts True/False Question Amongst others, ICICI also provided long and medium term loans or equity finance. Correct Answer True Your Answer True Multiple Choice Single Answer Question RBI is also referred to as :Correct Answer Apex Bank Your Answer Apex Bank True/False Question HDFC was set up by NHB. Correct Answer False Your Answer False True/False Question IDA does not lend to countries that are not in a position to borrow from IBRD. Correct Answer False Your Answer False Multiple Choice Multiple Answer Question SFCs provide finance to :-

Correct Answer Corporates , Partnerships , Proprietary firms Your Answer Corporates , Partnerships , Proprietary firms Select The Blank Question AFC's membership consists of commercial, cooperative banks and DFIs including ________. Correct Answer EXIM bank Your Answer EXIM bank Multiple Choice Multiple Answer Question Treasuries could be either those of the :Correct Answer PSUs , Private Sector Cos. , Govt.bodies Your Answer PSUs , Private Sector Cos. , Govt.bodies Select The Blank Question The ________ are buy and hold investors and almost never trade on their investments. Correct Answer Provident Funds Your Answer Foreign Investors(FIIs) True/False Question The bids for T-Bills are on price/interest basis. Correct Answer True Your Answer True Multiple Choice Multiple Answer Question IFC provides financial assistance :Correct Answer Through loans , Through purchase of shares , For economic development Your Answer Through loans , Through purchase of shares , For economic development Multiple Choice Single Answer Question Money Markets Instruments have a maturity of less than :Correct Answer One year Your Answer One year Multiple Choice Single Answer Question Treasury Bills are actually a class of :Correct Answer Govt.Securities Your Answer Govt.Securities True/False Question NABARD provides short term refinance for procurement activities also. Correct Answer True Your Answer False Match The Following Question Correct Answer Your Answer GOI securities

ZCB, ILBonds

ZCB, ILBonds

Long Term Debt instruments

PSU Bonds

PSU Bonds

Money Market Instruments

CP, T- Bills

CP, T- Bills

Negotaible Instruments

Bills of exchange ,Cheques

Multiple Choice Single Answer Question IRBI was set up with a focus on :Correct Answer Sick Units

Bills of exchange ,Cheques

Your Answer

Public sector

Select The Blank Question The ________ was set up to help corrections in BOP maladjustments. Correct Answer IMF Your Answer IBRD Select The Blank Question Mutual Funds provide the benefits of ________. Correct Answer Portfolio management Your Answer High returns Multiple Choice Single Answer Question Financial institutions deal in :Correct Answer Financial assets Your Answer Financial assets Multiple Choice Multiple Answer Question Duties of an International Banking unit cover :Correct Answer Handling foreign currency , Handling Doc. credits , Granting lines of Credit Your Answer Handling foreign currency , Handling Doc. credits , Granting lines of Credit

FI & B Binu -LIST OF ATTEMPTED QUESTIONS AND ANSWERS Multiple Choice Single Answer Question The type of banking involving a small number of Large Customers viz.Corporates & multinationals is :Correct Answer Wholesale Banking Your Answer Wholesale Banking Multiple Choice Single Answer Question PFC is committed to the integrated development of :Correct Answer Power Your Answer Power Multiple Choice Single Answer Question IDBI has helped in the establishment of :Correct Answer EXIM Bank Your Answer EXIM Bank Select The Blank Question NABARD took over the functions of ACD, RPCC of RBI & ________. Correct Answer ARDC Your Answer ARDC Multiple Choice Multiple Answer Question The major area of AFC's specialisation included Crop Production as well as:Correct Answer Crop Planning , Plant Protection , Farm Management Your Answer Crop Planning , Plant Protection , Farm Management True/False Question Banks issue Guarantees on behalf of their clients. Correct Answer True Your Answer False Select The Blank Question The HDFC was set up in 1977 by the ________. Correct Answer ICICI Your Answer ICICI Multiple Choice Multiple Answer Question The main objective of ICICI was :Correct Answer To augment export finance in India , To channelise World Bank's funds to industries in India , To help in building up a Capital Market in India Your Answer To augment export finance in India , To channelise World Bank's funds to industries in India , To help in building up a Capital Market in India Multiple Choice Multiple Answer Question ECGC Board comprises of Directors representing :Correct Answer Government , Banking/Insurance , Trade/Industry Your Answer Government , Banking/Insurance , Trade/Industry True/False Question Bank Guarantee can be Performance of financial Guarantee. Correct Answer False Your Answer False Multiple Choice Single Answer Question Which of the following institution was set up as a subsidy of RBI in 1964? Correct Answer IDBI

Your Answer

IDBI

Match The Following Question Correct Answer Your Answer Advances

Loans/cash credits

Cash withdrawal facility Remittances

ATM

Loans/cash credits

ATM

MT/DD/TT/EFT MT/DD/TT/EFT

Government business

Collection of taxes

Collection of taxes

Multiple Choice Multiple Answer Question The objective NEF Scheme of SIDBI is to provide equity support to :Correct Answer New Units in tiny Sector , New Units in Small Sector , Existing units in tiny & small scale sector Your Answer New Units in tiny Sector , New Units in Small Sector , Existing units in tiny & small scale sector Select The Blank Question 55% of HUDCO loans are earmarked for________& low income groups. Correct Answer Weaker sections Your Answer Agriculturists Select The Blank Question The ________ of NABARD covers release of refinance without prior sanction for refinance limit. Correct Answer ARS Your Answer CFSF Multiple Choice Multiple Answer Question Mutual fund has main entities like :Correct Answer Sponsor , Trust , Asset Management company Your Answer Sponsor , Trust , Asset Management company True/False Question EXIM Bank provides Lines of Credit to other nations for Indian exports. Correct Answer True Your Answer True Select The Blank Question Legally speaking Pass Through Certificates are ________ and are tradeable freely. Correct Answer Promissory notes Your Answer Promissory notes Multiple Choice Single Answer Question A record of the holdings of GOI dated securities of each holder is contained in :Correct Answer SGL pass book Your Answer SGL pass book Multiple Choice Single Answer Question The underdeveloped markets are characterised by :Correct Answer Government regulation and control Your Answer Government regulation and control Multiple Choice Multiple Answer Question Functions of R.O. of a bank include :Correct Answer Inspection/audit , Control on branches , Grant loans/advances Your Answer Inspection/audit , Control on branches , Grant loans/advances

Multiple Choice Single Answer Question Liquid assets to be maintained to a stipulated ratio under Section 24 of BR Act is called :Correct Answer Statutory Liquidity Ratio Your Answer Cash Reserves Ratio Select The Blank Question In a Bank, the investment decisions are taken at ________ level. Correct Answer Head Office Your Answer Head Office Multiple Choice Multiple Answer Question RBI sells securities through :Correct Answer Auction program , Open market operations , Private placements Your Answer Auction program , Open market operations , Private placements Multiple Choice Multiple Answer Question R B I conducts monetary control by way of O M O in :Correct Answer Treasury bills , State /central Government Securities Your Answer Treasury bills , State /central Government Securities True/False Question The SFCs also borrow from SIDBI & IDBI. Correct Answer False Your Answer False True/False Question No TDS is deducted on Interest paid on CDs. Correct Answer True Your Answer True Multiple Choice Single Answer Question Corporate Debentures are :Correct Answer Long term debt instruments Your Answer Short term deposit receipts True/False Question Amongst others, ICICI also provided long and medium term loans or equity finance. Correct Answer True Your Answer True Multiple Choice Single Answer Question RBI is also referred to as :Correct Answer Apex Bank Your Answer Apex Bank True/False Question HDFC was set up by NHB. Correct Answer False Your Answer False True/False Question IDA does not lend to countries that are not in a position to borrow from IBRD. Correct Answer False Your Answer False Multiple Choice Multiple Answer Question SFCs provide finance to :-

Correct Answer Corporates , Partnerships , Proprietary firms Your Answer Corporates , Partnerships , Proprietary firms Select The Blank Question AFC's membership consists of commercial, cooperative banks and DFIs including ________. Correct Answer EXIM bank Your Answer EXIM bank Multiple Choice Multiple Answer Question Treasuries could be either those of the :Correct Answer PSUs , Private Sector Cos. , Govt.bodies Your Answer PSUs , Private Sector Cos. , Govt.bodies Select The Blank Question The ________ are buy and hold investors and almost never trade on their investments. Correct Answer Provident Funds Your Answer Foreign Investors(FIIs) True/False Question The bids for T-Bills are on price/interest basis. Correct Answer True Your Answer True Multiple Choice Multiple Answer Question IFC provides financial assistance :Correct Answer Through loans , Through purchase of shares , For economic development Your Answer Through loans , Through purchase of shares , For economic development Multiple Choice Single Answer Question Money Markets Instruments have a maturity of less than :Correct Answer One year Your Answer One year Multiple Choice Single Answer Question Treasury Bills are actually a class of :Correct Answer Govt.Securities Your Answer Govt.Securities True/False Question NABARD provides short term refinance for procurement activities also. Correct Answer True Your Answer False Match The Following Question Correct Answer Your Answer GOI securities

ZCB, ILBonds

ZCB, ILBonds

Long Term Debt instruments

PSU Bonds

PSU Bonds

Money Market Instruments

CP, T- Bills

CP, T- Bills

Negotaible Instruments

Bills of exchange ,Cheques

Multiple Choice Single Answer Question IRBI was set up with a focus on :Correct Answer Sick Units

Bills of exchange ,Cheques

Your Answer

Public sector

Select The Blank Question The ________ was set up to help corrections in BOP maladjustments. Correct Answer IMF Your Answer IBRD Select The Blank Question Mutual Funds provide the benefits of ________. Correct Answer Portfolio management Your Answer High returns Multiple Choice Single Answer Question Financial institutions deal in :Correct Answer Financial assets Your Answer Financial assets Multiple Choice Multiple Answer Question Duties of an International Banking unit cover :Correct Answer Handling foreign currency , Handling Doc. credits , Granting lines of Credit Your Answer Handling foreign currency , Handling Doc. credits , Granting lines of Credit

LIST OF ATTEMPTED QUESTIONS AND ANSWERS Multiple Choice Single Answer Question Commercial Papers are issued by :Correct Answer Corporates Your Answer

RBI

Select The Blank Question The loans under________ are based on Poverty Reduction Strategy Paper. Correct Answer PRGF Your Answer

PRGF

Multiple Choice Single Answer Question Treasury Bills are issued by the :Correct Answer RBI Your Answer

RBI

Multiple Choice Multiple Answer Question IFCI raises its funds by way of :Correct Answer Issuing bonds , Borrowing from RBI , Borrowing from IDBI Your Answer Issuing bonds , Borrowing from RBI , Issuing shares True/False Question

NABARD does not provide Refinance for stocking & distribution of Chemical Fertilisers. Correct Answer False Your Answer

True

True/False Question

Non bank financial institutions undertake fund and non fund based activities. Correct Answer True Your Answer

False

Multiple Choice Multiple Answer

Question

DICGC does not insure the deposits :-

Correct Answer Of foreign Governments , Of State Cooperative Banks , Of the Central Government Your Answer Of foreign Governments , Of State Cooperative Banks , Of the Central Government Select The Blank Question Development financial institutions have been established to cater to ________ of the industrial sector. Correct Answer Long term financial needs Your Answer

Long term financial needs

Multiple Choice Multiple Answer Question The unorganised financial system comprises of :Correct Answer Money lenders , Indigenous bankers , Lending pawn brokers Your Answer Money lenders , Indigenous bankers , Lending pawn brokers Multiple Choice Single Answer Question Which of the following institution was set up as a subsidy of RBI in 1964? Correct Answer IDBI Your Answer

IDBI

Multiple Choice Multiple Answer Question Under Project Finance Scheme IDBI provides assistance for :Correct Answer Modernisation , New Projects , Renovation Your Answer

New Projects , Renovation , Modernisation

Select The Blank Question One of the objectives of NHB is to augment the ________ resources for the housing sector. Correct Answer Financial Your Answer

Financial

Match The Following

Question

Correct Answer

Your Answer

Small Savings

Chit Funds

Chit Funds

Gen.Insurance

NICo of India Ltd.

NICo of India Ltd.

Life Insurance

LIC of India Ltd

LIC of India Ltd

Mutual fund

UTI

UTI

Multiple Choice Single Answer Question Which one of the following is a Cooperative Bank? Correct Answer COSMOS BANK Your Answer

COSMOS BANK

True/False Question

Bank Guarantee can be Performance of financial Guarantee. Correct Answer False Your Answer

True

Multiple Choice Multiple Answer Question Credit Information report is also known as :Correct Answer Credit report , Banker's report , Confidential report Your Answer

Credit report , Banker's report , Confidential report

True/False Question

Most of the Indian PS Banks practice retail banking.

Correct Answer True Your Answer

False

Multiple Choice Multiple Answer Question Prominent Institutional Investors are :Correct Answer Foreign Investors(FIIs) , Corporate Treasuries , Mutual Funds Your Answer Foreign Investors(FIIs) , Corporate Treasuries , Mutual Funds

Select The Blank Question The price in financial markets is known as ________. Correct Answer Rate of interest Your Answer

Rate of interest

Select The Blank Question Financial markets are the centres that provide facilities for buying and selling of ________. Correct Answer Financial claims and services Your Answer

Financial claims and services

Multiple Choice Single Answer Question Treasury Bills are actually a class of :Correct Answer Govt.Securities Your Answer

Govt.Securities

True/False Question

IMF discourages excessive use of its resources by imposing a surcharge on large loans. Correct Answer True Your Answer

True

Multiple Choice Single Answer Question Initial Quantum of resources of IMF were contributed by members according to :Correct Answer Quotas fixed for members Your Answer

Quotas fixed for members

Select The Blank Question The extent of powers enjoyed by a branch depends on the size of its ________. Correct Answer Business Your Answer

Deposits

Multiple Choice Single Answer

Question

The unorganised financial system includes :-

Correct Answer Indigenous bankers Your Answer

Indigenous bankers

Multiple Choice Single Answer Question Development Financial institutions include :Correct Answer State financial institutions Your Answer

State financial institutions

Multiple Choice Single Answer Question Deposits per depositor per bank are secured by DICGC up to a sum of :Correct Answer Rs. one lac. Your Answer

Rs. one lac.

Multiple Choice Multiple Answer Question The general insurance business can be :Correct Answer Marine insurance , Fire insurance , Miscellaneous Your Answer

Marine insurance , Fire insurance , Miscellaneous

Multiple Choice Multiple Answer Question The objective NEF Scheme of SIDBI is to provide equity support to :Correct Answer New Units in tiny Sector , New Units in Small Sector , Existing units in tiny & small scale sector Your Answer New Units in tiny Sector , New Units in Small Sector , Existing units in tiny & small scale sector Match The Following Question Correct Answer

Your Answer

CRR

Primary reserve requirement

Primary reserve requirement

Bankers Bank

Controlling the reserves of commercial banks FEMA

Controlling the reserves of commercial banks FEMA

Exchange control

OMO

Government securities Government securities

Multiple Choice Single Answer Question Financial institutions deal in :Correct Answer Financial assets Your Answer

Financial assets

Multiple Choice Multiple Answer Question Treasuries could be either those of the :Correct Answer PSUs , Private Sector Cos. , Govt.bodies Your Answer

PSUs , Private Sector Cos. , Govt.bodies

Multiple Choice Single Answer Question Long term debt instruments have a maturity of :Correct Answer Over one year Your Answer

Over one year

Select The Blank Question The Shipping Credit and Investment Company of India Ltd. has changed its name to ________ effective from October 1992. Correct Answer SCICI Ltd Your Answer

SCICI Ltd

True/False Question

A Bill of Exchange is negotiable.

Correct Answer True Your Answer

True

Multiple Choice Multiple Answer Question IDA lends to poor countries :Correct Answer By borrowing from IBRD , By lending at 0% interest. , For periods up to ten years Your Answer By lending at 0% interest. , For periods up to ten years

Multiple Choice Single Answer Question The organised financial system includes :Correct Answer Commercial banks Your Answer

Commercial banks

Multiple Choice Single Answer Question Liabilities of Insurance Cos are mostly :Correct Answer Long term Your Answer

Long term

Select The Blank Question SCICI provides domestic & foreign loans for capital equipment and________ services. Correct Answer Technical Your Answer

Technical

Multiple Choice Single Answer Question Development Financial Institutions provide finance to the needy Correct Answer Corporates & govt. institutions Your Answer

Corporates & govt. institutions

Select The Blank Question GIC raises funds to meet claims from ________. Correct Answer Premium collected Your Answer

Premium collected

True/False Question

The currency notes issued by RBI are legal tender everywhere in India without any limit. Correct Answer True Your Answer

True/False

True

Question

Issuance of CD attracts reserve requirements.

Correct Answer False Your Answer

True

True/False Question

IDA does not lend to countries that are not in a position to borrow from IBRD. Correct Answer False Your Answer

False

Multiple Choice Multiple Answer Question Mutual fund has main entities like :Correct Answer Sponsor , Trust , Asset Management company Your Answer

Sponsor , Trust , Investment bank

Multiple Choice Multiple Answer Question R B I issues currency notes against the security of:Correct Answer Gold bullion , Foreign securities , GOI securities Your Answer

Gold bullion , Foreign securities , GOI securities

True/False Question

The SFCs also borrow from SIDBI & IDBI.

Correct Answer False Your Answer

True

FI & B Binu -LIST OF ATTEMPTED QUESTIONS AND ANSWERS Multiple Choice Single Answer Question The type of banking involving a small number of Large Customers viz.Corporates & multinationals is :Correct Answer Wholesale Banking Your Answer Wholesale Banking Multiple Choice Single Answer Question PFC is committed to the integrated development of :Correct Answer Power Your Answer Power Multiple Choice Single Answer Question IDBI has helped in the establishment of :Correct Answer EXIM Bank Your Answer EXIM Bank Select The Blank Question NABARD took over the functions of ACD, RPCC of RBI & ________. Correct Answer ARDC Your Answer ARDC Multiple Choice Multiple Answer Question The major area of AFC's specialisation included Crop Production as well as:Correct Answer Crop Planning , Plant Protection , Farm Management Your Answer Crop Planning , Plant Protection , Farm Management True/False Question Banks issue Guarantees on behalf of their clients. Correct Answer True Your Answer False Select The Blank Question The HDFC was set up in 1977 by the ________. Correct Answer ICICI Your Answer ICICI Multiple Choice Multiple Answer Question The main objective of ICICI was :Correct Answer To augment export finance in India , To channelise World Bank's funds to industries in India , To help in building up a Capital Market in India Your Answer To augment export finance in India , To channelise World Bank's funds to industries in India , To help in building up a Capital Market in India Multiple Choice Multiple Answer Question ECGC Board comprises of Directors representing :Correct Answer Government , Banking/Insurance , Trade/Industry Your Answer Government , Banking/Insurance , Trade/Industry True/False Question Bank Guarantee can be Performance of financial Guarantee. Correct Answer False Your Answer False Multiple Choice Single Answer Question Which of the following institution was set up as a subsidy of RBI in 1964? Correct Answer IDBI

Your Answer

IDBI

Match The Following Question Correct Answer Your Answer Advances

Loans/cash credits

Cash withdrawal facility Remittances

ATM

Loans/cash credits

ATM

MT/DD/TT/EFT MT/DD/TT/EFT

Government business

Collection of taxes

Collection of taxes

Multiple Choice Multiple Answer Question The objective NEF Scheme of SIDBI is to provide equity support to :Correct Answer New Units in tiny Sector , New Units in Small Sector , Existing units in tiny & small scale sector Your Answer New Units in tiny Sector , New Units in Small Sector , Existing units in tiny & small scale sector Select The Blank Question 55% of HUDCO loans are earmarked for________& low income groups. Correct Answer Weaker sections Your Answer Agriculturists Select The Blank Question The ________ of NABARD covers release of refinance without prior sanction for refinance limit. Correct Answer ARS Your Answer CFSF Multiple Choice Multiple Answer Question Mutual fund has main entities like :Correct Answer Sponsor , Trust , Asset Management company Your Answer Sponsor , Trust , Asset Management company True/False Question EXIM Bank provides Lines of Credit to other nations for Indian exports. Correct Answer True Your Answer True Select The Blank Question Legally speaking Pass Through Certificates are ________ and are tradeable freely. Correct Answer Promissory notes Your Answer Promissory notes Multiple Choice Single Answer Question A record of the holdings of GOI dated securities of each holder is contained in :Correct Answer SGL pass book Your Answer SGL pass book Multiple Choice Single Answer Question The underdeveloped markets are characterised by :Correct Answer Government regulation and control Your Answer Government regulation and control Multiple Choice Multiple Answer Question Functions of R.O. of a bank include :Correct Answer Inspection/audit , Control on branches , Grant loans/advances Your Answer Inspection/audit , Control on branches , Grant loans/advances

Multiple Choice Single Answer Question Liquid assets to be maintained to a stipulated ratio under Section 24 of BR Act is called :Correct Answer Statutory Liquidity Ratio Your Answer Cash Reserves Ratio Select The Blank Question In a Bank, the investment decisions are taken at ________ level. Correct Answer Head Office Your Answer Head Office Multiple Choice Multiple Answer Question RBI sells securities through :Correct Answer Auction program , Open market operations , Private placements Your Answer Auction program , Open market operations , Private placements Multiple Choice Multiple Answer Question R B I conducts monetary control by way of O M O in :Correct Answer Treasury bills , State /central Government Securities Your Answer Treasury bills , State /central Government Securities True/False Question The SFCs also borrow from SIDBI & IDBI. Correct Answer False Your Answer False True/False Question No TDS is deducted on Interest paid on CDs. Correct Answer True Your Answer True Multiple Choice Single Answer Question Corporate Debentures are :Correct Answer Long term debt instruments Your Answer Short term deposit receipts True/False Question Amongst others, ICICI also provided long and medium term loans or equity finance. Correct Answer True Your Answer True Multiple Choice Single Answer Question RBI is also referred to as :Correct Answer Apex Bank Your Answer Apex Bank True/False Question HDFC was set up by NHB. Correct Answer False Your Answer False True/False Question IDA does not lend to countries that are not in a position to borrow from IBRD. Correct Answer False Your Answer False Multiple Choice Multiple Answer Question SFCs provide finance to :-

Correct Answer Corporates , Partnerships , Proprietary firms Your Answer Corporates , Partnerships , Proprietary firms Select The Blank Question AFC's membership consists of commercial, cooperative banks and DFIs including ________. Correct Answer EXIM bank Your Answer EXIM bank Multiple Choice Multiple Answer Question Treasuries could be either those of the :Correct Answer PSUs , Private Sector Cos. , Govt.bodies Your Answer PSUs , Private Sector Cos. , Govt.bodies Select The Blank Question The ________ are buy and hold investors and almost never trade on their investments. Correct Answer Provident Funds Your Answer Foreign Investors(FIIs) True/False Question The bids for T-Bills are on price/interest basis. Correct Answer True Your Answer True Multiple Choice Multiple Answer Question IFC provides financial assistance :Correct Answer Through loans , Through purchase of shares , For economic development Your Answer Through loans , Through purchase of shares , For economic development Multiple Choice Single Answer Question Money Markets Instruments have a maturity of less than :Correct Answer One year Your Answer One year Multiple Choice Single Answer Question Treasury Bills are actually a class of :Correct Answer Govt.Securities Your Answer Govt.Securities True/False Question NABARD provides short term refinance for procurement activities also. Correct Answer True Your Answer False Match The Following Question Correct Answer Your Answer GOI securities

ZCB, ILBonds

ZCB, ILBonds

Long Term Debt instruments

PSU Bonds

PSU Bonds

Money Market Instruments

CP, T- Bills

CP, T- Bills

Negotaible Instruments

Bills of exchange ,Cheques

Multiple Choice Single Answer Question IRBI was set up with a focus on :Correct Answer Sick Units

Bills of exchange ,Cheques

Your Answer

Public sector

Select The Blank Question The ________ was set up to help corrections in BOP maladjustments. Correct Answer IMF Your Answer IBRD Select The Blank Question Mutual Funds provide the benefits of ________. Correct Answer Portfolio management Your Answer High returns Multiple Choice Single Answer Question Financial institutions deal in :Correct Answer Financial assets Your Answer Financial assets Multiple Choice Multiple Answer Question Duties of an International Banking unit cover :Correct Answer Handling foreign currency , Handling Doc. credits , Granting lines of Credit Your Answer Handling foreign currency , Handling Doc. credits , Granting lines of Credit

LIST OF ATTEMPTED QUESTIONS AND ANSWERS Select The Blank Question The Shipping Credit and Investment Company of India Ltd. has changed its name to ________ effective from October 1992. Correct Answer SCICI Ltd Your Answer SCICI Ltd Select The Blank Question AFC's membership consists of commercial, cooperative banks and DFIs including ________. Correct Answer EXIM bank Your Answer EXIM bank Multiple Choice Single Answer Question Financial institutions deal in :Correct Answer Financial assets Your Answer Financial assets Multiple Choice Single Answer Question The lender of the last resort to the market! is the :Correct Answer RBI Your Answer RBI Select The Blank Question One of the objectives of NHB is to augment the ________ resources for the housing sector. Correct Answer Financial Your Answer Financial Multiple Choice Multiple Answer Question IDA lends to poor countries :Correct Answer By borrowing from IBRD , By lending at 0% interest. , For periods up to ten years Your Answer By borrowing from IBRD , By lending at 0% interest. , For periods up to ten years Multiple Choice Single Answer Question A company issues CP to save on :Correct Answer Interest cost Your Answer Interest cost True/False Question PFC has been persuading State Governments restructuring of their power sector to make them viable.

Correct Answer True Your Answer True Multiple Choice Single Answer Question The ultimate source of money in India is:Correct Answer RBI Your Answer RBI True/False Question The SFCs also borrow from SIDBI & IDBI. Correct Answer False Your Answer True Multiple Choice Single Answer Question RBI was established under the:Correct Answer RBI Act Your Answer RBI Act Multiple Choice Multiple Answer Question The major area of AFC's specialisation included Crop Production as well as:Correct Answer Crop Planning , Plant Protection , Farm Management Your Answer Crop Planning , Plant Protection , Farm Management Multiple Choice Multiple Answer Question A member's IMF Quota determines its :Correct Answer Financial commitment , Voting power , Drawing power Your Answer Financial commitment , Voting power , Drawing power Select The Blank Question Period of Holiday Insurance is ________. Correct Answer Six months or less Your Answer Three years Multiple Choice Single Answer Question The underdeveloped markets are characterised by :Correct Answer Government regulation and control Your Answer Government reg! ulation and control Multiple Choice Single Answer Question Cetificate of deposit are issued by :Correct Answer Banks Your Answer Banks

True/False Question The bids for T-Bills are on price/interest basis. Correct Answer True Your Answer True Multiple Choice Single Answer Question The unorganised financial system includes :Correct Answer Indigenous bankers Your Answer Indigenous bankers Multiple Choice Single Answer Question Long term debt instruments have a maturity of :Correct Answer Over one year Your Answer Over one year Multiple Choice Multiple Answer Question The general insurance business can be :Correct Answer Marine insurance , Fire insurance , Miscellaneous Your Answer Fire insurance , Miscellaneous , Marine insurance True/False Question NABARD does not provide Refinance for stocking & distribution of Chemical Fertilisers. Correct Answer False Your Answer False Multiple Choice Multiple Answer Question The unorganised financial system comprises of :Correct Answer Money lenders , Indigenous bankers , Lending pawn brokers Your Answer Money lenders , Indigenous bankers , Lending pawn brokers True/False Question NABARD is an apex institution in the field of Integrated Rural Development. &nbs! p; Correct Answer True Your Answer True Select The Blank Question 55% of HUDCO loans are earmarked for________& low income groups. Correct Answer Weaker sections Your Answer Weaker sections

Multiple Choice Single Answer Question Development Financial institutions include :Correct Answer State financial institutions Your Answer State financial institutions Multiple Choice Single Answer Question Money Markets Instruments have a maturity of less than :Correct Answer One year Your Answer One year Select The Blank Question The price in financial markets is known as ________. Correct Answer &n! bsp; Rate of interest Your Answer Rate of interest Match The Following Question

Correct Answer

Development financial institutions

IFCI

Organised sector

Banking System

Unorganised sector

Indigenous banks

Mutual funds

UTI

Select The Blank Question The HDFC was set up in 1977 by the ________. Correct Answer ICICI Your Answer ICICI Multiple Choice Single Answer Question Certificate of deposit is a :Correct Answer Usance promissory note Your Answer Term Deposit Receipt True/False Question A Bill of Exchange is negotiable. Correct Answer True Your Answer True Multiple Choice Single Answer Question Commercial Papers are issued by :-

Your Answer IFCI Banking System Indigenous banks UTI

Correct Answer Corporates Your Answer Corporates Multiple Choice Single Answer Question A current account where the customer can withdraw more than the balance is :Correct Answer Overdraft Your Answer Overdraft Multiple Choice Multiple Answer Question The loans of finance companies are generally :Correct Answer At high interest , Unsecured , Based on borrowers worth Your Answer At high interest , Unsecured , Based on borrowers worth Multiple Choice Single Answer Question For a focus on the SSI Sector in 1986 IDBI created :Correct Answer SIDF Your Answer SIDF True/False Question Issuance of CD attracts reserve requirements. Correct Answer False Your Answer False True/False Question Banks issue Guarantees on behalf of their clients. Correct Answer True Your Answer True Match The Following Question

Correct Answer

NABARD

Rural Development

AFC Ltd

Consultancy

HUDCO

Housing

NCDC

Your Answer Rural Development Consultancy Housing

Development of Cooperatives Development of Cooperatives

True/False Question Under MIGA the World bank offers various forms of Political Insurance. Correct Answer True Your Answer True

Multiple Choice Multiple Answer Question In case of a Life policy it is possible to withdraw from the obligation to contribute further premium by :Correct Answer Surrendering the policy , Converting it into paid up policy , Assigning it in the open market Your Answer Surrendering the policy , Converting it into paid up policy , Assigning it in the open market Multiple Choice Single Answer Question The organised financial system includes :Correct Answer Commercial banks Your Answer Commercial banks Select The Blank Question The term ________ refers to the act of a bank extending finance to the seller against a letter of credit. Correct Answer Negotiation Your Answer Negotiation Multiple Choice Multiple Answer Question Functions of a Bank's H.O. include:Correct Answer Liaison with RBI , Evolve Policies , Vigilance & Control Your Answer Liaison with RBI , Evolve Policies , Vigilance & Control Multiple Choice Multiple Answer Question Bonds issued by institutions through SPVs to execute infrastructure projects are :Correct Answer MSR Dev.Corporation , MKV Dev. Corporation. , SSN Nigam Ltd Your Answer MSR Dev.Corporation , MKV Dev. Corporation. , SSN Nigam Ltd Select The Blank Question GIC raises funds to meet claims from ________. Correct Answer Premium collected Your Answer Premium collected ! Multiple Choice Multiple Answer Question Treasuries could be either those of the :Correct Answer PSUs , Private Sector Cos. , Govt.bodies Your Answer PSUs , Private Sector Cos. , Govt.bodies Multiple Choice Multiple Answer Question Under Project Finance Scheme IDBI provides assistance for :-

Correct Answer New Projects , Renovation , Modernisation Your Answer New Projects , Renovation , Modernisation Select The Blank Question The Shipping Credit and Investment Company of India Ltd. has changed its name to ________ effective from October 1992. Correct Answer SCICI Ltd Your Answer SCICI Ltd Select The Blank Question AFC's membership consists of commercial, cooperative banks and DFIs including ________. Correct Answer EXIM bank Your Answer EXIM bank Multiple Choice Single Answer Question Financial institutions deal in :Correct Answer Financial assets Your Answer Financial assets Multiple Choice Single Answer Question The lender of the last resort to the market is the :Correct Answer RBI Your Answer RBI Select The Blank Question One of the objectives of NHB is to augment the ________ resources for the housing sector. Correct Answer Financial Your Answer Financial Multiple Choice Multiple Answer Question IDA lends to poor countries :Correct Answer By borrowing from IBRD , By lending at 0% interest. , For periods up to ten years Your Answer By borrowing from IBRD , By lending at 0% interest. , For periods up to ten years Multiple Choice Single Answer Question A company issues CP to save on :Correct Answer Interest cost Your Answer Interest cost

True/False Question PFC has been persuading State Governments restructuring of their power sector to make them viable. Correct Answer True Your Answer True Multiple Choice Single Answer Question The ultimate source of money in India is:Correct Answer RBI Your Answer RBI True/False Question The SFCs also borrow from SIDBI & IDBI. Correct Answer False Your Answer True Multiple Choice Single Answer Question RBI was established under the:Correct Answer RBI Act Your Answer RBI Act Multiple Choice Multiple Answer Question The major area of AFC's specialisation included Crop Production as well as:Correct Answer Crop Planning , Plant Protection , Farm Management Your Answer Crop Planning , Plant Protection , Farm Management Multiple Choice Multiple Answer Question A member's IMF Quota determines its :Correct Answer Financial commitment , Voting power , Drawing power Your Answer Financial commitment , Voting power , Drawing power Select The Blank Question Period of Holiday Insurance is ________. Correct Answer Six months or less Your Answer Three years Multiple Choice Single Answer Question The underdeveloped markets are characterised by :Correct Answer Government regulation and control Your Answer Government regulation and control

Multiple Choice Single Answer Question Cetificate of deposit are issued by :Correct Answer Banks Your Answer Banks True/False Question The bids for T-Bills are on price/interest basis. Correct Answer True Your Answer True Multiple Choice Single Answer Question The unorganised financial system includes :Correct Answer Indigenous bankers Your Answer Indigenous bankers Multiple Choice Single Answer Question Long term debt instruments have a maturity of :Correct Answer Over one year Your Answer Over one year Multiple Choice Multiple Answer Question The general insurance business can be :Correct Answer Marine insurance , Fire insurance , Miscellaneous Your Answer Fire insurance , Miscellaneous , Marine insurance True/False Question NABARD does not provide Refinance for stocking & distribution of Chemical Fertilisers. Correct Answer False Your Answer False Multiple Choice Multiple Answer Question The unorganised financial system comprises of :Correct Answer Money lenders , Indigenous bankers , Lending pawn brokers Your Answer Money lenders , Indigenous bankers , Lending pawn brokers True/False Question NABARD is an apex institution in the field of Integrated Rural Development. Correct Answer True Your Answer True Select The Blank Question 55% of HUDCO loans are earmarked for________& low income groups.

Correct Answer Weaker sections Your Answer Weaker sections Multiple Choice Single Answer Question Development Financial institutions include :Correct Answer State financial institutions Your Answer State financial institutions Multiple Choice Single Answer Question Money Markets Instruments have a maturity of less than :Correct Answer One year Your Answer One year Select The Blank Question The price in financial markets is known as ________. Correct Answer Rate of interest Your Answer Rate of interest Match The Following Question

Correct Answer

Development financial institutions

IFCI

Your Answer IFCI

Organised sector

Banking System

Banking System

Unorganised sector

Indigenous banks

Indigenous banks

UTI

UTI

Mutual funds

Select The Blank Question The HDFC was set up in 1977 by the ________. Correct Answer ICICI Your Answer ICICI Multiple Choice Single Answer Question Certificate of deposit is a :Correct Answer Usance promissory note Your Answer Term Deposit Receipt True/False Question A Bill of Exchange is negotiable. Correct Answer True Your Answer True

Multiple Choice Single Answer Question Commercial Papers are issued by :Correct Answer Corporates Your Answer Corporates Multiple Choice Single Answer Question A current account where the customer can withdraw more than the balance is :Correct Answer Overdraft Your Answer Overdraft Multiple Choice Multiple Answer Question The loans of finance companies are generally :Correct Answer At high interest , Unsecured , Based on borrowers worth Your Answer At high interest , Unsecured , Based on borrowers worth Multiple Choice Single Answer Question For a focus on the SSI Sector in 1986 IDBI created :Correct Answer SIDF Your Answer SIDF True/False Question Issuance of CD attracts reserve requirements. Correct Answer False Your Answer False True/False Question Banks issue Guarantees on behalf of their clients. Correct Answer True Your Answer True True/False Question Under MIGA the World bank offers various forms of Political Insurance. Correct Answer True Your Answer True Multiple Choice Multiple Answer Question In case of a Life policy it is possible to withdraw from the obligation to contribute further premium by :-

Correct Answer Surrendering the policy , Converting it into paid up policy , Assigning it in the open market Your Answer Surrendering the policy , Converting it into paid up policy , Assigning it in the open market Multiple Choice Single Answer Question The organised financial system includes :Correct Answer Commercial banks Your Answer Commercial banks Select The Blank Question The term ________ refers to the act of a bank extending finance to the seller against a letter of credit. Correct Answer Negotiation Your Answer Negotiation Multiple Choice Multiple Answer Question Functions of a Bank's H.O. include:Correct Answer Liaison with RBI , Evolve Policies , Vigilance & Control Your Answer Liaison with RBI , Evolve Policies , Vigilance & Control Multiple Choice Multiple Answer Question Bonds issued by institutions through SPVs to execute infrastructure projects are :Correct Answer MSR Dev.Corporation , MKV Dev. Corporation. , SSN Nigam Ltd Your Answer MSR Dev.Corporation , MKV Dev. Corporation. , SSN Nigam Ltd Select The Blank Question GIC raises funds to meet claims from ________. Correct Answer Premium collected Your Answer Premium collected Multiple Choice Multiple Answer Question Treasuries could be either those of the :Correct Answer PSUs , Private Sector Cos. , Govt.bodies Your Answer PSUs , Private Sector Cos. , Govt.bodies Multiple Choice Multiple Answer Question Under Project Finance Scheme IDBI provides assistance for :Correct Answer New Projects , Renovation , Modernisation Your Answer New Projects , Renovation , Modernisation Select The Blank Question Kisan Vikas Patra are issued through ________.

Correct Answer Post Offices Your Answer Post Offices Multiple Choice Single Answer Question For a focus on the SSI Sector in 1986 IDBI created :Correct Answer SIDF Your Answer SIDF Match The Following Question

Correct Answer

Corporates NGOs DFI PSUs

RIL, HPCL RIL, HPCL CSP, CRY, PLAN CSP, CRY, PLAN IDBI, IFCI, ICICI IDBI, IFCI, ICICI NTPC, NHPC, Coal India NTPC, NHPC, Coal India

Your Answer

Multiple Choice Multiple Answer Question DFHI was set up jointly by :Correct Answer RBI , Banks , All India Fis Your Answer RBI , Banks , All India Fis , SEBI Select The Blank Question In addition to IFCI and ________ the Soft Loan Scheme for modernisation is also undertaken by IDBI. Correct Answer ICICI Your Answer ICICI Multiple Choice Single Answer Question The unorganised financial system includes :Correct Answer Indigenous bankers Your Answer Indigenous bankers Multiple Choice Single Answer Question Investment on a monthly basis and repayment on due date is made in :Correct Answer Recurring deposit Your Answer Term deposit account Multiple Choice Multiple Answer Question Tourism promotes :Correct Answer Credit Card Business , International remittances , Travellers' Cheques Your Answer Credit Card Business , International remittances , Travellers' Cheques

Multiple Choice Single Answer Question A company issues CP to save on :Correct Answer Interest cost Your Answer Taxes Multiple Choice Single Answer Question A current account where the customer can withdraw more than the balance is :Correct Answer Overdraft Your Answer Overdraft Select The Blank Question AFC's membership consists of commercial, cooperative banks and DFIs including ________. Correct Answer EXIM bank Your Answer EXIM bank Multiple Choice Multiple Answer Question Functions of R.O. do not cover :Correct Answer SLR/CRR maintenance , Liaison with RBI/IBA , Issuing Lending Policy Your Answer Liaison with RBI/IBA , Issuing Lending Policy Select The Blank Question The ________ of NABARD covers release of refinance without prior sanction for refinance limit. Correct Answer ARS Your Answer CFSF Select The Blank Question The ________ regulation makes the banks the largest investor in the GOI securities. Correct Answer SLR Your Answer SLR Multiple Choice Multiple Answer Question Post Offices are popular for mobilizing small savings of the public due to :Correct Answer Wide network , Tax concessions , Saftey of funds Your Answer Wide network , Tax concessions , Saftey of funds True/False Question The SFCs also borrow from SIDBI & IDBI.

Correct Answer False Your Answer True Multiple Choice Single Answer Question SIDBI commenced its operations from :Correct Answer April 2,1990 Your Answer April 2,1990 Multiple Choice Single Answer Question The ultimate source of money in India is:Correct Answer RBI Your Answer RBI Multiple Choice Single Answer Question The organised financial system includes :Correct Answer Commercial banks Your Answer Financial companies Multiple Choice Multiple Answer Question IDA lends to poor countries :Correct Answer By borrowing from IBRD , By lending at 0% interest. , For periods up to ten years Your Answer By borrowing from IBRD , By lending at 0% interest. , For periods up to ten years Multiple Choice Multiple Answer Question The prominent Bond issuing PSU are :Correct Answer MTNL , NTPC , SAIL Your Answer MTNL , NTPC , SAIL Match The Following Question

Correct Answer

Your Answer

Hill ,Tribal Area development

Area Development

Area Development

Brackish Water

Fisheries Fisheries

Aquaculture ,

Ploughing, weeding, transplantation Seasonal Agricultural Operations Market Yards, Storage & Warehousing

Agricultural Marketing

Irrigation & Drainage Agricultural Marketing

True/False Question Small Savings are source of Capital Receipts for the Government.

Correct Answer True Your Answer False Multiple Choice Multiple Answer Question The financial Assistance provided by the IMF enables member countries to :Correct Answer Rebuild resources , Stabilise currencies , Pay for imports Your Answer Rebuild resources , Stabilise currencies , Undertake specific projects Multiple Choice Single Answer Question Cetificate of deposit are issued by :Correct Answer Banks Your Answer Banks Multiple Choice Single Answer Question IDBI has helped in the establishment of :Correct Answer EXIM Bank Your Answer ICICI Select The Blank Question NHB at present has a ________ Capital of Rs. 350 crores. Correct Answer Paid-up Your Answer Paid-up True/False Question The Capital of NABARD is subscribed by World Bank. Correct Answer False Your Answer False Select The Blank Question In a Bank, the investment decisions are taken at ________ level. Correct Answer Head Office Your Answer Head Office Multiple Choice Single Answer Question RBI is also referred to as :Correct Answer Apex Bank Your Answer Apex Bank Multiple Choice Multiple Answer Question EXIM Bank can raise resources by:Correct Answer Issuing bonds , Borrowing from RBI , Borrowing from International Financial Institutions

Your Answer Borrowing from RBI , Borrowing from International Financial Institutions Multiple Choice Single Answer Question PFC is committed to the integrated development of :Correct Answer Power Your Answer Power True/False Question PFC has been persuading State Governments restructuring of their power sector to make them viable. Correct Answer True Your Answer True Multiple Choice Multiple Answer Question SIDBI provides :Correct Answer Refinance , Rediscounting of Bills , Financial support to SSIDC Your Answer Refinance , Financial support to SSIDC , Cash Credit facilities True/False Question Under MIGA the World bank offers various forms of Political Insurance. Correct Answer True Your Answer True Multiple Choice Multiple Answer Question Other than Tourism, International funds remittances comprise of :Correct Answer Donations/Charity , Payments of Salaries , Payments of Royalty Your Answer Payments of Royalty , Travel related payments True/False Question SCICI is a member of the OTCEI. Correct Answer True Your Answer False Select The Blank Question HDFCs loans were linked up with________. Correct Answer Planned Savings Your Answer Planned Savings True/False Question A Bill of Exchange is negotiable. Correct Answer True Your Answer True

True/False Question HDFC was set up by NHB. Correct Answer False Your Answer False Select The Blank Question Mutual Funds provide the benefits of ________. Correct Answer Portfolio management Your Answer Portfolio management True/False Question In case of T-Bills potential investors have to put in competitive bids. Correct Answer True Your Answer True True/False Question Non bank financial institutions undertake fund and non fund based activities. Correct Answer True Your Answer True Multiple Choice Multiple Answer Question Functions of R.O. of a bank include :Correct Answer Inspection/audit , Control on branches , Grant loans/advances Your Answer Inspection/audit , Control on branches , Grant loans/advances Multiple Choice Multiple Answer Question Duties of an International Banking unit cover :Correct Answer Handling foreign currency , Handling Doc. credits , Granting lines of Credit Your Answer Maintaining SLR/CRR , Handling foreign currency , Handling Doc. credits , Granting lines of Credit Select The Blank Question Secured Premium Notes are ________. Correct Answer Debentures Your Answer Debentures Select The Blank Question Syndication involves ________ sanction. Correct Answer In principle Your Answer Funded

Financial Institutions and Banking_6 Select The Blank Question Kisan Vikas Patra are issued through ________. Correct Answer Post Offices Your Answer Post Offices Multiple Choice Single Answer Question For a focus on the SSI Sector in 1986 IDBI created :Correct Answer SIDF Your Answer SIDF Match The Following Question Correct Answer Your Answer Corporates RIL, HPCL RIL, HPCL NGOs CASP, CRY, PLAN CASP, CRY, PLAN DFI IDBI, IFCI, ICICI IDBI, IFCI, ICICI PSUs NTPC, NHPC, Coal India NTPC, NHPC, Coal India Multiple Choice Multiple Answer Question DFHI was set up jointly by :Correct Answer RBI , Banks , All India Fis Your Answer RBI , Banks , All India Fis , SEBI Select The Blank Question In addition to IFCI and ________ the Soft Loan Scheme for modernisation is also undertaken by IDBI. Correct Answer ICICI Your Answer ICICI Multiple Choice Single Answer Question The unorganised financial system includes :Correct Answer Indigenous bankers Your Answer Indigenous bankers Multiple Choice Single Answer Question Investment on a monthly basis and repayment on due date is made in :Correct Answer Recurring deposit Page 1

Financial Institutions and Banking_6 Your Answer Term deposit account Multiple Choice Multiple Answer Question Tourism promotes :Correct Answer Credit Card Business , International remittances , Travellers' Cheques Your Answer Credit Card Business , International remittances , Travellers' Cheques Multiple Choice Single Answer Question A company issues CP to save on :Correct Answer Interest cost Your Answer Taxes Multiple Choice Single Answer Question A current account where the customer can withdraw more than the balance is :Correct Answer Overdraft Your Answer Overdraft Select The Blank Question AFC's membership consists of commercial, cooperative banks and DFIs including ________. Correct Answer EXIM bank Your Answer EXIM bank Multiple Choice Multiple Answer Question Functions of R.O. do not cover :Correct Answer SLR/CRR maintenance , Liaison with RBI/IBA , Issuing Lending Policy Your Answer Liaison with RBI/IBA , Issuing Lending Policy Select The Blank Question The ________ of NABARD covers release of refinance without prior sanction for refinance limit. Correct Answer ARS Your Answer CFSF Select The Blank Question The ________ regulation makes the banks the largest investor in the GOI securities. Correct Answer SLR Page 2

Financial Institutions and Banking_6 Your Answer SLR Multiple Choice Multiple Answer Question Post Offices are popular for mobilizing small savings of the public due to :Correct Answer Wide network , Tax concessions , Saftey of funds Your Answer Wide network , Tax concessions , Saftey of funds True/False Question The SFCs also borrow from SIDBI & IDBI. Correct Answer False Your Answer True Multiple Choice Single Answer Question SIDBI commenced its operations from :Correct Answer April 2,1990 Your Answer April 2,1990 Multiple Choice Single Answer Question The ultimate source of money in India is:Correct Answer RBI Your Answer RBI Multiple Choice Single Answer Question The organised financial system includes :Correct Answer Commercial banks Your Answer Financial companies Multiple Choice Multiple Answer Question IDA lends to poor countries :Correct Answer By borrowing from IBRD , By lending at 0% interest. , For periods up to ten years Your Answer By borrowing from IBRD , By lending at 0% interest. , For periods up to ten years Multiple Choice Multiple Answer Question The prominent Bond issuing PSU are :Correct Answer MTNL , NTPC , SAIL Your Answer MTNL , NTPC , SAIL Match The Following Question Correct Answer Your Answer Page 3

Financial Institutions and Banking_6 Hill ,Tribal Area development Area Development Area Development Aquaculture ,Brackish Water Fisheries Fisheries Ploughing, weeding, transplantation Seasonal Agricultural Operations Irrigation & Drainage Market Yards, Storage & Warehousing Agricultural Marketing Agricultural Marketing True/False Question Small Savings are source of Capital Receipts for the Government. Correct Answer True Your Answer False Multiple Choice Multiple Answer Question The financial Assistance provided by the IMF enables member countries to :Correct Answer Rebuild resources , Stabilise currencies , Pay for imports Your Answer Rebuild resources , Stabilise currencies , Undertake specific projects Multiple Choice Single Answer Question Cetificate of deposit are issued by :Correct Answer Banks Your Answer Banks Multiple Choice Single Answer Question IDBI has helped in the establishment of :Correct Answer EXIM Bank Your Answer ICICI Select The Blank Question NHB at present has a ________ Capital of Rs. 350 crores. Correct Answer Paid-up Your Answer Paid-up True/False Page 4

Financial Institutions and Banking_6 Question The Capital of NABARD is subscribed by World Bank. Correct Answer False Your Answer False Select The Blank Question In a Bank, the investment decisions are taken at ________ level. Correct Answer Head Office Your Answer Head Office Multiple Choice Single Answer Question RBI is also referred to as :Correct Answer Apex Bank Your Answer Apex Bank Multiple Choice Multiple Answer Question EXIM Bank can raise resources by:Correct Answer Issuing bonds , Borrowing from RBI , Borrowing from International Financial Institutions Your Answer Borrowing from RBI , Borrowing from International Financial Institutions Multiple Choice Single Answer Question PFC is committed to the integrated development of :Correct Answer Power Your Answer Power True/False Question PFC has been persuading State Governments restructuring of their power sector to make them viable. Correct Answer True Your Answer True Multiple Choice Multiple Answer Question SIDBI provides :Correct Answer Refinance , Rediscounting of Bills , Financial support to SSIDC Your Answer Refinance , Financial support to SSIDC , Cash Credit facilities True/False Question Under MIGA the World bank offers various forms of Page 5

Financial Institutions and Banking_6 Political Insurance. Correct Answer True Your Answer True Multiple Choice Multiple Answer Question Other than Tourism, International funds remittances comprise of :Correct Answer Donations/Charity , Payments of Salaries , Payments of Royalty Your Answer Payments of Royalty , Travel related payments True/False Question SCICI is a member of the OTCEI. Correct Answer True Your Answer False Select The Blank Question HDFCs loans were linked up with________. Correct Answer Planned Savings Your Answer Planned Savings True/False Question A Bill of Exchange is negotiable. Correct Answer True Your Answer True True/False Question HDFC was set up by NHB. Correct Answer False Your Answer False Select The Blank Question Mutual Funds provide the benefits of ________. Correct Answer Portfolio management Your Answer Portfolio management True/False Question In case of T-Bills potential investors have to put in competitive bids. Correct Answer True Your Answer True Page 6

Financial Institutions and Banking_6 True/False Question Non bank financial institutions undertake fund and non fund based activities. Correct Answer True Your Answer True Multiple Choice Multiple Answer Question Functions of R.O. of a bank include :Correct Answer Inspection/audit , Control on branches , Grant loans/advances Your Answer Inspection/audit , Control on branches , Grant loans/advances Multiple Choice Multiple Answer Question Duties of an International Banking unit cover :Correct Answer Handling foreign currency , Handling Doc. credits , Granting lines of Credit Your Answer Maintaining SLR/CRR , Handling foreign currency , Handling Doc. credits , Granting lines of Credit Select The Blank Question Secured Premium Notes are ________. Correct Answer Debentures Your Answer Debentures Select The Blank Question Syndication involves ________ sanction. Correct Answer In principle Your Answer Funded

Page 7

FI & B Binu -LIST OF ATTEMPTED QUESTIONS AND ANSWERS Multiple Choice Single Answer Question The type of banking involving a small number of Large Customers viz.Corporates & multinationals is :Correct Answer Wholesale Banking Your Answer Wholesale Banking Multiple Choice Single Answer Question PFC is committed to the integrated development of :Correct Answer Power Your Answer Power Multiple Choice Single Answer Question IDBI has helped in the establishment of :Correct Answer EXIM Bank Your Answer EXIM Bank Select The Blank Question NABARD took over the functions of ACD, RPCC of RBI & ________. Correct Answer ARDC Your Answer ARDC Multiple Choice Multiple Answer Question The major area of AFC's specialisation included Crop Production as well as:Correct Answer Crop Planning , Plant Protection , Farm Management Your Answer Crop Planning , Plant Protection , Farm Management True/False Question Banks issue Guarantees on behalf of their clients. Correct Answer True Your Answer False Select The Blank Question The HDFC was set up in 1977 by the ________. Correct Answer ICICI Your Answer ICICI Multiple Choice Multiple Answer Question The main objective of ICICI was :Correct Answer To augment export finance in India , To channelise World Bank's funds to industries in India , To help in building up a Capital Market in India Your Answer To augment export finance in India , To channelise World Bank's funds to industries in India , To help in building up a Capital Market in India Multiple Choice Multiple Answer Question ECGC Board comprises of Directors representing :Correct Answer Government , Banking/Insurance , Trade/Industry Your Answer Government , Banking/Insurance , Trade/Industry True/False Question Bank Guarantee can be Performance of financial Guarantee. Correct Answer False Your Answer False Multiple Choice Single Answer Question Which of the following institution was set up as a subsidy of RBI in 1964? Correct Answer IDBI

Your Answer

IDBI

Match The Following Question Correct Answer Your Answer Advances

Loans/cash credits

Cash withdrawal facility Remittances

ATM

Loans/cash credits

ATM

MT/DD/TT/EFT MT/DD/TT/EFT

Government business

Collection of taxes

Collection of taxes

Multiple Choice Multiple Answer Question The objective NEF Scheme of SIDBI is to provide equity support to :Correct Answer New Units in tiny Sector , New Units in Small Sector , Existing units in tiny & small scale sector Your Answer New Units in tiny Sector , New Units in Small Sector , Existing units in tiny & small scale sector Select The Blank Question 55% of HUDCO loans are earmarked for________& low income groups. Correct Answer Weaker sections Your Answer Agriculturists Select The Blank Question The ________ of NABARD covers release of refinance without prior sanction for refinance limit. Correct Answer ARS Your Answer CFSF Multiple Choice Multiple Answer Question Mutual fund has main entities like :Correct Answer Sponsor , Trust , Asset Management company Your Answer Sponsor , Trust , Asset Management company True/False Question EXIM Bank provides Lines of Credit to other nations for Indian exports. Correct Answer True Your Answer True Select The Blank Question Legally speaking Pass Through Certificates are ________ and are tradeable freely. Correct Answer Promissory notes Your Answer Promissory notes Multiple Choice Single Answer Question A record of the holdings of GOI dated securities of each holder is contained in :Correct Answer SGL pass book Your Answer SGL pass book Multiple Choice Single Answer Question The underdeveloped markets are characterised by :Correct Answer Government regulation and control Your Answer Government regulation and control Multiple Choice Multiple Answer Question Functions of R.O. of a bank include :Correct Answer Inspection/audit , Control on branches , Grant loans/advances Your Answer Inspection/audit , Control on branches , Grant loans/advances

Multiple Choice Single Answer Question Liquid assets to be maintained to a stipulated ratio under Section 24 of BR Act is called :Correct Answer Statutory Liquidity Ratio Your Answer Cash Reserves Ratio Select The Blank Question In a Bank, the investment decisions are taken at ________ level. Correct Answer Head Office Your Answer Head Office Multiple Choice Multiple Answer Question RBI sells securities through :Correct Answer Auction program , Open market operations , Private placements Your Answer Auction program , Open market operations , Private placements Multiple Choice Multiple Answer Question R B I conducts monetary control by way of O M O in :Correct Answer Treasury bills , State /central Government Securities Your Answer Treasury bills , State /central Government Securities True/False Question The SFCs also borrow from SIDBI & IDBI. Correct Answer False Your Answer False True/False Question No TDS is deducted on Interest paid on CDs. Correct Answer True Your Answer True Multiple Choice Single Answer Question Corporate Debentures are :Correct Answer Long term debt instruments Your Answer Short term deposit receipts True/False Question Amongst others, ICICI also provided long and medium term loans or equity finance. Correct Answer True Your Answer True Multiple Choice Single Answer Question RBI is also referred to as :Correct Answer Apex Bank Your Answer Apex Bank True/False Question HDFC was set up by NHB. Correct Answer False Your Answer False True/False Question IDA does not lend to countries that are not in a position to borrow from IBRD. Correct Answer False Your Answer False Multiple Choice Multiple Answer Question SFCs provide finance to :-

Correct Answer Corporates , Partnerships , Proprietary firms Your Answer Corporates , Partnerships , Proprietary firms Select The Blank Question AFC's membership consists of commercial, cooperative banks and DFIs including ________. Correct Answer EXIM bank Your Answer EXIM bank Multiple Choice Multiple Answer Question Treasuries could be either those of the :Correct Answer PSUs , Private Sector Cos. , Govt.bodies Your Answer PSUs , Private Sector Cos. , Govt.bodies Select The Blank Question The ________ are buy and hold investors and almost never trade on their investments. Correct Answer Provident Funds Your Answer Foreign Investors(FIIs) True/False Question The bids for T-Bills are on price/interest basis. Correct Answer True Your Answer True Multiple Choice Multiple Answer Question IFC provides financial assistance :Correct Answer Through loans , Through purchase of shares , For economic development Your Answer Through loans , Through purchase of shares , For economic development Multiple Choice Single Answer Question Money Markets Instruments have a maturity of less than :Correct Answer One year Your Answer One year Multiple Choice Single Answer Question Treasury Bills are actually a class of :Correct Answer Govt.Securities Your Answer Govt.Securities True/False Question NABARD provides short term refinance for procurement activities also. Correct Answer True Your Answer False Match The Following Question Correct Answer Your Answer GOI securities

ZCB, ILBonds

ZCB, ILBonds

Long Term Debt instruments

PSU Bonds

PSU Bonds

Money Market Instruments

CP, T- Bills

CP, T- Bills

Negotaible Instruments

Bills of exchange ,Cheques

Multiple Choice Single Answer Question IRBI was set up with a focus on :Correct Answer Sick Units

Bills of exchange ,Cheques

Your Answer

Public sector

Select The Blank Question The ________ was set up to help corrections in BOP maladjustments. Correct Answer IMF Your Answer IBRD Select The Blank Question Mutual Funds provide the benefits of ________. Correct Answer Portfolio management Your Answer High returns Multiple Choice Single Answer Question Financial institutions deal in :Correct Answer Financial assets Your Answer Financial assets Multiple Choice Multiple Answer Question Duties of an International Banking unit cover :Correct Answer Handling foreign currency , Handling Doc. credits , Granting lines of Credit Your Answer Handling foreign currency , Handling Doc. credits , Granting lines of Credit

LIST OF ATTEMPTED QUESTIONS AND ANSWERS Multiple Choice Single Answer Question Balance of Payment is :Correct Answer Analysis of monetary aspects of Foreign trade Your Answer

Analysis of monetary aspects of Foreign trade

Select The Blank Question Capital Account consists of all transactions of ________ nature. Correct Answer Financial Your Answer

Temporary

True/False Question

Free trade policy is not suited to developing & underdeveloped countries because the industries of the developed countries need protection from well established and well organized industries of the developed countries. Correct Answer True Your Answer

True

Select The Blank Question The important distinction between domestic trade and foreign trade relates to difference in ________. Correct Answer Currency Your Answer

Currency

True/False Question

In case of export of goods on consignment basis, the freight and insurance is arranged by the agent of exporter in his country. Correct Answer False Your Answer

True

Multiple Choice Single Answer Question Japan is our major market for :Correct Answer Marine products exports Your Answer

Marine products exports

Select The Blank Question Foreign Trade (Development and Regulation) Act 1992 came into force on ________.

Correct Answer 6/19/2005 Your Answer

6/19/2005

Multiple Choice Multiple Answer Question Exports requiring prior approval of Reserve Bank / Govt authorities are :Correct Answer Counter Trade Agreements , Project Exports , Exports under trade agreements between Govt. of India and the Govt of Foreign state Your Answer Counter Trade Agreements , Project Exports , Exports under trade agreements between Govt. of India and the Govt of Foreign state Select The Blank Question As stated in FERA 1973, the objectives of exchange control in India is ________ of foreign exchange resources of the country and proper utilizations Correct Answer Conservation Your Answer

Conservation

Multiple Choice Single Answer Question The secretariat of SAARC is at Correct Answer Kathmandu, Nepal Your Answer

Kathmandu, Nepal

Multiple Choice Multiple Answer Question Double weightage on FOB on the export of :Correct Answer Products from small/ cottage units , Fruits and vegetables , Horticultural products Your Answer Products from small/ cottage units , Fruits and vegetables , Horticultural products True/False Question

Capital Account covers all transactions relating to India's foreign financial assets and liabilities. Correct Answer True Your Answer

True

Multiple Choice Single Answer Question Imports are recorded in the bo oks of accounts when:Correct Answer Payment is received Your Answer

Payment is received

Select The Blank Question Free trade benefits the ________ because they are able to buy a variety of commodities from abroad and at economical prices. Correct Answer Consumers Your Answer

Consumers

Multiple Choice Multiple Answer Question Policies relating to commerce, trade, taxation :Correct Answer Are same within the country , Different outside the country , Incorporate artificial barriers in form of quotas import duties Your Answer Are same within the country , Different outside the country , Incorporate artificial barriers in form of quotas import duties Match The Following Question

Correct Answer

Your Answer

Value added exports

Finer processing and Incorrect branding packing for earning higher export returns

Faceless presence

Indian raw material with foreign brand name

Unreliability

Lack of prompt After sales Lack of prompt After sales service service

Problem recognition

Lack of innovativeness

Finer processing and packing for earning higher export returns

Lack of innovativeness

Select The Blank Question Long Term Capital involves International transfer of ________ Correct Answer Purchasing power Your Answer

Purchasing power

Match The Following Question

Correct Answer

Your Answer

IEC

Importer Exporter Code

Importer Exporter Code

NDDB

National Dairy Development Board

National Dairy Development Board

MML

Minerals & Metals Ltd

Minerals & Metals Ltd

CCIE

Chief controller of Imports Chief controller of Imports and Exports and Exports

Select The Blank Question The economic argument in favor of protectionism is that the

infant industries have to be protected against stiff________ Correct Answer Competition Your Answer

Import duty

Select The Blank Question India gain due to membership of WTO of other developing countries like china because the ________ ratio of India is low Correct Answer Trade-GDP Your Answer

Trade-GDP

Multiple Choice Single Answer Question Indian products are not sold in Foreign Market because of :Correct Answer Traditional approach Your Answer

Traditional approach

Multiple Choice Single Answer Question Merchandise trade means :Correct Answer Transactions relating to movable goods Your Answer

Transactions relating to movable goods

True/False Question

Foreign Trade helps to understand the life style, culture, philosophies, social & political institutions of the different countries Correct Answer True Your Answer

True

Multiple Choice Multiple Answer Question Free Trade refers to a situation :Correct Answer Where there is unrestricted exchange of goods and services between countries. , Where the Govt. follows a policy of noninterference , Where there is absence of tariffs quotas, exchange restriction taxes and subsidies on production Your Answer Where there is unrestricted exchange of goods and services between countries. , Where the Govt. follows a policy of noninterference , Where there is absence of tariffs quotas, exchange restriction taxes and subsidies on production Multiple Choice Multiple Answer Question The country has recorded a rapid growth in the manufactures of chemicals during the last few years. Prominent among these are

Correct Answer Caustic soda , Soda ash , Carbon black Your Answer

Caustic soda , Soda ash , Carbon black

Multiple Choice Single Answer Question BIS stands for Correct Answer Bank for International Settlements Your Answer

Bank for International Settlements

Multiple Choice Multiple Answer Question Units manufacturing capital goods and consumer durables are scattered all over the country but they are concentrated mainly in Correct Answer Maharashtra , West Bengal , Karnataka Your Answer

Maharashtra , West Bengal , Uttar Pradesh

Multiple Choice Multiple Answer Question The gems and Jewellery export sector is highly import oriented, import of item constituting raw materials for this industry are namely :Correct Answer Rough diamonds , Rough precious and semi precious stones , Pearls Your Answer Rough diamonds , Rough precious and semi precious stones , Pearls True/False Question

Free Trade is the best policy for economic development.

Correct Answer True Your Answer

True

Multiple Choice Multiple Answer Question The woolen industry in the country is small in size. It is basically located in the states of Correct Answer Punjab , Haryana , Rajasthan Your Answer

Punjab , Haryana , Rajasthan

Select The Blank Question Manganese Ores with more than ________ Manganese is canalised. Correct Answer 0.46 Your Answer

0.46

Multiple Choice Single Answer

Question

Cost of production is higher in India because of :-

Correct Answer Time and Cost over runs Your Answer

Time and Cost over runs

Multiple Choice Multiple Answer Question The three corresponding agreements to WTO are Correct Answer GATT , GATS , General Agreement on Trade Related aspects of Intellectual Property Rights Your Answer GATT , GATS True/False Question

The international monetary fund is an organization of countries that seeks to promote international monetary cooperation Correct Answer True Your Answer

True

True/False Question

IRPS is replaced by Engineering Products (Replenishment of Iron and Steel Intermediates) Scheme. Correct Answer True Your Answer

True

Multiple Choice Multiple Answer Question In 1997-98 there was a decline in world trade due to :Correct Answer South east Asian crisis , Continued recession in Japan , Several economic crisis in Russia Your Answer South east Asian crisis , Continued recession in Japan , Several economic crisis in Russia Multiple Choice Multiple Answer Question In BOP surplus nations International Payment equilibrium creates:Correct Answer Increase in price level , Increase in Income Level , Fall in Interest rates Your Answer Increase in price level , Increase in Income Level , Fall in Interest rates Multiple Choice Single Answer Question Duty drawback/exemption avoids :Correct Answer Commodity taxes like excise duty Your Answer

Commodity taxes like excise duty

Multiple Choice Single Answer

Question

Private Resident & non profit bodies comprises of people engaged in :Correct Answer Social, healthcare activities Your Answer

Social, healthcare activities

Select The Blank Question Nearly________ % of the member of WTO are developing countries Correct Answer 80 Your Answer

80

Multiple Choice Single Answer Question India commands an important place in would trade in Correct Answer Tea Your Answer

Tea

True/False Question

Handicrafts are exported to more than 100 countries out of which 11 countries buy and absorb about 80% of India's export of Handicraft Correct Answer True Your Answer

True

Multiple Choice Multiple Answer Question FDI may involve :Correct Answer Transfer of financial assets , Transfer of trademark , Transfer of technology Your Answer Transfer of financial assets , Transfer of trademark , Transfer of technology Multiple Choice Single Answer Question Non traditional sector consists broadly of:Correct Answer Consumer electronics Your Answer

Consumer electronics

Multiple Choice Multiple Answer Question Special Import License based on net foreign exchange is entitled as follows :Correct Answer Export House 6% on FOB , Export House 7.5% on NFE , Superstar Trading House 12% on FOB Your Answer Export House 6% on FOB , Export House 7.5% on NFE , Superstar Trading House 12% on FOB

Select The Blank Question Readymade Garments are still a product of the ________ and decentralized sectors Correct Answer Small Your Answer

Small

14.

Cement and Steel prices were de-controlled in :-

a)

1980-82

b)

1981-82

c)

1982-83

d)

1979-80

As per estimates of IMF if oil price rises by USD 5 per barrel it will amount to

25.

reduced GDP in industrial countries by

34.

During

43.

.

plan period the average annual trade deficit reached its peak

Current Account is regarded as most basic of all sub accounts because :-

a)

Its volumes are bigger in size

b)

It contains all transactions giving rise to countries national income

c)

It helps movement of long term capital goods

d)

It is a day to day activity

45.

has commented on adverse impact of exports due to technological backwardness.

49.

FCNR deposit's details are sent to RBI Monthly on :-

a)

STAT 5 From

b)

STAT 4 From

c)

STAT 3 Form

d)

STAT 10 Form

.

The theory of comparative costs was put forth by :-

a)

Modern economists

b)

Traditional economists

c)

Classical economists

d)

Conservative economists

51.

Faceless presence of Indian exporters in International market is denoted by :-

a)

In ability to export in consumer packs

b)

Create market for the brand

c)

Maintaining quality

d)

Dominance of MNCs

LIST OF ATTEMPTED QUESTIONS AND ANSWERS Multiple Choice Multiple Answer Question India Balance of Payments in 1999-2001 remained comfortable because of :Correct Answer Increase is software exports , Recovery of exports , Net inflow of invisibles Your Answer Sale of Gold in open market , Recovery of exports , Increase is software exports True/False Question

Demonetisation means transfer of gold from Non-Monetary to Monetary stock of reserves. Correct Answer False Your Answer

False

Select The Blank Question Canalised items are imported or exported only through ________. Correct Answer Public Sector agencies

Your Answer

Public Sector agencies

Select The Blank Question Government has allowed Overseas Business operation through issue of ________. Correct Answer ADR/DGR Your Answer

Removal of restrictions

True/False Question

CCS scheme was abolished following the economic reforms.

Correct Answer True Your Answer

False

Multiple Choice Single Answer Question What is the general effect of Foreign Trade? Correct Answer It encourages competition Your Answer

It encourages competition

Select The Blank Question The IFC's role is to stimulate the few of private capital into production private, mixed, private/public enterprises. It acts as a ________ in bringing together entrepreneurship, investment capital E- production Correct Answer Catalyst Your Answer

Catalyst

Multiple Choice Multiple Answer Question In the context of fish exports Correct Answer India is the seventh largest production of fish , Second in inland fish , Japan is the major market Your Answer India is the seventh largest production of fish True/False Question

Coffee has been identified for making special thrust in foreign markets Correct Answer False Your Answer

True

Multiple Choice Single Answer Question Imports results in :Correct Answer Outflow of foreign exchange Your Answer

Outflow of foreign exchange

Multiple Choice Multiple Answer Question As a further step in the path of liberalization Reserve Bank of India has announced on 19th August 1994 :Correct Answer relaxation on the current account transactions and further delegation of powers , The authorized dealers have been permitted to allow remittances for various purposes , The country has achieved full convertibility on current account Your Answer relaxation on the current account transactions and further delegation of powers , The authorized dealers have been permitted to allow remittances for various purposes , The country has achieved full convertibility on current account Select The Blank Question License not required if location of project is beyond 25 Kms periphery from urban area having population of ________. Correct Answer 1 million Your Answer

1 million

Multiple Choice Multiple Answer Question Risks and uncertainties in the external environment are :Correct Answer Uneven GDP Growth pattern , Demand in major currencies like Dollars , Misalignment between EURO and US Dollar Your Answer Uneven GDP Growth pattern , Demand in major currencies like Dollars , Misalignment between EURO and US Dollar Match The Following Question

Correct Answer

IMD

India Millennium Deposits India Millennium Deposits

FER

Foreign Exchange Reserves

Foreign Exchange Reserves

GDPmp

Gross Domestic Product

Gross Domestic Product

FII

Foreign Institutional Investor

Foreign Institutional Investor

True/False Question

Your Answer

Capital Account covers all transactions relating to India's foreign financial assets and liabilities.

Correct Answer True Your Answer

True

Multiple Choice Multiple Answer Question During 1990-91, push was given to the export effort resulting in Correct Answer A surplus of Rs 32,558 crores , An increase of 17.7% in exports , Trade deficit of 10,35 crores Your Answer An increase of 17.7% in exports Multiple Choice Single Answer Question The following country is the founder member of IBRD Correct Answer India Your Answer

USA

Multiple Choice Single Answer Question Export of goods causes :Correct Answer Inflow of foreign exchange Your Answer

Inflow of foreign exchange

Multiple Choice Single Answer Question Foreign institutions like the IMF, IBRD, World Bank, GATT have been established after :Correct Answer The second world war Your Answer

The second world war

Multiple Choice Multiple Answer Question In BOP Official Capital Account consists of following items :Correct Answer Loans , Amortization , Miscellaneous Your Answer

Loans , Amortization , Defence Equipment

Multiple Choice Multiple Answer Question Important Technological factors for low exports are :Correct Answer High cost of production , Low productivity , Poor infrastructure Your Answer

High cost of production , Low productivity , Poor infrastructure

Multiple Choice Single Answer

Question

Until 1970's India had adopted a :-

Correct Answer Protectionist policy Your Answer

Protectionist policy

Multiple Choice Single Answer Question In domestic trade :Correct Answer Volume of trade is large Your Answer

Volume of trade is large

Multiple Choice Single Answer Question The term 'Trade' means :Correct Answer Exchange of goods among people Your Answer

Exchange of goods among people

Multiple Choice Multiple Answer Question As per Double Entry Accounting System timings for recording of various transactions is as follows :Correct Answer Current account- when legal ownership changes , Interest and Dividends- when they are due for payment , Unrequited transfers -when change of ownership takes place Your Answer Current account- when legal ownership changes True/False Question

Adverse International economic situation immediately affects Indian markets and exports. Correct Answer True Your Answer

True

Multiple Choice Single Answer Question In Jan 1997 Foreign equity participation in automatic route raised to :Correct Answer 0.74 Your Answer

0.71

Select The Blank Question Nearly________ % of the member of WTO are developing countries Correct Answer 80

Your Answer

80

Multiple Choice Single Answer Question Exports of Super Phosphates is :Correct Answer Permitted Under License Your Answer

Permitted Under License

True/False Question

IRPS compensates the exporters price difference between domestic market and International market on purchase of raw materials. Correct Answer True Your Answer

False

Multiple Choice Single Answer Question Non Resident External Rupee Account and Foreign Currency Non resident Account fall under :Correct Answer Private Capital Your Answer

Private Capital

Multiple Choice Single Answer Question India is the :Correct Answer Seventh largest producer of fish in the world Your Answer

Seventh largest producer of fish in the world

True/False Question

Export Processing Zones cannot be set up by Private sector.

Correct Answer False Your Answer

True

Multiple Choice Single Answer Question Openness of an economy with respect to foreign trade refers to its Correct Answer Permissiveness towards exports & imports Your Answer True/False

Permissiveness towards exports & imports

Question

Asian Development Bank is established to assist development of the developing countries under its region Correct Answer True Your Answer

True

True/False Question

Short terms Capital involves movement of ownership with less than one year maturity. Correct Answer True Your Answer

True

Multiple Choice Single Answer Question The annual average value of imports during the first plan period was Correct Answer Rs 730 cores Your Answer

Rs 730 cores

Match The Following Question

Correct Answer

Your Answer

FIPB

Foreign Investment Promotion Board

Foreign Investment Promotion Board

PAB

Project Approval Board

Project Approval Board

SAP

Structural Adjustment Programme

Structural Adjustment Programme

PMP

Phased Manufacturing Programme

Phased Manufacturing Programme

Select The Blank Question The equilibrium in BOP exists only when Debit is equal to Credit without compensatory ________. Correct Answer Capital Transfers Your Answer

Devaluation of currency

Select The Blank Question The Invisible transactions except Insurance and Transportation are recorded on ________ basis. Correct Answer Gross Your Answer

Approximation

Multiple Choice Single Answer Question Survey of Unclassified Receipt means :Correct Answer Classification of receipts reported in R returns for the purpose of BOP Your Answer Investigation of non reported transactions Multiple Choice Multiple Answer Question Result of liberalization is :Correct Answer More export led growth , Encouragement towards foreign investments Your Answer More export led growth , Encouragement towards foreign investments , That on import front too there has been an accelerated libaralization in the trade policy Select The Blank Question In India imports are recorded on ________ basis. Correct Answer CIF Your Answer

CIF

Multiple Choice Single Answer Question Until the following country became the member of the world bank in 1980, India was the largest beneficiary Correct Answer China Your Answer

China

True/False Question

Foreign Trade helps to understand the life style, culture, philosophies, social & political institutions of the different countries Correct Answer True Your Answer

True

Multiple Choice Multiple Answer Question The gems and Jewellery export sector is highly import oriented, import of item constituting raw materials for this industry are namely :Correct Answer Rough diamonds , Rough precious and semi precious stones , Pearls Your Answer Rough diamonds , Rough precious and semi precious stones Multiple Choice Single Answer

Question

Foreign Trade is

Correct Answer Trade between different countries Your Answer

Trade between different countries

14.

Cement and Steel prices were de-controlled in :-

a)

1980-82

b)

1981-82

c)

1982-83

d)

1979-80

As per estimates of IMF if oil price rises by USD 5 per barrel it will amount to

25.

reduced GDP in industrial countries by

34.

During

43.

.

plan period the average annual trade deficit reached its peak

Current Account is regarded as most basic of all sub accounts because :-

a)

Its volumes are bigger in size

b)

It contains all transactions giving rise to countries national income

c)

It helps movement of long term capital goods

d)

It is a day to day activity

45.

has commented on adverse impact of exports due to technological backwardness.

49.

FCNR deposit's details are sent to RBI Monthly on :-

a)

STAT 5 From

b)

STAT 4 From

c)

STAT 3 Form

d)

STAT 10 Form

.

The theory of comparative costs was put forth by :-

a)

Modern economists

b)

Traditional economists

c)

Classical economists

d)

Conservative economists

51.

Faceless presence of Indian exporters in International market is denoted by :-

a)

In ability to export in consumer packs

b)

Create market for the brand

c)

Maintaining quality

d)

Dominance of MNCs

LIST OF ATTEMPTED QUESTIONS AND ANSWERS Multiple Choice Multiple Answer Question India Balance of Payments in 1999-2001 remained comfortable because of :Correct Answer Increase is software exports , Recovery of exports , Net inflow of invisibles Your Answer Sale of Gold in open market , Recovery of exports , Increase is software exports True/False Question

Demonetisation means transfer of gold from Non-Monetary to Monetary stock of reserves. Correct Answer False Your Answer

False

Select The Blank Question Canalised items are imported or exported only through ________. Correct Answer Public Sector agencies

Your Answer

Public Sector agencies

Select The Blank Question Government has allowed Overseas Business operation through issue of ________. Correct Answer ADR/DGR Your Answer

Removal of restrictions

True/False Question

CCS scheme was abolished following the economic reforms.

Correct Answer True Your Answer

False

Multiple Choice Single Answer Question What is the general effect of Foreign Trade? Correct Answer It encourages competition Your Answer

It encourages competition

Select The Blank Question The IFC's role is to stimulate the few of private capital into production private, mixed, private/public enterprises. It acts as a ________ in bringing together entrepreneurship, investment capital E- production Correct Answer Catalyst Your Answer

Catalyst

Multiple Choice Multiple Answer Question In the context of fish exports Correct Answer India is the seventh largest production of fish , Second in inland fish , Japan is the major market Your Answer India is the seventh largest production of fish True/False Question

Coffee has been identified for making special thrust in foreign markets Correct Answer False Your Answer

True

Multiple Choice Single Answer Question Imports results in :Correct Answer Outflow of foreign exchange Your Answer

Outflow of foreign exchange

Multiple Choice Multiple Answer Question As a further step in the path of liberalization Reserve Bank of India has announced on 19th August 1994 :Correct Answer relaxation on the current account transactions and further delegation of powers , The authorized dealers have been permitted to allow remittances for various purposes , The country has achieved full convertibility on current account Your Answer relaxation on the current account transactions and further delegation of powers , The authorized dealers have been permitted to allow remittances for various purposes , The country has achieved full convertibility on current account Select The Blank Question License not required if location of project is beyond 25 Kms periphery from urban area having population of ________. Correct Answer 1 million Your Answer

1 million

Multiple Choice Multiple Answer Question Risks and uncertainties in the external environment are :Correct Answer Uneven GDP Growth pattern , Demand in major currencies like Dollars , Misalignment between EURO and US Dollar Your Answer Uneven GDP Growth pattern , Demand in major currencies like Dollars , Misalignment between EURO and US Dollar Match The Following Question

Correct Answer

IMD

India Millennium Deposits India Millennium Deposits

FER

Foreign Exchange Reserves

Foreign Exchange Reserves

GDPmp

Gross Domestic Product

Gross Domestic Product

FII

Foreign Institutional Investor

Foreign Institutional Investor

True/False Question

Your Answer

Capital Account covers all transactions relating to India's foreign financial assets and liabilities.

Correct Answer True Your Answer

True

Multiple Choice Multiple Answer Question During 1990-91, push was given to the export effort resulting in Correct Answer A surplus of Rs 32,558 crores , An increase of 17.7% in exports , Trade deficit of 10,35 crores Your Answer An increase of 17.7% in exports Multiple Choice Single Answer Question The following country is the founder member of IBRD Correct Answer India Your Answer

USA

Multiple Choice Single Answer Question Export of goods causes :Correct Answer Inflow of foreign exchange Your Answer

Inflow of foreign exchange

Multiple Choice Single Answer Question Foreign institutions like the IMF, IBRD, World Bank, GATT have been established after :Correct Answer The second world war Your Answer

The second world war

Multiple Choice Multiple Answer Question In BOP Official Capital Account consists of following items :Correct Answer Loans , Amortization , Miscellaneous Your Answer

Loans , Amortization , Defence Equipment

Multiple Choice Multiple Answer Question Important Technological factors for low exports are :Correct Answer High cost of production , Low productivity , Poor infrastructure Your Answer

High cost of production , Low productivity , Poor infrastructure

Multiple Choice Single Answer

Question

Until 1970's India had adopted a :-

Correct Answer Protectionist policy Your Answer

Protectionist policy

Multiple Choice Single Answer Question In domestic trade :Correct Answer Volume of trade is large Your Answer

Volume of trade is large

Multiple Choice Single Answer Question The term 'Trade' means :Correct Answer Exchange of goods among people Your Answer

Exchange of goods among people

Multiple Choice Multiple Answer Question As per Double Entry Accounting System timings for recording of various transactions is as follows :Correct Answer Current account- when legal ownership changes , Interest and Dividends- when they are due for payment , Unrequited transfers -when change of ownership takes place Your Answer Current account- when legal ownership changes True/False Question

Adverse International economic situation immediately affects Indian markets and exports. Correct Answer True Your Answer

True

Multiple Choice Single Answer Question In Jan 1997 Foreign equity participation in automatic route raised to :Correct Answer 0.74 Your Answer

0.71

Select The Blank Question Nearly________ % of the member of WTO are developing countries Correct Answer 80

Your Answer

80

Multiple Choice Single Answer Question Exports of Super Phosphates is :Correct Answer Permitted Under License Your Answer

Permitted Under License

True/False Question

IRPS compensates the exporters price difference between domestic market and International market on purchase of raw materials. Correct Answer True Your Answer

False

Multiple Choice Single Answer Question Non Resident External Rupee Account and Foreign Currency Non resident Account fall under :Correct Answer Private Capital Your Answer

Private Capital

Multiple Choice Single Answer Question India is the :Correct Answer Seventh largest producer of fish in the world Your Answer

Seventh largest producer of fish in the world

True/False Question

Export Processing Zones cannot be set up by Private sector.

Correct Answer False Your Answer

True

Multiple Choice Single Answer Question Openness of an economy with respect to foreign trade refers to its Correct Answer Permissiveness towards exports & imports Your Answer True/False

Permissiveness towards exports & imports

Question

Asian Development Bank is established to assist development of the developing countries under its region Correct Answer True Your Answer

True

True/False Question

Short terms Capital involves movement of ownership with less than one year maturity. Correct Answer True Your Answer

True

Multiple Choice Single Answer Question The annual average value of imports during the first plan period was Correct Answer Rs 730 cores Your Answer

Rs 730 cores

Match The Following Question

Correct Answer

Your Answer

FIPB

Foreign Investment Promotion Board

Foreign Investment Promotion Board

PAB

Project Approval Board

Project Approval Board

SAP

Structural Adjustment Programme

Structural Adjustment Programme

PMP

Phased Manufacturing Programme

Phased Manufacturing Programme

Select The Blank Question The equilibrium in BOP exists only when Debit is equal to Credit without compensatory ________. Correct Answer Capital Transfers Your Answer

Devaluation of currency

Select The Blank Question The Invisible transactions except Insurance and Transportation are recorded on ________ basis. Correct Answer Gross Your Answer

Approximation

Multiple Choice Single Answer Question Survey of Unclassified Receipt means :Correct Answer Classification of receipts reported in R returns for the purpose of BOP Your Answer Investigation of non reported transactions Multiple Choice Multiple Answer Question Result of liberalization is :Correct Answer More export led growth , Encouragement towards foreign investments Your Answer More export led growth , Encouragement towards foreign investments , That on import front too there has been an accelerated libaralization in the trade policy Select The Blank Question In India imports are recorded on ________ basis. Correct Answer CIF Your Answer

CIF

Multiple Choice Single Answer Question Until the following country became the member of the world bank in 1980, India was the largest beneficiary Correct Answer China Your Answer

China

True/False Question

Foreign Trade helps to understand the life style, culture, philosophies, social & political institutions of the different countries Correct Answer True Your Answer

True

Multiple Choice Multiple Answer Question The gems and Jewellery export sector is highly import oriented, import of item constituting raw materials for this industry are namely :Correct Answer Rough diamonds , Rough precious and semi precious stones , Pearls Your Answer Rough diamonds , Rough precious and semi precious stones Multiple Choice Single Answer

Question

Foreign Trade is

Correct Answer Trade between different countries Your Answer

Trade between different countries

LIST OF ATTEMPTED QUESTIONS AND ANSWERS Multiple Choice Multiple Answer Question The major market for handicrafts and jewellery include Correct Answer UK , USA , Germany Your Answer

UK , USA , Germany

Multiple Choice Multiple Answer Question Important Technological factors for low exports are :Correct Answer High cost of production , Low productivity , Poor infrastructure Your Answer

High cost of production , Low productivity , Poor infrastructure

True/False Question

The trade balance was positive during the year 2000-01 amounting to approx US$ 1 Bill only. Correct Answer False Your Answer

False

Multiple Choice Multiple Answer Question Risks and uncertainties in the external environment are :Correct Answer Uneven GDP Growth pattern , Demand in major currencies like Dollars , Misalignment between EURO and US Dollar Your Answer Uneven GDP Growth pattern , Demand in major currencies like Dollars , Lack of support to developing countries Select The Blank Question Free trade may encourage the import of ________ products in the country and hunt the interest of the nations Correct Answer Harmful Your Answer

Harmful

Multiple Choice Single Answer Question Pre Shipment inspection is part of :Correct Answer Export promotion by maintaining quality Your Answer

Export promotion by maintaining quality

Multiple Choice Single Answer Question India is the :Correct Answer Seventh largest producer of fish in the world

Your Answer

Seventh largest producer of fish in the world

Multiple Choice Single Answer Question Remittance against imports should be completed within Correct Answer 6 months from the data of import Your Answer

6 months from the data of import

True/False Question

WTO is simply renaming of GATT an d the there is no major difference Correct Answer False Your Answer

False

Multiple Choice Single Answer Question One of the major issues related to the controversy of GATT was the issue of Correct Answer Agricultural subsidies Your Answer

Agricultural subsidies

Multiple Choice Single Answer Question Where the import is in a non-physical form(e.g. data over internet) :Correct Answer A certificate from the chartered accountant to the effect that the data/design/software has been received by the importer, and is acceptable, is required Your Answer A certificate from the chartered accountant to the effect that the data/design/software has been received by the importer, and is acceptable, is required Multiple Choice Multiple Answer Question If after taking all the efforts, the export bill is not realized to write off the bill with supporting documentary evidence the bank will consider the case subject to following conditions :Correct Answer The bill is outstanding for over a year , The total amount of write-off allowed by the bank in a year does not exceed 10% of the average annual export realization of the exporter, during the preceding three calendar years , Exporter has submitted satisfactory evidence to show that he has taken all the possible efforts Your Answer The bill is outstanding for over a year , The total amount of write-off allowed by the bank in a year does not exceed 10% of the average annual export realization of the exporter, during the preceding three calendar years , Exporter has submitted satisfactory evidence to show that he has taken all the possible efforts Multiple Choice Single Answer

Question

The theory of comparative costs was put forth by :-

Correct Answer Classical economists Your Answer

Classical economists

True/False Question

Consignment exports means goods dispatched to the agent of the exporter on sale or return basis Correct Answer True Your Answer

True

Multiple Choice Single Answer Question Export of goods causes :Correct Answer Inflow of foreign exchange Your Answer

Inflow of foreign exchange

Multiple Choice Multiple Answer Question The general council will serve following main functions: Correct Answer To act as dispute settlement body , To establishment goods council , To supervise on a regular basis the operations of revised agreements Your Answer To act as dispute settlement body , To establishment goods council , To supervise on a regular basis the operations of revised agreements Multiple Choice Multiple Answer Question The WTO deals with the special needs of the developing countries in three ways Correct Answer It contain special provision on developing countries , It has a committee on trade and development , WTO secretariat provides technical assistance Your Answer It contain special provision on developing countries , It has a committee on trade and development , WTO secretariat provides technical assistance Multiple Choice Single Answer Question In domestic trade :Correct Answer Volume of trade is large Your Answer

Volume of trade is large

Multiple Choice Single Answer Question India's contribution in the World Trade in 1950 was :Correct Answer 0.018

Your Answer

0.018

Multiple Choice Single Answer Question Government approval for Joint Venture with Small Scale sector is given by :Correct Answer District Industrial Centre Your Answer

Director General Foreign Trade

True/False Question

Membership in the world bank is optional & not prerequisite for membership in International Finance Corporation (IFC) Correct Answer False Your Answer

False

Multiple Choice Single Answer Question Imports results in :Correct Answer Outflow of foreign exchange Your Answer

Outflow of foreign exchange

Multiple Choice Single Answer Question What is the general effect of Foreign Trade? Correct Answer It encourages competition Your Answer

It increases lower levels of output

Multiple Choice Single Answer Question Until 1970's India had adopted a :Correct Answer Protectionist policy Your Answer

Free trade policy

Multiple Choice Multiple Answer Question Objectives of Planning Commission were:Correct Answer Augment infrastructure , Formulation of policies for growth , Assessment of available resources Your Answer Augment infrastructure , Formulation of policies for growth , Assessment of available resources Multiple Choice Multiple Answer Question Bertil Ohlin :Correct Answer Contends that there is no difference between domestic and foreign trade , Is not a classical economist , Is a modern economist

Your Answer

Is a modern economist , Contends that there is no difference between domestic and foreign trade , Is not a classical economist

Multiple Choice Multiple Answer Question Labour is not mobile between countries because of :Correct Answer Difference in languages , Difference in customs , Occupational skills Your Answer Difference in languages , Difference in customs , Occupational skills Select The Blank Question The woolen industry in the country is ________ in size and widely scattered Correct Answer Small Your Answer

Small

True/False Question

Export growth was slow due to lack of value addition.

Correct Answer True Your Answer

True

Multiple Choice Multiple Answer Question Result of liberalization is :Correct Answer More export led growth , Encouragement towards foreign investments Your Answer More export led growth , Encouragement towards foreign investments , That on import front too there has been an accelerated libaralization in the trade policy Select The Blank Question The current account deficit in 1999 was ________ of GDP. Correct Answer 0.009 Your Answer

0.9.5%

Multiple Choice Multiple Answer Question Classical economists :Correct Answer Say that there is a fundamental difference between domestic and foreign trade. , Have put forward the theory of comparative costs , Differ from modern economists Your Answer Say that there is a fundamental difference between domestic and foreign trade. , Have put forward the theory of comparative costs , Differ from modern economists Multiple Choice Single Answer

Question

The data regarding sale and purchase of foreign currencies by FCNR account holders is maintained by :Correct Answer DEIO Your Answer

Authorised Dealers

Select The Blank Question With a view to boost agricultural exports and provide remunerative returns to the farming community, the exportimport policy 2002-2007 proposed the setting up of________ Correct Answer Agricultural Export Zones Your Answer

Agricultural Export Zones

Multiple Choice Single Answer Question Asian Development bank was set up in December 1966 under the auspices of the Correct Answer United Nations Economics Commission for Asia and Far East ( ECAFE) Your Answer United Nations Economics Commission for Asia and Far East ( ECAFE) Multiple Choice Multiple Answer Question Transportation Credit includes ………. Correct Answer Freight on exports , remittances received by Shipping/Air lines , Remittances received by Foreign Agents of Indian companies Your Answer Freight on exports , remittances received by Shipping/Air lines , Remittances received by Foreign Agents of Indian companies True/False Question

Export Processing Zones cannot be set up by Private sector.

Correct Answer False Your Answer

True

Select The Blank Question Most of the leather exports go to highly industrialized country like________ Correct Answer America Your Answer

America

True/False Question

Trade samples of goods and publicity materials supplied free of payment are exempted from declaration in the form prescribed by RBI Correct Answer True Your Answer

True

Multiple Choice Single Answer Question Foreign Trade is Correct Answer Trade between different countries Your Answer

Trade between different countries

Multiple Choice Single Answer Question The term 'Trade' means :Correct Answer Exchange of goods among people Your Answer

Exchange of goods among people

Select The Blank Question Capital fund flows have been augmented by SBI with floating of ________. Correct Answer India Millennium Deposits Your Answer

India Millennium Deposits

True/False Question

A deficit occurs when Autonomous Transactions give rise to receipts as against payments. Correct Answer False Your Answer

True

Select The Blank Question India's Trade Policy makers have realized that there is no alternative but to give thrust to the ________. Correct Answer Export Your Answer

Export

Match The Following Question

Correct Answer

Your Answer

ECB

External Commercial Borrowings

External Commercial Borrowings

FCNR

Foreign Currency Non Resident Account

Foreign Currency Non Resident Account

SDR

Special Drawings Rights

Special Drawings Rights

GDP

Gross Domestic Product

Gross Domestic Product

Match The Following

Question

Correct Answer

Your Answer

DTA

Domestic Tariff Area

Domestic Tariff Area

MODVAT

Modified Value Added Tax

Modified Value Added Tax

IRPS

International Price Reimbursement Scheme

International Price Reimbursement Scheme

CCS

Cash Compensatory Support Scheme

Cash Compensatory Support Scheme

Select The Blank Question In UP Export Processing Zone is established in ________. Correct Answer Noida Your Answer

Noida

LIST OF ATTEMPTED QUESTIONS AND ANSWERS Select The Blank Question The IFC's role is to stimulate the few of private capital into production private, mixed, private/public enterprises. It acts as a ________ in bringing together entrepreneurship, investment capital E- production Correct Answer Catalyst Your Answer

Catalyst

Multiple Choice Single Answer Question Travel credits are :Correct Answer Foreign exchange received from abroad for tourist/ students Your Answer

Foreign exchange received from abroad for tourist/ students

Select The Blank Question Agency commission is not allowed to be remitted in case of exports under ________ credit route. Correct Answer Rupee Your Answer

Rupee

Multiple Choice Single Answer Question During the second plan period Correct Answer Food grains import had to be resorted to overcome internal shortages Your Answer Food grains import had to be resorted to overcome internal shortages Multiple Choice Single Answer Question One of oldest organized industries is Correct Answer Indian Textiles Your Answer

Indian Textiles

Multiple Choice Single Answer Question Prohibited Imports and Exports are punishable under :Correct Answer Section 11 of the Customs Act Your Answer

Section 11 of the Customs Act

Multiple Choice Single Answer Question The woolen industry in the country is basically located in the state of :Correct Answer Punjab Your Answer

Punjab

Select The Blank Question The gems & jewellery export sector is highly________ oriented Correct Answer Import Your Answer

Import

Multiple Choice Multiple Answer Question The agreements of WTO :Correct Answer Spell out principles of liberalization , Set procedures for settling disputes , Prescribe special treatment for developing countries Your Answer Spell out principles of liberalization , Set procedures for settling disputes , Prescribe special treatment for developing countries True/False Question

Capital Account of BOP has undergone major structural change in favour of Non Debt creating Foreign Investment inflows. Correct Answer True Your Answer

True

Multiple Choice Multiple Answer Question Faceless presence of Indian exporters in International market is denoted by :Correct Answer Maintaining quality , In ability to export in consumer packs , Create market for the brand Your Answer In ability to export in consumer packs , Create market for the brand , Maintaining quality Multiple Choice Single Answer Question Export prices are high because of :Correct Answer Economies of scale not available Your Answer

Economies of scale not available

Select The Blank Question Declaration of export in PP forms are required for exports in physical form through________

Correct Answer Post Your Answer

Post

Multiple Choice Multiple Answer Question During 1990-91, push was given to the export effort resulting in Correct Answer A surplus of Rs 32,558 crores , An increase of 17.7% in exports , Trade deficit of 10,35 crores Your Answer A surplus of Rs 32,558 crores , An increase of 17.7% in exports , Trade deficit of 10,35 crores Multiple Choice Multiple Answer Question Long Terms capital consists of mainly transactions in :Correct Answer Equities , Loans , Real Estate Your Answer

Equities , Loans , Real Estate

True/False Question

Coffee has been identified for making special thrust in foreign markets Correct Answer False Your Answer

False

True/False Question

India has a natural competitive advantage in agricultural exports because of lower need of inputs, reasonable labour cost and diverse agro-climatic conditions. Correct Answer True Your Answer

True

Select The Blank Question International Gold Standard is one form of ________. Correct Answer Fixed exchange System for adjustment of BOP Your Answer

Fixed exchange System for adjustment of BOP

True/False Question

Under the Gold Standard burden of adjustment is on Internal Economy. Correct Answer True Your Answer

True

Multiple Choice Single Answer Question Openness of an economy with respect to foreign trade refers to its Correct Answer Permissiveness towards exports & imports Your Answer

Permissiveness towards exports & imports

Multiple Choice Single Answer Question On the eve of planning 1948-49 to 1950-51 there was a rise in imports largely due to Correct Answer Pent up demand of the war and the post war period Your Answer

Pent up demand of the war and the post war period

Select The Blank Question Santa Cruz Export Processing Zone established in ________. Correct Answer 1974 Your Answer

1974

Select The Blank Question The Invisible transactions except Insurance and Transportation are recorded on ________ basis. Correct Answer Gross Your Answer

Gross

Match The Following Question

Correct Answer

Your Answer

Developed countries such as Japan, USA, Canada, West Germany, Australia etc.

A range of financial services companies and provide technical assistance

Contributes to funds of ADB

Asian development bank Make its investments in Development of the (ADB) has been partnership with investors developing countries under establishment to from the capital exporting its region country features of IFC's assistance

Facilitate free trade within Make its investments in countries partnership with investors from the capital exporting country

In India, IFC will invest in Development of the developing countries under its region

A range of financial services companies and provide technical assistance

Select The Blank Question ________ has commented on adverse impact of exports due to technological backwardness. Correct Answer Tandon committee Your Answer

Tandon committee

True/False Question

International Monetary Fund was established to create a monetary system that would provide relief and reconstruction to the countries affected by World War. Correct Answer False Your Answer

False

Multiple Choice Single Answer Question India is world leader in :Correct Answer Tea production Your Answer

Tea production

Select The Blank Question During ________ plan period the average annual trade deficit reached its peak Correct Answer Seventh Your Answer

Ninth

Multiple Choice Single Answer Question The invisible Accounts consists of services provided such as :Correct Answer Travel, Gifts etc Your Answer

Travel, Gifts etc

Multiple Choice Single Answer Question Whenever under the trade policy, the import of any item is subjected to an import the banks should not open any letter of credit or effect any remittance unless the original Correct Answer Exchange control copy of the license is submitted

Your Answer

Exchange control copy of the license is submitted

True/False Question

Membership of IMF is the principle condition for membership of the world bank Correct Answer True Your Answer

True

Multiple Choice Single Answer Question Letters of credit can be opened and/or remittances can be effected by a person other than the importer himself :Correct Answer On the strength of the letter of authority issued by the importer in the name of such a person Your Answer On the strength of the letter of authority issued by the importer in the name of such a person Multiple Choice Single Answer Question The principles of non-discriminations of WTO requires that Correct Answer All trading partners shall be granted the most fevoured nation status (MFN) Your Answer All trading partners shall be granted the most fevoured nation status (MFN) Multiple Choice Single Answer Question CIS stands for Correct Answer Commonwealth of independent states Your Answer

Commonwealth of independent states

Multiple Choice Multiple Answer Question Measures which could be taken for reducing disequilibrium in BOP as per Pegged System are :Correct Answer Raising interest rates , Discourage imports , Encourage Capital inflows. Your Answer Raising interest rates , Discourage imports , Encourage Capital inflows. Multiple Choice Multiple Answer Question The value of rupee was devalued by 36.5% in June 1966 because of Correct Answer Persistent adverse balance of trade since 1951 , Acute shortage of foreign exchange , Extensive borrowing by India from foreign countries Your Answer Persistent adverse balance of trade since 1951 , Acute shortage of foreign exchange , Extensive borrowing by India from foreign countries

True/False Question

By being part of WTO, India does not enjoy many benefits.

Correct Answer False Your Answer

False

Multiple Choice Single Answer Question FCNR Operations details are sent to RBI on :Correct Answer STAT 6 From Your Answer

STAT 6 From

Select The Blank Question Massive programmes of industrialization were initiated during ________ plan period. Correct Answer Second Your Answer

Second

Multiple Choice Multiple Answer Question The status holders will be eligible for the following new facilities Correct Answer Fixation of input-output norms on priority , 100% retention of foreign exchange in EEFC account , Enhancement in normal repatriation period from 180 to 360 days Your Answer Fixation of input-output norms on priority , 100% retention of foreign exchange in EEFC account , Enhancement in normal repatriation period from 180 to 360 days Multiple Choice Multiple Answer Question Mechanisms available for adjustments of BOP are :Correct Answer Changes in Income , Changes in Prices , Changes in Interest Rates Your Answer Changes in Income , Changes in Prices , Changes in Interest Rates Multiple Choice Multiple Answer Question Exporters are entitled to following subsidies :Correct Answer Air Freight subsidy , Export Credit Interest rates , Excise duties Your Answer True/False

Air Freight subsidy , Export Credit Interest rates , Excise duties

Question

For the exports to Nepal and Bhutan payment has to be in their respective currencies Correct Answer False Your Answer

True

Select The Blank Question GR and SDF forms in duplicate are to be submitted to the ________ Correct Answer Commissioner of Customs Your Answer

Commissioner of Customs

Multiple Choice Single Answer Question For exchange control purposes the Rupee account of Bhutanese and Nepalese nationals and firms, maintained with any Indian Banks are treated as Correct Answer Resident account Your Answer

Resident account

Select The Blank Question India was forced to devalue the rupee by 36.5 percent in June________ Correct Answer 1966 Your Answer

1966

Match The Following Question

Correct Answer

Your Answer

one major controversy of Agricultural subsidies GATT is the issue related to

Agricultural subsidies

The WTO agreement has many provisions granting special favours to developing & learnt developed countries

Learnt developed countries are exempted from fulfilling their commitments

Developing countries have been given extra time to fulfill their comments

Uruguay Round of agreements have come in India & many politicians have agreed that

Developing countries have been given extra time to fulfill their comments

India should withdraw from GATT

The Uruguay round mostly benefit

Developed countries

Developed countries

LIST OF ATTEMPTED QUESTIONS AND ANSWERS Multiple Choice Single Answer Question Balance of Payment is :Correct Answer Analysis of monetary aspects of Foreign trade Your Answer

Analysis of monetary aspects of Foreign trade

Select The Blank Question Capital Account consists of all transactions of ________ nature. Correct Answer Financial Your Answer

Temporary

True/False Question

Free trade policy is not suited to developing & underdeveloped countries because the industries of the developed countries need protection from well established and well organized industries of the developed countries. Correct Answer True Your Answer

True

Select The Blank Question The important distinction between domestic trade and foreign trade relates to difference in ________. Correct Answer Currency Your Answer

Currency

True/False Question

In case of export of goods on consignment basis, the freight and insurance is arranged by the agent of exporter in his country. Correct Answer False Your Answer

True

Multiple Choice Single Answer Question Japan is our major market for :Correct Answer Marine products exports Your Answer

Marine products exports

Select The Blank Question Foreign Trade (Development and Regulation) Act 1992 came into force on ________.

Correct Answer 6/19/2005 Your Answer

6/19/2005

Multiple Choice Multiple Answer Question Exports requiring prior approval of Reserve Bank / Govt authorities are :Correct Answer Counter Trade Agreements , Project Exports , Exports under trade agreements between Govt. of India and the Govt of Foreign state Your Answer Counter Trade Agreements , Project Exports , Exports under trade agreements between Govt. of India and the Govt of Foreign state Select The Blank Question As stated in FERA 1973, the objectives of exchange control in India is ________ of foreign exchange resources of the country and proper utilizations Correct Answer Conservation Your Answer

Conservation

Multiple Choice Single Answer Question The secretariat of SAARC is at Correct Answer Kathmandu, Nepal Your Answer

Kathmandu, Nepal

Multiple Choice Multiple Answer Question Double weightage on FOB on the export of :Correct Answer Products from small/ cottage units , Fruits and vegetables , Horticultural products Your Answer Products from small/ cottage units , Fruits and vegetables , Horticultural products True/False Question

Capital Account covers all transactions relating to India's foreign financial assets and liabilities. Correct Answer True Your Answer

True

Multiple Choice Single Answer Question Imports are recorded in the bo oks of accounts when:Correct Answer Payment is received Your Answer

Payment is received

Select The Blank Question Free trade benefits the ________ because they are able to buy a variety of commodities from abroad and at economical prices. Correct Answer Consumers Your Answer

Consumers

Multiple Choice Multiple Answer Question Policies relating to commerce, trade, taxation :Correct Answer Are same within the country , Different outside the country , Incorporate artificial barriers in form of quotas import duties Your Answer Are same within the country , Different outside the country , Incorporate artificial barriers in form of quotas import duties Match The Following Question

Correct Answer

Your Answer

Value added exports

Finer processing and Incorrect branding packing for earning higher export returns

Faceless presence

Indian raw material with foreign brand name

Unreliability

Lack of prompt After sales Lack of prompt After sales service service

Problem recognition

Lack of innovativeness

Finer processing and packing for earning higher export returns

Lack of innovativeness

Select The Blank Question Long Term Capital involves International transfer of ________ Correct Answer Purchasing power Your Answer

Purchasing power

Match The Following Question

Correct Answer

Your Answer

IEC

Importer Exporter Code

Importer Exporter Code

NDDB

National Dairy Development Board

National Dairy Development Board

MML

Minerals & Metals Ltd

Minerals & Metals Ltd

CCIE

Chief controller of Imports Chief controller of Imports and Exports and Exports

Select The Blank Question The economic argument in favor of protectionism is that the

infant industries have to be protected against stiff________ Correct Answer Competition Your Answer

Import duty

Select The Blank Question India gain due to membership of WTO of other developing countries like china because the ________ ratio of India is low Correct Answer Trade-GDP Your Answer

Trade-GDP

Multiple Choice Single Answer Question Indian products are not sold in Foreign Market because of :Correct Answer Traditional approach Your Answer

Traditional approach

Multiple Choice Single Answer Question Merchandise trade means :Correct Answer Transactions relating to movable goods Your Answer

Transactions relating to movable goods

True/False Question

Foreign Trade helps to understand the life style, culture, philosophies, social & political institutions of the different countries Correct Answer True Your Answer

True

Multiple Choice Multiple Answer Question Free Trade refers to a situation :Correct Answer Where there is unrestricted exchange of goods and services between countries. , Where the Govt. follows a policy of noninterference , Where there is absence of tariffs quotas, exchange restriction taxes and subsidies on production Your Answer Where there is unrestricted exchange of goods and services between countries. , Where the Govt. follows a policy of noninterference , Where there is absence of tariffs quotas, exchange restriction taxes and subsidies on production Multiple Choice Multiple Answer Question The country has recorded a rapid growth in the manufactures of chemicals during the last few years. Prominent among these are

Correct Answer Caustic soda , Soda ash , Carbon black Your Answer

Caustic soda , Soda ash , Carbon black

Multiple Choice Single Answer Question BIS stands for Correct Answer Bank for International Settlements Your Answer

Bank for International Settlements

Multiple Choice Multiple Answer Question Units manufacturing capital goods and consumer durables are scattered all over the country but they are concentrated mainly in Correct Answer Maharashtra , West Bengal , Karnataka Your Answer

Maharashtra , West Bengal , Uttar Pradesh

Multiple Choice Multiple Answer Question The gems and Jewellery export sector is highly import oriented, import of item constituting raw materials for this industry are namely :Correct Answer Rough diamonds , Rough precious and semi precious stones , Pearls Your Answer Rough diamonds , Rough precious and semi precious stones , Pearls True/False Question

Free Trade is the best policy for economic development.

Correct Answer True Your Answer

True

Multiple Choice Multiple Answer Question The woolen industry in the country is small in size. It is basically located in the states of Correct Answer Punjab , Haryana , Rajasthan Your Answer

Punjab , Haryana , Rajasthan

Select The Blank Question Manganese Ores with more than ________ Manganese is canalised. Correct Answer 0.46 Your Answer

0.46

Multiple Choice Single Answer

Question

Cost of production is higher in India because of :-

Correct Answer Time and Cost over runs Your Answer

Time and Cost over runs

Multiple Choice Multiple Answer Question The three corresponding agreements to WTO are Correct Answer GATT , GATS , General Agreement on Trade Related aspects of Intellectual Property Rights Your Answer GATT , GATS True/False Question

The international monetary fund is an organization of countries that seeks to promote international monetary cooperation Correct Answer True Your Answer

True

True/False Question

IRPS is replaced by Engineering Products (Replenishment of Iron and Steel Intermediates) Scheme. Correct Answer True Your Answer

True

Multiple Choice Multiple Answer Question In 1997-98 there was a decline in world trade due to :Correct Answer South east Asian crisis , Continued recession in Japan , Several economic crisis in Russia Your Answer South east Asian crisis , Continued recession in Japan , Several economic crisis in Russia Multiple Choice Multiple Answer Question In BOP surplus nations International Payment equilibrium creates:Correct Answer Increase in price level , Increase in Income Level , Fall in Interest rates Your Answer Increase in price level , Increase in Income Level , Fall in Interest rates Multiple Choice Single Answer Question Duty drawback/exemption avoids :Correct Answer Commodity taxes like excise duty Your Answer

Commodity taxes like excise duty

Multiple Choice Single Answer

Question

Private Resident & non profit bodies comprises of people engaged in :Correct Answer Social, healthcare activities Your Answer

Social, healthcare activities

Select The Blank Question Nearly________ % of the member of WTO are developing countries Correct Answer 80 Your Answer

80

Multiple Choice Single Answer Question India commands an important place in would trade in Correct Answer Tea Your Answer

Tea

True/False Question

Handicrafts are exported to more than 100 countries out of which 11 countries buy and absorb about 80% of India's export of Handicraft Correct Answer True Your Answer

True

Multiple Choice Multiple Answer Question FDI may involve :Correct Answer Transfer of financial assets , Transfer of trademark , Transfer of technology Your Answer Transfer of financial assets , Transfer of trademark , Transfer of technology Multiple Choice Single Answer Question Non traditional sector consists broadly of:Correct Answer Consumer electronics Your Answer

Consumer electronics

Multiple Choice Multiple Answer Question Special Import License based on net foreign exchange is entitled as follows :Correct Answer Export House 6% on FOB , Export House 7.5% on NFE , Superstar Trading House 12% on FOB Your Answer Export House 6% on FOB , Export House 7.5% on NFE , Superstar Trading House 12% on FOB

Select The Blank Question Readymade Garments are still a product of the ________ and decentralized sectors Correct Answer Small Your Answer

Small

LIST OF ATTEMPTED QUESTIONS AND ANSWERS Multiple Choice Multiple Answer Question The major market for handicrafts and jewellery include Correct Answer UK , USA , Germany Your Answer

UK , USA , Germany

Multiple Choice Multiple Answer Question Important Technological factors for low exports are :Correct Answer High cost of production , Low productivity , Poor infrastructure Your Answer

High cost of production , Low productivity , Poor infrastructure

True/False Question

The trade balance was positive during the year 2000-01 amounting to approx US$ 1 Bill only. Correct Answer False Your Answer

False

Multiple Choice Multiple Answer Question Risks and uncertainties in the external environment are :Correct Answer Uneven GDP Growth pattern , Demand in major currencies like Dollars , Misalignment between EURO and US Dollar Your Answer Uneven GDP Growth pattern , Demand in major currencies like Dollars , Lack of support to developing countries Select The Blank Question Free trade may encourage the import of ________ products in the country and hunt the interest of the nations Correct Answer Harmful Your Answer

Harmful

Multiple Choice Single Answer Question Pre Shipment inspection is part of :Correct Answer Export promotion by maintaining quality Your Answer

Export promotion by maintaining quality

Multiple Choice Single Answer Question India is the :Correct Answer Seventh largest producer of fish in the world

Your Answer

Seventh largest producer of fish in the world

Multiple Choice Single Answer Question Remittance against imports should be completed within Correct Answer 6 months from the data of import Your Answer

6 months from the data of import

True/False Question

WTO is simply renaming of GATT an d the there is no major difference Correct Answer False Your Answer

False

Multiple Choice Single Answer Question One of the major issues related to the controversy of GATT was the issue of Correct Answer Agricultural subsidies Your Answer

Agricultural subsidies

Multiple Choice Single Answer Question Where the import is in a non-physical form(e.g. data over internet) :Correct Answer A certificate from the chartered accountant to the effect that the data/design/software has been received by the importer, and is acceptable, is required Your Answer A certificate from the chartered accountant to the effect that the data/design/software has been received by the importer, and is acceptable, is required Multiple Choice Multiple Answer Question If after taking all the efforts, the export bill is not realized to write off the bill with supporting documentary evidence the bank will consider the case subject to following conditions :Correct Answer The bill is outstanding for over a year , The total amount of write-off allowed by the bank in a year does not exceed 10% of the average annual export realization of the exporter, during the preceding three calendar years , Exporter has submitted satisfactory evidence to show that he has taken all the possible efforts Your Answer The bill is outstanding for over a year , The total amount of write-off allowed by the bank in a year does not exceed 10% of the average annual export realization of the exporter, during the preceding three calendar years , Exporter has submitted satisfactory evidence to show that he has taken all the possible efforts Multiple Choice Single Answer

Question

The theory of comparative costs was put forth by :-

Correct Answer Classical economists Your Answer

Classical economists

True/False Question

Consignment exports means goods dispatched to the agent of the exporter on sale or return basis Correct Answer True Your Answer

True

Multiple Choice Single Answer Question Export of goods causes :Correct Answer Inflow of foreign exchange Your Answer

Inflow of foreign exchange

Multiple Choice Multiple Answer Question The general council will serve following main functions: Correct Answer To act as dispute settlement body , To establishment goods council , To supervise on a regular basis the operations of revised agreements Your Answer To act as dispute settlement body , To establishment goods council , To supervise on a regular basis the operations of revised agreements Multiple Choice Multiple Answer Question The WTO deals with the special needs of the developing countries in three ways Correct Answer It contain special provision on developing countries , It has a committee on trade and development , WTO secretariat provides technical assistance Your Answer It contain special provision on developing countries , It has a committee on trade and development , WTO secretariat provides technical assistance Multiple Choice Single Answer Question In domestic trade :Correct Answer Volume of trade is large Your Answer

Volume of trade is large

Multiple Choice Single Answer Question India's contribution in the World Trade in 1950 was :Correct Answer 0.018

Your Answer

0.018

Multiple Choice Single Answer Question Government approval for Joint Venture with Small Scale sector is given by :Correct Answer District Industrial Centre Your Answer

Director General Foreign Trade

True/False Question

Membership in the world bank is optional & not prerequisite for membership in International Finance Corporation (IFC) Correct Answer False Your Answer

False

Multiple Choice Single Answer Question Imports results in :Correct Answer Outflow of foreign exchange Your Answer

Outflow of foreign exchange

Multiple Choice Single Answer Question What is the general effect of Foreign Trade? Correct Answer It encourages competition Your Answer

It increases lower levels of output

Multiple Choice Single Answer Question Until 1970's India had adopted a :Correct Answer Protectionist policy Your Answer

Free trade policy

Multiple Choice Multiple Answer Question Objectives of Planning Commission were:Correct Answer Augment infrastructure , Formulation of policies for growth , Assessment of available resources Your Answer Augment infrastructure , Formulation of policies for growth , Assessment of available resources Multiple Choice Multiple Answer Question Bertil Ohlin :Correct Answer Contends that there is no difference between domestic and foreign trade , Is not a classical economist , Is a modern economist

Your Answer

Is a modern economist , Contends that there is no difference between domestic and foreign trade , Is not a classical economist

Multiple Choice Multiple Answer Question Labour is not mobile between countries because of :Correct Answer Difference in languages , Difference in customs , Occupational skills Your Answer Difference in languages , Difference in customs , Occupational skills Select The Blank Question The woolen industry in the country is ________ in size and widely scattered Correct Answer Small Your Answer

Small

True/False Question

Export growth was slow due to lack of value addition.

Correct Answer True Your Answer

True

Multiple Choice Multiple Answer Question Result of liberalization is :Correct Answer More export led growth , Encouragement towards foreign investments Your Answer More export led growth , Encouragement towards foreign investments , That on import front too there has been an accelerated libaralization in the trade policy Select The Blank Question The current account deficit in 1999 was ________ of GDP. Correct Answer 0.009 Your Answer

0.9.5%

Multiple Choice Multiple Answer Question Classical economists :Correct Answer Say that there is a fundamental difference between domestic and foreign trade. , Have put forward the theory of comparative costs , Differ from modern economists Your Answer Say that there is a fundamental difference between domestic and foreign trade. , Have put forward the theory of comparative costs , Differ from modern economists Multiple Choice Single Answer

Question

The data regarding sale and purchase of foreign currencies by FCNR account holders is maintained by :Correct Answer DEIO Your Answer

Authorised Dealers

Select The Blank Question With a view to boost agricultural exports and provide remunerative returns to the farming community, the exportimport policy 2002-2007 proposed the setting up of________ Correct Answer Agricultural Export Zones Your Answer

Agricultural Export Zones

Multiple Choice Single Answer Question Asian Development bank was set up in December 1966 under the auspices of the Correct Answer United Nations Economics Commission for Asia and Far East ( ECAFE) Your Answer United Nations Economics Commission for Asia and Far East ( ECAFE) Multiple Choice Multiple Answer Question Transportation Credit includes ………. Correct Answer Freight on exports , remittances received by Shipping/Air lines , Remittances received by Foreign Agents of Indian companies Your Answer Freight on exports , remittances received by Shipping/Air lines , Remittances received by Foreign Agents of Indian companies True/False Question

Export Processing Zones cannot be set up by Private sector.

Correct Answer False Your Answer

True

Select The Blank Question Most of the leather exports go to highly industrialized country like________ Correct Answer America Your Answer

America

True/False Question

Trade samples of goods and publicity materials supplied free of payment are exempted from declaration in the form prescribed by RBI Correct Answer True Your Answer

True

Multiple Choice Single Answer Question Foreign Trade is Correct Answer Trade between different countries Your Answer

Trade between different countries

Multiple Choice Single Answer Question The term 'Trade' means :Correct Answer Exchange of goods among people Your Answer

Exchange of goods among people

Select The Blank Question Capital fund flows have been augmented by SBI with floating of ________. Correct Answer India Millennium Deposits Your Answer

India Millennium Deposits

True/False Question

A deficit occurs when Autonomous Transactions give rise to receipts as against payments. Correct Answer False Your Answer

True

Select The Blank Question India's Trade Policy makers have realized that there is no alternative but to give thrust to the ________. Correct Answer Export Your Answer

Export

Match The Following Question

Correct Answer

Your Answer

ECB

External Commercial Borrowings

External Commercial Borrowings

FCNR

Foreign Currency Non Resident Account

Foreign Currency Non Resident Account

SDR

Special Drawings Rights

Special Drawings Rights

GDP

Gross Domestic Product

Gross Domestic Product

Match The Following

Question

Correct Answer

Your Answer

DTA

Domestic Tariff Area

Domestic Tariff Area

MODVAT

Modified Value Added Tax

Modified Value Added Tax

IRPS

International Price Reimbursement Scheme

International Price Reimbursement Scheme

CCS

Cash Compensatory Support Scheme

Cash Compensatory Support Scheme

Select The Blank Question In UP Export Processing Zone is established in ________. Correct Answer Noida Your Answer

Noida

LIST OF ATTEMPTED QUESTIONS AND ANSWERS Select The Blank Question The IFC's role is to stimulate the few of private capital into production private, mixed, private/public enterprises. It acts as a ________ in bringing together entrepreneurship, investment capital E- production Correct Answer Catalyst Your Answer

Catalyst

Multiple Choice Single Answer Question Travel credits are :Correct Answer Foreign exchange received from abroad for tourist/ students Your Answer

Foreign exchange received from abroad for tourist/ students

Select The Blank Question Agency commission is not allowed to be remitted in case of exports under ________ credit route. Correct Answer Rupee Your Answer

Rupee

Multiple Choice Single Answer Question During the second plan period Correct Answer Food grains import had to be resorted to overcome internal shortages Your Answer Food grains import had to be resorted to overcome internal shortages Multiple Choice Single Answer Question One of oldest organized industries is Correct Answer Indian Textiles Your Answer

Indian Textiles

Multiple Choice Single Answer Question Prohibited Imports and Exports are punishable under :Correct Answer Section 11 of the Customs Act Your Answer

Section 11 of the Customs Act

Multiple Choice Single Answer Question The woolen industry in the country is basically located in the state of :Correct Answer Punjab Your Answer

Punjab

Select The Blank Question The gems & jewellery export sector is highly________ oriented Correct Answer Import Your Answer

Import

Multiple Choice Multiple Answer Question The agreements of WTO :Correct Answer Spell out principles of liberalization , Set procedures for settling disputes , Prescribe special treatment for developing countries Your Answer Spell out principles of liberalization , Set procedures for settling disputes , Prescribe special treatment for developing countries True/False Question

Capital Account of BOP has undergone major structural change in favour of Non Debt creating Foreign Investment inflows. Correct Answer True Your Answer

True

Multiple Choice Multiple Answer Question Faceless presence of Indian exporters in International market is denoted by :Correct Answer Maintaining quality , In ability to export in consumer packs , Create market for the brand Your Answer In ability to export in consumer packs , Create market for the brand , Maintaining quality Multiple Choice Single Answer Question Export prices are high because of :Correct Answer Economies of scale not available Your Answer

Economies of scale not available

Select The Blank Question Declaration of export in PP forms are required for exports in physical form through________

Correct Answer Post Your Answer

Post

Multiple Choice Multiple Answer Question During 1990-91, push was given to the export effort resulting in Correct Answer A surplus of Rs 32,558 crores , An increase of 17.7% in exports , Trade deficit of 10,35 crores Your Answer A surplus of Rs 32,558 crores , An increase of 17.7% in exports , Trade deficit of 10,35 crores Multiple Choice Multiple Answer Question Long Terms capital consists of mainly transactions in :Correct Answer Equities , Loans , Real Estate Your Answer

Equities , Loans , Real Estate

True/False Question

Coffee has been identified for making special thrust in foreign markets Correct Answer False Your Answer

False

True/False Question

India has a natural competitive advantage in agricultural exports because of lower need of inputs, reasonable labour cost and diverse agro-climatic conditions. Correct Answer True Your Answer

True

Select The Blank Question International Gold Standard is one form of ________. Correct Answer Fixed exchange System for adjustment of BOP Your Answer

Fixed exchange System for adjustment of BOP

True/False Question

Under the Gold Standard burden of adjustment is on Internal Economy. Correct Answer True Your Answer

True

Multiple Choice Single Answer Question Openness of an economy with respect to foreign trade refers to its Correct Answer Permissiveness towards exports & imports Your Answer

Permissiveness towards exports & imports

Multiple Choice Single Answer Question On the eve of planning 1948-49 to 1950-51 there was a rise in imports largely due to Correct Answer Pent up demand of the war and the post war period Your Answer

Pent up demand of the war and the post war period

Select The Blank Question Santa Cruz Export Processing Zone established in ________. Correct Answer 1974 Your Answer

1974

Select The Blank Question The Invisible transactions except Insurance and Transportation are recorded on ________ basis. Correct Answer Gross Your Answer

Gross

Match The Following Question

Correct Answer

Your Answer

Developed countries such as Japan, USA, Canada, West Germany, Australia etc.

A range of financial services companies and provide technical assistance

Contributes to funds of ADB

Asian development bank Make its investments in Development of the (ADB) has been partnership with investors developing countries under establishment to from the capital exporting its region country features of IFC's assistance

Facilitate free trade within Make its investments in countries partnership with investors from the capital exporting country

In India, IFC will invest in Development of the developing countries under its region

A range of financial services companies and provide technical assistance

Select The Blank Question ________ has commented on adverse impact of exports due to technological backwardness. Correct Answer Tandon committee Your Answer

Tandon committee

True/False Question

International Monetary Fund was established to create a monetary system that would provide relief and reconstruction to the countries affected by World War. Correct Answer False Your Answer

False

Multiple Choice Single Answer Question India is world leader in :Correct Answer Tea production Your Answer

Tea production

Select The Blank Question During ________ plan period the average annual trade deficit reached its peak Correct Answer Seventh Your Answer

Ninth

Multiple Choice Single Answer Question The invisible Accounts consists of services provided such as :Correct Answer Travel, Gifts etc Your Answer

Travel, Gifts etc

Multiple Choice Single Answer Question Whenever under the trade policy, the import of any item is subjected to an import the banks should not open any letter of credit or effect any remittance unless the original Correct Answer Exchange control copy of the license is submitted

Your Answer

Exchange control copy of the license is submitted

True/False Question

Membership of IMF is the principle condition for membership of the world bank Correct Answer True Your Answer

True

Multiple Choice Single Answer Question Letters of credit can be opened and/or remittances can be effected by a person other than the importer himself :Correct Answer On the strength of the letter of authority issued by the importer in the name of such a person Your Answer On the strength of the letter of authority issued by the importer in the name of such a person Multiple Choice Single Answer Question The principles of non-discriminations of WTO requires that Correct Answer All trading partners shall be granted the most fevoured nation status (MFN) Your Answer All trading partners shall be granted the most fevoured nation status (MFN) Multiple Choice Single Answer Question CIS stands for Correct Answer Commonwealth of independent states Your Answer

Commonwealth of independent states

Multiple Choice Multiple Answer Question Measures which could be taken for reducing disequilibrium in BOP as per Pegged System are :Correct Answer Raising interest rates , Discourage imports , Encourage Capital inflows. Your Answer Raising interest rates , Discourage imports , Encourage Capital inflows. Multiple Choice Multiple Answer Question The value of rupee was devalued by 36.5% in June 1966 because of Correct Answer Persistent adverse balance of trade since 1951 , Acute shortage of foreign exchange , Extensive borrowing by India from foreign countries Your Answer Persistent adverse balance of trade since 1951 , Acute shortage of foreign exchange , Extensive borrowing by India from foreign countries

True/False Question

By being part of WTO, India does not enjoy many benefits.

Correct Answer False Your Answer

False

Multiple Choice Single Answer Question FCNR Operations details are sent to RBI on :Correct Answer STAT 6 From Your Answer

STAT 6 From

Select The Blank Question Massive programmes of industrialization were initiated during ________ plan period. Correct Answer Second Your Answer

Second

Multiple Choice Multiple Answer Question The status holders will be eligible for the following new facilities Correct Answer Fixation of input-output norms on priority , 100% retention of foreign exchange in EEFC account , Enhancement in normal repatriation period from 180 to 360 days Your Answer Fixation of input-output norms on priority , 100% retention of foreign exchange in EEFC account , Enhancement in normal repatriation period from 180 to 360 days Multiple Choice Multiple Answer Question Mechanisms available for adjustments of BOP are :Correct Answer Changes in Income , Changes in Prices , Changes in Interest Rates Your Answer Changes in Income , Changes in Prices , Changes in Interest Rates Multiple Choice Multiple Answer Question Exporters are entitled to following subsidies :Correct Answer Air Freight subsidy , Export Credit Interest rates , Excise duties Your Answer True/False

Air Freight subsidy , Export Credit Interest rates , Excise duties

Question

For the exports to Nepal and Bhutan payment has to be in their respective currencies Correct Answer False Your Answer

True

Select The Blank Question GR and SDF forms in duplicate are to be submitted to the ________ Correct Answer Commissioner of Customs Your Answer

Commissioner of Customs

Multiple Choice Single Answer Question For exchange control purposes the Rupee account of Bhutanese and Nepalese nationals and firms, maintained with any Indian Banks are treated as Correct Answer Resident account Your Answer

Resident account

Select The Blank Question India was forced to devalue the rupee by 36.5 percent in June________ Correct Answer 1966 Your Answer

1966

Match The Following Question

Correct Answer

Your Answer

one major controversy of Agricultural subsidies GATT is the issue related to

Agricultural subsidies

The WTO agreement has many provisions granting special favours to developing & learnt developed countries

Learnt developed countries are exempted from fulfilling their commitments

Developing countries have been given extra time to fulfill their comments

Uruguay Round of agreements have come in India & many politicians have agreed that

Developing countries have been given extra time to fulfill their comments

India should withdraw from GATT

The Uruguay round mostly benefit

Developed countries

Developed countries

LIST OF ATTEMPTED QUESTIONS AND ANSWERS Multiple Choice Single Answer Question Balance of Payment is :Correct Answer Analysis of monetary aspects of Foreign trade Your Answer

Analysis of monetary aspects of Foreign trade

Select The Blank Question Capital Account consists of all transactions of ________ nature. Correct Answer Financial Your Answer

Temporary

True/False Question

Free trade policy is not suited to developing & underdeveloped countries because the industries of the developed countries need protection from well established and well organized industries of the developed countries. Correct Answer True Your Answer

True

Select The Blank Question The important distinction between domestic trade and foreign trade relates to difference in ________. Correct Answer Currency Your Answer

Currency

True/False Question

In case of export of goods on consignment basis, the freight and insurance is arranged by the agent of exporter in his country. Correct Answer False Your Answer

True

Multiple Choice Single Answer Question Japan is our major market for :Correct Answer Marine products exports Your Answer

Marine products exports

Select The Blank Question Foreign Trade (Development and Regulation) Act 1992 came into force on ________.

Correct Answer 6/19/2005 Your Answer

6/19/2005

Multiple Choice Multiple Answer Question Exports requiring prior approval of Reserve Bank / Govt authorities are :Correct Answer Counter Trade Agreements , Project Exports , Exports under trade agreements between Govt. of India and the Govt of Foreign state Your Answer Counter Trade Agreements , Project Exports , Exports under trade agreements between Govt. of India and the Govt of Foreign state Select The Blank Question As stated in FERA 1973, the objectives of exchange control in India is ________ of foreign exchange resources of the country and proper utilizations Correct Answer Conservation Your Answer

Conservation

Multiple Choice Single Answer Question The secretariat of SAARC is at Correct Answer Kathmandu, Nepal Your Answer

Kathmandu, Nepal

Multiple Choice Multiple Answer Question Double weightage on FOB on the export of :Correct Answer Products from small/ cottage units , Fruits and vegetables , Horticultural products Your Answer Products from small/ cottage units , Fruits and vegetables , Horticultural products True/False Question

Capital Account covers all transactions relating to India's foreign financial assets and liabilities. Correct Answer True Your Answer

True

Multiple Choice Single Answer Question Imports are recorded in the bo oks of accounts when:Correct Answer Payment is received Your Answer

Payment is received

Select The Blank Question Free trade benefits the ________ because they are able to buy a variety of commodities from abroad and at economical prices. Correct Answer Consumers Your Answer

Consumers

Multiple Choice Multiple Answer Question Policies relating to commerce, trade, taxation :Correct Answer Are same within the country , Different outside the country , Incorporate artificial barriers in form of quotas import duties Your Answer Are same within the country , Different outside the country , Incorporate artificial barriers in form of quotas import duties Match The Following Question

Correct Answer

Your Answer

Value added exports

Finer processing and Incorrect branding packing for earning higher export returns

Faceless presence

Indian raw material with foreign brand name

Unreliability

Lack of prompt After sales Lack of prompt After sales service service

Problem recognition

Lack of innovativeness

Finer processing and packing for earning higher export returns

Lack of innovativeness

Select The Blank Question Long Term Capital involves International transfer of ________ Correct Answer Purchasing power Your Answer

Purchasing power

Match The Following Question

Correct Answer

Your Answer

IEC

Importer Exporter Code

Importer Exporter Code

NDDB

National Dairy Development Board

National Dairy Development Board

MML

Minerals & Metals Ltd

Minerals & Metals Ltd

CCIE

Chief controller of Imports Chief controller of Imports and Exports and Exports

Select The Blank Question The economic argument in favor of protectionism is that the

infant industries have to be protected against stiff________ Correct Answer Competition Your Answer

Import duty

Select The Blank Question India gain due to membership of WTO of other developing countries like china because the ________ ratio of India is low Correct Answer Trade-GDP Your Answer

Trade-GDP

Multiple Choice Single Answer Question Indian products are not sold in Foreign Market because of :Correct Answer Traditional approach Your Answer

Traditional approach

Multiple Choice Single Answer Question Merchandise trade means :Correct Answer Transactions relating to movable goods Your Answer

Transactions relating to movable goods

True/False Question

Foreign Trade helps to understand the life style, culture, philosophies, social & political institutions of the different countries Correct Answer True Your Answer

True

Multiple Choice Multiple Answer Question Free Trade refers to a situation :Correct Answer Where there is unrestricted exchange of goods and services between countries. , Where the Govt. follows a policy of noninterference , Where there is absence of tariffs quotas, exchange restriction taxes and subsidies on production Your Answer Where there is unrestricted exchange of goods and services between countries. , Where the Govt. follows a policy of noninterference , Where there is absence of tariffs quotas, exchange restriction taxes and subsidies on production Multiple Choice Multiple Answer Question The country has recorded a rapid growth in the manufactures of chemicals during the last few years. Prominent among these are

Correct Answer Caustic soda , Soda ash , Carbon black Your Answer

Caustic soda , Soda ash , Carbon black

Multiple Choice Single Answer Question BIS stands for Correct Answer Bank for International Settlements Your Answer

Bank for International Settlements

Multiple Choice Multiple Answer Question Units manufacturing capital goods and consumer durables are scattered all over the country but they are concentrated mainly in Correct Answer Maharashtra , West Bengal , Karnataka Your Answer

Maharashtra , West Bengal , Uttar Pradesh

Multiple Choice Multiple Answer Question The gems and Jewellery export sector is highly import oriented, import of item constituting raw materials for this industry are namely :Correct Answer Rough diamonds , Rough precious and semi precious stones , Pearls Your Answer Rough diamonds , Rough precious and semi precious stones , Pearls True/False Question

Free Trade is the best policy for economic development.

Correct Answer True Your Answer

True

Multiple Choice Multiple Answer Question The woolen industry in the country is small in size. It is basically located in the states of Correct Answer Punjab , Haryana , Rajasthan Your Answer

Punjab , Haryana , Rajasthan

Select The Blank Question Manganese Ores with more than ________ Manganese is canalised. Correct Answer 0.46 Your Answer

0.46

Multiple Choice Single Answer

Question

Cost of production is higher in India because of :-

Correct Answer Time and Cost over runs Your Answer

Time and Cost over runs

Multiple Choice Multiple Answer Question The three corresponding agreements to WTO are Correct Answer GATT , GATS , General Agreement on Trade Related aspects of Intellectual Property Rights Your Answer GATT , GATS True/False Question

The international monetary fund is an organization of countries that seeks to promote international monetary cooperation Correct Answer True Your Answer

True

True/False Question

IRPS is replaced by Engineering Products (Replenishment of Iron and Steel Intermediates) Scheme. Correct Answer True Your Answer

True

Multiple Choice Multiple Answer Question In 1997-98 there was a decline in world trade due to :Correct Answer South east Asian crisis , Continued recession in Japan , Several economic crisis in Russia Your Answer South east Asian crisis , Continued recession in Japan , Several economic crisis in Russia Multiple Choice Multiple Answer Question In BOP surplus nations International Payment equilibrium creates:Correct Answer Increase in price level , Increase in Income Level , Fall in Interest rates Your Answer Increase in price level , Increase in Income Level , Fall in Interest rates Multiple Choice Single Answer Question Duty drawback/exemption avoids :Correct Answer Commodity taxes like excise duty Your Answer

Commodity taxes like excise duty

Multiple Choice Single Answer

Question

Private Resident & non profit bodies comprises of people engaged in :Correct Answer Social, healthcare activities Your Answer

Social, healthcare activities

Select The Blank Question Nearly________ % of the member of WTO are developing countries Correct Answer 80 Your Answer

80

Multiple Choice Single Answer Question India commands an important place in would trade in Correct Answer Tea Your Answer

Tea

True/False Question

Handicrafts are exported to more than 100 countries out of which 11 countries buy and absorb about 80% of India's export of Handicraft Correct Answer True Your Answer

True

Multiple Choice Multiple Answer Question FDI may involve :Correct Answer Transfer of financial assets , Transfer of trademark , Transfer of technology Your Answer Transfer of financial assets , Transfer of trademark , Transfer of technology Multiple Choice Single Answer Question Non traditional sector consists broadly of:Correct Answer Consumer electronics Your Answer

Consumer electronics

Multiple Choice Multiple Answer Question Special Import License based on net foreign exchange is entitled as follows :Correct Answer Export House 6% on FOB , Export House 7.5% on NFE , Superstar Trading House 12% on FOB Your Answer Export House 6% on FOB , Export House 7.5% on NFE , Superstar Trading House 12% on FOB

Select The Blank Question Readymade Garments are still a product of the ________ and decentralized sectors Correct Answer Small Your Answer

Small

14.

Cement and Steel prices were de-controlled in :-

a)

1980-82

b)

1981-82

c)

1982-83

d)

1979-80

As per estimates of IMF if oil price rises by USD 5 per barrel it will amount to

25.

reduced GDP in industrial countries by

34.

During

43.

.

plan period the average annual trade deficit reached its peak

Current Account is regarded as most basic of all sub accounts because :-

a)

Its volumes are bigger in size

b)

It contains all transactions giving rise to countries national income

c)

It helps movement of long term capital goods

d)

It is a day to day activity

45.

has commented on adverse impact of exports due to technological backwardness.

49.

FCNR deposit's details are sent to RBI Monthly on :-

a)

STAT 5 From

b)

STAT 4 From

c)

STAT 3 Form

d)

STAT 10 Form

.

The theory of comparative costs was put forth by :-

a)

Modern economists

b)

Traditional economists

c)

Classical economists

d)

Conservative economists

51.

Faceless presence of Indian exporters in International market is denoted by :-

a)

In ability to export in consumer packs

b)

Create market for the brand

c)

Maintaining quality

d)

Dominance of MNCs

LIST OF ATTEMPTED QUESTIONS AND ANSWERS Multiple Choice Multiple Answer Question India Balance of Payments in 1999-2001 remained comfortable because of :Correct Answer Increase is software exports , Recovery of exports , Net inflow of invisibles Your Answer Sale of Gold in open market , Recovery of exports , Increase is software exports True/False Question

Demonetisation means transfer of gold from Non-Monetary to Monetary stock of reserves. Correct Answer False Your Answer

False

Select The Blank Question Canalised items are imported or exported only through ________. Correct Answer Public Sector agencies

Your Answer

Public Sector agencies

Select The Blank Question Government has allowed Overseas Business operation through issue of ________. Correct Answer ADR/DGR Your Answer

Removal of restrictions

True/False Question

CCS scheme was abolished following the economic reforms.

Correct Answer True Your Answer

False

Multiple Choice Single Answer Question What is the general effect of Foreign Trade? Correct Answer It encourages competition Your Answer

It encourages competition

Select The Blank Question The IFC's role is to stimulate the few of private capital into production private, mixed, private/public enterprises. It acts as a ________ in bringing together entrepreneurship, investment capital E- production Correct Answer Catalyst Your Answer

Catalyst

Multiple Choice Multiple Answer Question In the context of fish exports Correct Answer India is the seventh largest production of fish , Second in inland fish , Japan is the major market Your Answer India is the seventh largest production of fish True/False Question

Coffee has been identified for making special thrust in foreign markets Correct Answer False Your Answer

True

Multiple Choice Single Answer Question Imports results in :Correct Answer Outflow of foreign exchange Your Answer

Outflow of foreign exchange

Multiple Choice Multiple Answer Question As a further step in the path of liberalization Reserve Bank of India has announced on 19th August 1994 :Correct Answer relaxation on the current account transactions and further delegation of powers , The authorized dealers have been permitted to allow remittances for various purposes , The country has achieved full convertibility on current account Your Answer relaxation on the current account transactions and further delegation of powers , The authorized dealers have been permitted to allow remittances for various purposes , The country has achieved full convertibility on current account Select The Blank Question License not required if location of project is beyond 25 Kms periphery from urban area having population of ________. Correct Answer 1 million Your Answer

1 million

Multiple Choice Multiple Answer Question Risks and uncertainties in the external environment are :Correct Answer Uneven GDP Growth pattern , Demand in major currencies like Dollars , Misalignment between EURO and US Dollar Your Answer Uneven GDP Growth pattern , Demand in major currencies like Dollars , Misalignment between EURO and US Dollar Match The Following Question

Correct Answer

IMD

India Millennium Deposits India Millennium Deposits

FER

Foreign Exchange Reserves

Foreign Exchange Reserves

GDPmp

Gross Domestic Product

Gross Domestic Product

FII

Foreign Institutional Investor

Foreign Institutional Investor

True/False Question

Your Answer

Capital Account covers all transactions relating to India's foreign financial assets and liabilities.

Correct Answer True Your Answer

True

Multiple Choice Multiple Answer Question During 1990-91, push was given to the export effort resulting in Correct Answer A surplus of Rs 32,558 crores , An increase of 17.7% in exports , Trade deficit of 10,35 crores Your Answer An increase of 17.7% in exports Multiple Choice Single Answer Question The following country is the founder member of IBRD Correct Answer India Your Answer

USA

Multiple Choice Single Answer Question Export of goods causes :Correct Answer Inflow of foreign exchange Your Answer

Inflow of foreign exchange

Multiple Choice Single Answer Question Foreign institutions like the IMF, IBRD, World Bank, GATT have been established after :Correct Answer The second world war Your Answer

The second world war

Multiple Choice Multiple Answer Question In BOP Official Capital Account consists of following items :Correct Answer Loans , Amortization , Miscellaneous Your Answer

Loans , Amortization , Defence Equipment

Multiple Choice Multiple Answer Question Important Technological factors for low exports are :Correct Answer High cost of production , Low productivity , Poor infrastructure Your Answer

High cost of production , Low productivity , Poor infrastructure

Multiple Choice Single Answer

Question

Until 1970's India had adopted a :-

Correct Answer Protectionist policy Your Answer

Protectionist policy

Multiple Choice Single Answer Question In domestic trade :Correct Answer Volume of trade is large Your Answer

Volume of trade is large

Multiple Choice Single Answer Question The term 'Trade' means :Correct Answer Exchange of goods among people Your Answer

Exchange of goods among people

Multiple Choice Multiple Answer Question As per Double Entry Accounting System timings for recording of various transactions is as follows :Correct Answer Current account- when legal ownership changes , Interest and Dividends- when they are due for payment , Unrequited transfers -when change of ownership takes place Your Answer Current account- when legal ownership changes True/False Question

Adverse International economic situation immediately affects Indian markets and exports. Correct Answer True Your Answer

True

Multiple Choice Single Answer Question In Jan 1997 Foreign equity participation in automatic route raised to :Correct Answer 0.74 Your Answer

0.71

Select The Blank Question Nearly________ % of the member of WTO are developing countries Correct Answer 80

Your Answer

80

Multiple Choice Single Answer Question Exports of Super Phosphates is :Correct Answer Permitted Under License Your Answer

Permitted Under License

True/False Question

IRPS compensates the exporters price difference between domestic market and International market on purchase of raw materials. Correct Answer True Your Answer

False

Multiple Choice Single Answer Question Non Resident External Rupee Account and Foreign Currency Non resident Account fall under :Correct Answer Private Capital Your Answer

Private Capital

Multiple Choice Single Answer Question India is the :Correct Answer Seventh largest producer of fish in the world Your Answer

Seventh largest producer of fish in the world

True/False Question

Export Processing Zones cannot be set up by Private sector.

Correct Answer False Your Answer

True

Multiple Choice Single Answer Question Openness of an economy with respect to foreign trade refers to its Correct Answer Permissiveness towards exports & imports Your Answer True/False

Permissiveness towards exports & imports

Question

Asian Development Bank is established to assist development of the developing countries under its region Correct Answer True Your Answer

True

True/False Question

Short terms Capital involves movement of ownership with less than one year maturity. Correct Answer True Your Answer

True

Multiple Choice Single Answer Question The annual average value of imports during the first plan period was Correct Answer Rs 730 cores Your Answer

Rs 730 cores

Match The Following Question

Correct Answer

Your Answer

FIPB

Foreign Investment Promotion Board

Foreign Investment Promotion Board

PAB

Project Approval Board

Project Approval Board

SAP

Structural Adjustment Programme

Structural Adjustment Programme

PMP

Phased Manufacturing Programme

Phased Manufacturing Programme

Select The Blank Question The equilibrium in BOP exists only when Debit is equal to Credit without compensatory ________. Correct Answer Capital Transfers Your Answer

Devaluation of currency

Select The Blank Question The Invisible transactions except Insurance and Transportation are recorded on ________ basis. Correct Answer Gross Your Answer

Approximation

Multiple Choice Single Answer Question Survey of Unclassified Receipt means :Correct Answer Classification of receipts reported in R returns for the purpose of BOP Your Answer Investigation of non reported transactions Multiple Choice Multiple Answer Question Result of liberalization is :Correct Answer More export led growth , Encouragement towards foreign investments Your Answer More export led growth , Encouragement towards foreign investments , That on import front too there has been an accelerated libaralization in the trade policy Select The Blank Question In India imports are recorded on ________ basis. Correct Answer CIF Your Answer

CIF

Multiple Choice Single Answer Question Until the following country became the member of the world bank in 1980, India was the largest beneficiary Correct Answer China Your Answer

China

True/False Question

Foreign Trade helps to understand the life style, culture, philosophies, social & political institutions of the different countries Correct Answer True Your Answer

True

Multiple Choice Multiple Answer Question The gems and Jewellery export sector is highly import oriented, import of item constituting raw materials for this industry are namely :Correct Answer Rough diamonds , Rough precious and semi precious stones , Pearls Your Answer Rough diamonds , Rough precious and semi precious stones Multiple Choice Single Answer

Question

Foreign Trade is

Correct Answer Trade between different countries Your Answer

Trade between different countries

LIST OF ATTEMPTED QUESTIONS AND ANSWERS Multiple Choice Single Answer Question Who issues guidelines for Direct Investments abroad by any Indian? Correct Answer Exchange Control Department of RBI Your Answer Exchange Control Department of RBI Multiple Choice Multiple Answer Question What does a speculator accept while dealing with an option? Correct Answer The exercise or strike price, the amount , The expiration date and time, the method of payment Your Answer The exercise or strike price, the amount , The expiration date and time, the method of payment Match The Following Question Correct Answer Your Answer Returning NRI RFC INVESTMENT IN MMMF NRI Deposits FCNR, NRE Investment in GDR, ADR NRIS AND OCBS INVESTMENT IN MMMF FCA EXPORTERS EEFC FCNR, NRE True/False Question The Negotiating bank examines the documents submitted by the seller and makes payment if they comply the terms and conditions of the LC. Correct Answer True Your Answer True True/False Question Cash flows generated from aforeign project may replace revenue-producing imports to the host country Correct Answer False Your Answer True Select The Blank Question Global capital market can be understood from the study of ________. Correct Answer Yen,Euro Dollar, Swiss Francs, USD market, Govt Borrowings within and outside the country Your Answer Yen,Euro Dollar, Swiss Francs, USD market, Govt Borrowings within and outside the country Select The Blank Question Under the ________ standard the Country's Central Bank promised to buy back the currency at certain rate of Gold. Correct Answer Gold Your Answer Gold Multiple Choice Single Answer

Question What is the meaning of right to buy or sell the future contract? Correct Answer An Option Your Answer An Option Multiple Choice Single Answer Question Financial management is complex process for Correct Answer Companies which have international trade Your Answer Companies which have international trade True/False Question Bid rate is bidders buying rate and offered rate is bidders selling rate. Correct Answer True Your Answer True Multiple Choice Single Answer Question A Dealer Buys commodity in Bombay and sells it in Delhi due to price differentials. He is said to be resorting to which practice? Correct Answer Arbitrage Your Answer Cash market True/False Question Head office of ICC is in Paris because of its First President Eliance Clementel. Correct Answer True Your Answer True Multiple Choice Single Answer Question Under which category of exchange rate did the Indian Rupee came from June 92? Correct Answer Other managed Floating Your Answer Pegged to Single currency Select The Blank Question A Credit can be a confirmed credit only if it is also ________. Correct Answer An Irrevocable credit Your Answer Anticipatory Credit Select The Blank Question ________ is the price at which the market maker is prepared to borrow the money. Correct Answer Bid rate Your Answer Selling price True/False Question Sandby credis are introduced by US Banks because they were not permitted to issue on demand guarantees. Correct Answer True Your Answer True Multiple Choice Single Answer Question How does ECGC protect exporters against risks for export finance

Correct Answer Issuing various insurance policies covering different risks connected with exports at nominal premium Your Answer Issuing various insurance policies covering different risks connected with exports at nominal premium Multiple Choice Single Answer Question What are the functions of Export Import Bank of India? Correct Answer Finance functions and development functions Your Answer Commercial Banking, development Banking functions Multiple Choice Multiple Answer Question What were the primary objectives of the group of seven countries? Correct Answer Bring down the external value of US Dollar , Bring some stability to exchange rates of all the countries Your Answer Bring down the external value of US Dollar , Bring some stability to exchange rates of all the countries Select The Blank Question ________ is the agency of ICC which deals with the maritime crime. Correct Answer International Maritime Bureau Your Answer International Maritime Bureau Multiple Choice Single Answer Question Samuraibonds are Correct Answer Foreign yen bonds issued to non resident entities in Japanese Market Your Answer Yen bonds issued in USA Multiple Choice Single Answer Question Name the members of Exchange Banks association :Correct Answer Foreign Banks at Bombay, Calcutta, Delhi, Madras, Amritsar, Your Answer Foreign Banks at Bombay, Calcutta, Delhi, Madras, Amritsar, Multiple Choice Multiple Answer Question Export finance Correct Answer Preshipment credit , Post shipment credit , Duty Draw back Your Answer Post shipment credit , Duty Draw back Multiple Choice Single Answer Question How many Countries participated in the third survey on Foreign Exchange, coordinated by BIS in 1992? Correct Answer 26 Your Answer 25 Select The Blank Question FRNs are issued for a maturity period of ________ and are normally redeemed in one shot. Correct Answer 5-7 years Your Answer 1 to 3 years Multiple Choice Single Answer

Question What was the official foreign currency reserve of the group of seven central Banks taken togeather in mid 1992? Correct Answer USD 275 Billion Your Answer USD 275 Billion Multiple Choice Single Answer Question What is the volume of the world's total speculative transactions? Correct Answer Roughly 90 % of the total turnover Your Answer 70% of the total turnover Multiple Choice Single Answer Question Which Indian Co. is not eligible to make investment abroad under automatic route? Correct Answer Co which is caution listed by RBI Your Answer Co. which may not have proposed activity as core activity Multiple Choice Multiple Answer Question The maximum period for which a forward contract can be granted by the AD depends upon which RBI directives? Correct Answer Maximum Period of repatriation of export proceeds , Maximum Period of payment for Imports Your Answer Maximum Period of payment for Imports , The need of the importer Multiple Choice Multiple Answer Question As regards the due date of an Option exercise which methods are in vogue? Correct Answer American option , European Option Your Answer American option , European Option , Exercise date Multiple Choice Single Answer Question Which is the most actively traded currency in the international market today? Correct Answer US Dollar Your Answer US Dollar Multiple Choice Multiple Answer Question Mr A sells an option conferring the right to buy a kilogram of silver at Rs 8000/- for next six months means :Correct Answer The buyer of option can buy silver from Mr A at 8000/- per Kg, in next 6 months, even if the price goes up, , The buyer of option can sell silver to Mr A at 8000/- per Kg, in next 6 months, even if the price goes down. Your Answer The buyer of option can buy silver from Mr A at 8000/per Kg, in next 6 months, even if the price goes up, , The buyer of option can sell silver to Mr A at 8000/- per Kg, in next 6 months, even if the price goes down. True/False Question Funding Options in domestic yen market are Futures, FRAs, Call and put options. Correct Answer True Your Answer True Multiple Choice Single Answer

Question Whose permission was necessary for forming the Association of Foreign Exchange Dealers, in India? Correct Answer Reserve Bank of India Your Answer Reserve Bank of India Match The Following Question Correct Answer Your Answer 16 principles for good environmental conduct Business Charter of sustainable development Business Charter of sustainable development World Chambers Congress World Federation of Chambers World Federation of Chambers ICC Conference in 2004 Marrakesh Marrakesh 13 types of trade contracts INCOTERM Commercial crime Bureau Select The Blank Question Hotels and Airline companies are allowed to receive payments in Indian rupees from foreign tourist without insis iting on ________. Correct Answer Encashment certificate Your Answer Foreign currency Multiple Choice Single Answer Question Fixed, Floating or Limited flexibility are used in the context of Correct Answer Exchange Rates Your Answer Exchange Rates Multiple Choice Multiple Answer Question Internationally only those currencies are in demand Correct Answer Which are convertible , Which have liquidity confirmed in Euro market operations Your Answer Which are convertible , Which have greater intrinsic value Select The Blank Question Forward Margin makes a currency costlier then the currency is at ________. Correct Answer Premium Your Answer Loss Select The Blank Question The rate of interest on rupee finance for exports is not exceeding ________ less than PLR. Correct Answer 0.025 Your Answer 50 basis points over LIBOR Select The Blank Question The creditworthiness of futures exchange is maintained through imposition of ________. Correct Answer Margin Your Answer Penalty

True/False Question The volume of trading of Derivatives in the world trade market is growing but still only 5% of the total. Correct Answer False Your Answer True True/False Question With the advent of futures and Options there is no need for additional instruments of risk hedging. Correct Answer False Your Answer True True/False Question Charter party bills of lading are generally not acceptable to bankers as security because Sea Charters are full of problems. Correct Answer True Your Answer True Multiple Choice Multiple Answer Question In which cases can a Resident Individual in India acquire foreign securities without prior approval? Correct Answer Gift from a person outside India, or Inheritance from a person resident in or outside India , Security issued under Cashless employees stock option scheme Your Answer Purchased from NRO/NRNR accounts , Purchase from borrowings from Authorised dealers Select The Blank Question Over the years ________has been the most widely accepted currency in international capital market. Correct Answer American Dollar Your Answer American Dollar True/False Question The American Dollar market comprises of the domestic segment of USD the offshore segment of USD. Correct Answer True Your Answer True

LIST OF ATTEMPTED QUESTIONS AND ANSWERS Multiple Choice Single Answer Question Who issues guidelines for Direct Investments abroad by any Indian? Correct Answer Exchange Control Department of RBI Your Answer Exchange Control Department of RBI Multiple Choice Multiple Answer Question What does a speculator accept while dealing with an option? Correct Answer The exercise or strike price, the amount , The expiration date and time, the method of payment Your Answer The exercise or strike price, the amount , The expiration date and time, the method of payment Match The Following Question Correct Answer Your Answer Returning NRI RFC INVESTMENT IN MMMF NRI Deposits FCNR, NRE Investment in GDR, ADR NRIS AND OCBS INVESTMENT IN MMMF FCA EXPORTERS EEFC FCNR, NRE True/False Question The Negotiating bank examines the documents submitted by the seller and makes payment if they comply the terms and conditions of the LC. Correct Answer True Your Answer True True/False Question Cash flows generated from aforeign project may replace revenue-producing imports to the host country Correct Answer False Your Answer True Select The Blank Question Global capital market can be understood from the study of ________. Correct Answer Yen,Euro Dollar, Swiss Francs, USD market, Govt Borrowings within and outside the country Your Answer Yen,Euro Dollar, Swiss Francs, USD market, Govt Borrowings within and outside the country Select The Blank Question Under the ________ standard the Country's Central Bank promised to buy back the currency at certain rate of Gold. Correct Answer Gold Your Answer Gold Multiple Choice Single Answer

Question What is the meaning of right to buy or sell the future contract? Correct Answer An Option Your Answer An Option Multiple Choice Single Answer Question Financial management is complex process for Correct Answer Companies which have international trade Your Answer Companies which have international trade True/False Question Bid rate is bidders buying rate and offered rate is bidders selling rate. Correct Answer True Your Answer True Multiple Choice Single Answer Question A Dealer Buys commodity in Bombay and sells it in Delhi due to price differentials. He is said to be resorting to which practice? Correct Answer Arbitrage Your Answer Cash market True/False Question Head office of ICC is in Paris because of its First President Eliance Clementel. Correct Answer True Your Answer True Multiple Choice Single Answer Question Under which category of exchange rate did the Indian Rupee came from June 92? Correct Answer Other managed Floating Your Answer Pegged to Single currency Select The Blank Question A Credit can be a confirmed credit only if it is also ________. Correct Answer An Irrevocable credit Your Answer Anticipatory Credit Select The Blank Question ________ is the price at which the market maker is prepared to borrow the money. Correct Answer Bid rate Your Answer Selling price True/False Question Sandby credis are introduced by US Banks because they were not permitted to issue on demand guarantees. Correct Answer True Your Answer True Multiple Choice Single Answer Question How does ECGC protect exporters against risks for export finance

Correct Answer Issuing various insurance policies covering different risks connected with exports at nominal premium Your Answer Issuing various insurance policies covering different risks connected with exports at nominal premium Multiple Choice Single Answer Question What are the functions of Export Import Bank of India? Correct Answer Finance functions and development functions Your Answer Commercial Banking, development Banking functions Multiple Choice Multiple Answer Question What were the primary objectives of the group of seven countries? Correct Answer Bring down the external value of US Dollar , Bring some stability to exchange rates of all the countries Your Answer Bring down the external value of US Dollar , Bring some stability to exchange rates of all the countries Select The Blank Question ________ is the agency of ICC which deals with the maritime crime. Correct Answer International Maritime Bureau Your Answer International Maritime Bureau Multiple Choice Single Answer Question Samuraibonds are Correct Answer Foreign yen bonds issued to non resident entities in Japanese Market Your Answer Yen bonds issued in USA Multiple Choice Single Answer Question Name the members of Exchange Banks association :Correct Answer Foreign Banks at Bombay, Calcutta, Delhi, Madras, Amritsar, Your Answer Foreign Banks at Bombay, Calcutta, Delhi, Madras, Amritsar, Multiple Choice Multiple Answer Question Export finance Correct Answer Preshipment credit , Post shipment credit , Duty Draw back Your Answer Post shipment credit , Duty Draw back Multiple Choice Single Answer Question How many Countries participated in the third survey on Foreign Exchange, coordinated by BIS in 1992? Correct Answer 26 Your Answer 25 Select The Blank Question FRNs are issued for a maturity period of ________ and are normally redeemed in one shot. Correct Answer 5-7 years Your Answer 1 to 3 years Multiple Choice Single Answer

Question What was the official foreign currency reserve of the group of seven central Banks taken togeather in mid 1992? Correct Answer USD 275 Billion Your Answer USD 275 Billion Multiple Choice Single Answer Question What is the volume of the world's total speculative transactions? Correct Answer Roughly 90 % of the total turnover Your Answer 70% of the total turnover Multiple Choice Single Answer Question Which Indian Co. is not eligible to make investment abroad under automatic route? Correct Answer Co which is caution listed by RBI Your Answer Co. which may not have proposed activity as core activity Multiple Choice Multiple Answer Question The maximum period for which a forward contract can be granted by the AD depends upon which RBI directives? Correct Answer Maximum Period of repatriation of export proceeds , Maximum Period of payment for Imports Your Answer Maximum Period of payment for Imports , The need of the importer Multiple Choice Multiple Answer Question As regards the due date of an Option exercise which methods are in vogue? Correct Answer American option , European Option Your Answer American option , European Option , Exercise date Multiple Choice Single Answer Question Which is the most actively traded currency in the international market today? Correct Answer US Dollar Your Answer US Dollar Multiple Choice Multiple Answer Question Mr A sells an option conferring the right to buy a kilogram of silver at Rs 8000/- for next six months means :Correct Answer The buyer of option can buy silver from Mr A at 8000/- per Kg, in next 6 months, even if the price goes up, , The buyer of option can sell silver to Mr A at 8000/- per Kg, in next 6 months, even if the price goes down. Your Answer The buyer of option can buy silver from Mr A at 8000/per Kg, in next 6 months, even if the price goes up, , The buyer of option can sell silver to Mr A at 8000/- per Kg, in next 6 months, even if the price goes down. True/False Question Funding Options in domestic yen market are Futures, FRAs, Call and put options. Correct Answer True Your Answer True Multiple Choice Single Answer

Question Whose permission was necessary for forming the Association of Foreign Exchange Dealers, in India? Correct Answer Reserve Bank of India Your Answer Reserve Bank of India Match The Following Question Correct Answer Your Answer 16 principles for good environmental conduct Business Charter of sustainable development Business Charter of sustainable development World Chambers Congress World Federation of Chambers World Federation of Chambers ICC Conference in 2004 Marrakesh Marrakesh 13 types of trade contracts INCOTERM Commercial crime Bureau Select The Blank Question Hotels and Airline companies are allowed to receive payments in Indian rupees from foreign tourist without insis iting on ________. Correct Answer Encashment certificate Your Answer Foreign currency Multiple Choice Single Answer Question Fixed, Floating or Limited flexibility are used in the context of Correct Answer Exchange Rates Your Answer Exchange Rates Multiple Choice Multiple Answer Question Internationally only those currencies are in demand Correct Answer Which are convertible , Which have liquidity confirmed in Euro market operations Your Answer Which are convertible , Which have greater intrinsic value Select The Blank Question Forward Margin makes a currency costlier then the currency is at ________. Correct Answer Premium Your Answer Loss Select The Blank Question The rate of interest on rupee finance for exports is not exceeding ________ less than PLR. Correct Answer 0.025 Your Answer 50 basis points over LIBOR Select The Blank Question The creditworthiness of futures exchange is maintained through imposition of ________. Correct Answer Margin Your Answer Penalty

True/False Question The volume of trading of Derivatives in the world trade market is growing but still only 5% of the total. Correct Answer False Your Answer True True/False Question With the advent of futures and Options there is no need for additional instruments of risk hedging. Correct Answer False Your Answer True True/False Question Charter party bills of lading are generally not acceptable to bankers as security because Sea Charters are full of problems. Correct Answer True Your Answer True Multiple Choice Multiple Answer Question In which cases can a Resident Individual in India acquire foreign securities without prior approval? Correct Answer Gift from a person outside India, or Inheritance from a person resident in or outside India , Security issued under Cashless employees stock option scheme Your Answer Purchased from NRO/NRNR accounts , Purchase from borrowings from Authorised dealers Select The Blank Question Over the years ________has been the most widely accepted currency in international capital market. Correct Answer American Dollar Your Answer American Dollar True/False Question The American Dollar market comprises of the domestic segment of USD the offshore segment of USD. Correct Answer True Your Answer True

LIST OF ATTEMPTED QUESTIONS AND ANSWERS Select The Blank Question The counter party in option is ________. Correct Answer The exchange Your Answer The exchange Select The Blank Question An exchange rate for $ was48.2050, it is now 48.2075, then one says that it has moved ________ basis points Correct Answer 25 Your Answer 25 True/False Question Cross currency forward contract is not meaningful if original contract is in Rupees to USD Correct Answer True Your Answer True Multiple Choice Single Answer Question What do you understand by a swap? Correct Answer 2 counter parties agree to exchange streams of payments over a period of time Your Answer 2 counter parties agree to exchange streams of payments over a period of time True/False Question ECGC offers insurance protection to Indian exporters against payment risks. Correct Answer True Your Answer True Select The Blank Question Value of US Dollar is ________ in the world market but RBI is trying to match the Rupee value with it. Correct Answer Depreciating Your Answer Depreciating Multiple Choice Single Answer Question Financial management is complex process for Correct Answer Companies which have international trade Your Answer Companies which have international trade True/False Question The American Dollar market comprises of the domestic segment of USD the offshore segment of USD. Correct Answer True Your Answer True Multiple Choice Multiple Answer

Question What do you understand by 'Foreign Exchange Risk'? Correct Answer The change in value of currency alters the home currency value of an asset , liability, , The change in value of currency alters the home currency value of profit, loss, or future cash flows Your Answer The change in value of currency alters the home currency value of an asset , liability, , The change in value of currency alters the home currency value of profit, loss, or future cash flows Multiple Choice Single Answer Question Borrower from a developing country can borrow from US market Correct Answer Only if it has high credit rating and the country's Govt gives a guarantee for the same. Your Answer Only if it has high credit rating and the country's Govt gives a guarantee for the same. Multiple Choice Single Answer Question What was the purpose of reviewing FERA 1973 in the year 1993? Correct Answer Economic Liberalisation relating to Foreign trade and Foreign investments Your Answer Economic Liberalisation relating to Foreign trade and Foreign investments Select The Blank Question Legalised invoice is also called ________ invoice. Correct Answer Visaed Your Answer Visaed Multiple Choice Single Answer Question In Import financing through trust receipts, with whom does the ownership of goods remain? Correct Answer Bank releasing the import documents Your Answer Exporter Multiple Choice Single Answer Question Mr P has exported goods in May 2005 and is expecting payment of Euro 25000 by the last week of Sept 2005. Give details of the forward contract he can book :Correct Answer He can book a sale contract any time from May 2005 till Mid Sept 2005, for Euro 25000 or less, for maturity last week of Sept. Your Answer He can book a sale contract any time from May 2005 till Mid Sept 2005, for Euro 25000 or less, for maturity last week of Sept. True/False Question An Indian exporting goods to New York receives dollars in New York. Correct Answer True Your Answer True Multiple Choice Multiple Answer Question At the request of M/s Universal Traders Calcutta, State Bank of India opened a documentary credit in Favour of M/s Star Trading Co Dubai, for USD 25000/-.Here State Bank of India Calcutta is known as

Correct Answer Issuing Bank , Opening Bank Your Answer Issuing Bank , Opening Bank Multiple Choice Multiple Answer Question The integrated treasury branches of commercial banks handle which type of business? Correct Answer Forex transactions, Derivatives transactions , Mney market and bond market transactions Your Answer Forex transactions, Derivatives transactions , Mney market and bond market transactions Select The Blank Question For setting up joint ventures abroad or equity participation abroad, EXIM bank provides funds under ________ Scheme. Correct Answer Overseas investment facility Your Answer Overseas investment facility Match The Following Question Correct Answer Your Answer LastStage Form a single currency Form a single currency The First stage in the Europeand and Monetary Union (EMU) Free movement of capital in EC Free movement of capital in EC Stage II in EMU Establishment of European Monetary Institute Establishment of European Monetary Institute Stage III in EMU Members to meet the convergence criteria and fix the exchange rates Members to meet the convergence criteria and fix the exchange rates Multiple Choice Single Answer Question What do you call a 'decreasing principal swap'? Correct Answer Amortizing swap Your Answer Amortizing swap Multiple Choice Multiple Answer Question What does the word '"Tombstone" stand for? Correct Answer Advertisement which lists the managers, underwriters, , List of Providers of recently floated issue Your Answer Advertisement which lists the managers, underwriters, , List of Providers of recently floated issue Multiple Choice Single Answer Question Which of them are special funding facilities of IMF ? Correct Answer Buffer Stock Facility

Your Answer Buffer Stock Facility Multiple Choice Multiple Answer Question Mr A an Indian exporter is expecting to receive payment for exports in Hong kong dollar, after one month; How can he cover the exchange rate fluctuation risk? Correct Answer By booking a A forward contract in HKD itself , By booking a Cross currency option between USD and HKD and / or USD and Rs Your Answer By booking a A forward contract in HKD itself , By booking a Cross currency option between USD and HKD and / or USD and Rs True/False Question Bridge Loans are granted for construction of large size bridges, and flyovers. Correct Answer False Your Answer True True/False Question Integrated branches of Commercial banks handle forex transactions as well as options, swaps, futures, arbitrage operations Correct Answer True Your Answer True Match The Following Question

Correct Answer

Your Answer

Investment under Automatic route

Form ODA

Form ODA

Proposal under non-automatic route

RBI

RBI

Proposal under Normal route

Special Committee

Special Committee

Investment by partnerships not in core activity form ODI

form ODI

Multiple Choice Multiple Answer Question Name the services provided by 'Reuter 3000' in forex market:Correct Answer Quotation of forex rates, interest rates , Electronic broking by matching the rates Your Answer Quotation of forex rates, interest rates , Electronic broking by matching the rates Multiple Choice Single Answer Question Before abandoning the Fixed exchange rate system, the Dollar gold parity was fixed by USA at Correct Answer USD 35 per Ounce of gold Your Answer USD 35 per 10 grams of gold Select The Blank

Question After the Second World War many countries adopted the route of ________ to give facelift to sagging economies. Correct Answer Devaluation of their currencies Your Answer Devaluation of their currencies True/False Question Country risk analysis is a futile exercise in the background of cost of analysis. Correct Answer True Your Answer True Multiple Choice Multiple Answer Question Which factors affect the exchange rates in short time period? Correct Answer Demand and supply, interest rates, inflation, , Sentiment, Speculation Your Answer Demand and supply, interest rates, inflation, , Sentiment, Speculation , Literacy, Health of people Multiple Choice Single Answer Question A country needs external commercial borrowings to :Correct Answer Meet the adverse balance of payments position Your Answer Meet the adverse balance of payments position Multiple Choice Single Answer Question With whom can an exporter or importer book the forward contract? Correct Answer Authorised dealer/Authorised person Your Answer Authorised dealer/Authorised person Multiple Choice Multiple Answer Question World Bank gives loans for Correct Answer Structural Adjustments, Infrastructure projects, BOP problems Your Answer Structural Adjustments, Infrastructure projects, BOP problems Multiple Choice Multiple Answer Question Who can issue the Certificate of Inspection? Correct Answer Export Inspection Council, Person authorized by Importer. Your Answer Export Inspection Council, Person authorized by Importer. True/False Question In red clause credit some clauses are printed in Red ink. Correct Answer True Your Answer True True/False Question Out of 15 member countries in European Union 12 countries committed themselves for a single currency. Correct Answer False Your Answer False

Select The Blank Question FRNs are issued for a maturity period of ________ and are normally redeemed in one shot. Correct Answer 5-7 years Your Answer 5-7 years Multiple Choice Multiple Answer Question Criteria for investment abroad under automatic route :Correct Answer Activity for investment is core activity of Indian Company. , Total Financial commitment in JV/WOS as per prescribed ceiling Your Answer Activity for investment is core activity of Indian Company. , Total Financial commitment in JV/WOS as per prescribed ceiling Select The Blank Question ECGC provides cover to an Indian contractor for construction work abroad under ________. Correct Answer Construction works policy Your Answer Construction works policy Select The Blank Question A Credit can be a confirmed credit only if it is also ________. Correct Answer An Irrevocable credit Your Answer Redclause credit Multiple Choice Multiple Answer Question What is the criteria for an Indian Company for Direct Investment abroad? Correct Answer Activity proposed need to be core activity , Financial commitment in JV/WOS maximum $ 100 million or equivalent Your Answer Activity proposed need to be core activity , Financial commitment in JV/WOS maximum $ 100 million or equivalent , Indian co should not have been black listed Select The Blank Question A ________ Bill of lading does not specify the terms and conditions of carriage. Correct Answer Short Form Your Answer Copy of Multiple Choice Single Answer Question How does ECGC protect exporters against risks for export finance Correct Answer Issuing various insurance policies covering different risks connected with exports at nominal premium Your Answer Issuing various guarantees covering different risks connected with exports at nominal premium, True/False Question Sterling FRNs are issued in Europe, USA ,Asia

Correct Answer False Your Answer True Select The Blank Question Forward Margin makes a currency costlier then the currency is at ________. Correct Answer Premium Your Answer Premium Multiple Choice Multiple Answer Question Transaction exposures arises because of: Correct Answer The cost of settlement of future payment changes, Receipt denominated in a foreign currency varies due to exchange rate fluctuation Your Answer The cost of settlement of future payment changes, Receipt denominated in a foreign currency varies due to exchange rate fluctuation , Pricing of assets and liabilities in different countries

LIST OF ATTEMPTED QUESTIONS AND ANSWERS True/False Question

In a coupon swap the receiver is the one who receives fixed rate of interest. Correct Answer True Your Answer

True

Multiple Choice Multiple Answer Question The characteristics of an International documentary credit are :Correct Answer It is an undertaking given by the openers bank to pay certain sum of money , It is an undertaking given on behalf of a certain person to another person in different country Your Answer It is an undertaking given on behalf of a certain person to another person in different country , It is a stamped agreement and is not valid unless it is signed by the issuer , It is an undertaking given by the openers bank to pay certain sum of money True/False Question

Normally the forward contracts are booked for making huge profits. Correct Answer False Your Answer

True

Multiple Choice Single Answer Question What do you understand by a forex option? Correct Answer Right and not the obligation to take or give future delivery of an agreed quantity at certain price Your Answer Right and not the obligation to take or give future delivery of an agreed quantity at certain price Multiple Choice Single Answer Question In case the value date happens to be a holiday then the settlement is done :Correct Answer On the next working day Your Answer

On the next working day

True/False Question

Postal receipt is an acknowledgement of receipt of goods, a quasi negotiable instrument. Correct Answer False Your Answer

True/False

True

Question

Open Insurance policy is for any amount for specific period and specific number of shipments. Correct Answer False Your Answer

False

Multiple Choice Multiple Answer Question What schemes have been floated by RBI for facilitating Inter bank transactions? Correct Answer Clearing house for settlement and clearance for the operations of Ads , Set up a market intelligence cell to study and monitor developments in Forex Market Your Answer Clearing house for settlement and clearance for the operations of Ads , Set up a market intelligence cell to study and monitor developments in Forex Market Multiple Choice Single Answer Question Whose permission was necessary for forming the Association of Foreign Exchange Dealers, in India? Correct Answer Reserve Bank of India Your Answer

Reserve Bank of India

Multiple Choice Single Answer Question What was the collective assessment of International financial market about the country risk of many cnuntries in the last decade? Correct Answer Whatever it was, was disastrously wrong. Your Answer

Whatever it was, was disastrously wrong.

Select The Blank Question The international liquidity problem was tackled effectively by a reserve asset known as ________. Correct Answer Special drawing right Your Answer

Special drawing right

Select The Blank Question During the fixed rate exchange system prevailing in India, the intervention currency was ________. Correct Answer Sterling Pound Your Answer True/False Question

USDollar

Resident Individuals in India can acquire foreign bonus shares on the foreign securities already held by them without prior approval of RBI Correct Answer True

Your Answer

False

Multiple Choice Multiple Answer Question These are the parties to the documentary credit :Correct Answer Negotiating Bank, advising bank , Reimbursing bank, Confirming bank Your Answer Negotiating Bank, advising bank , Reimbursing bank, Confirming bank Multiple Choice Multiple Answer Question What are the functions of International Finance Corporation? Correct Answer Help private enterprise by financing projects , Provide technical assistance to private enterprise Your Answer Help private enterprise by financing projects , Provide technical assistance to private enterprise True/False Question

Both futures and options are traded on a public exchange.

Correct Answer False Your Answer

False

Multiple Choice Single Answer Question "Currency Snake" Correct Answer Plus or minus movement of exchange rate around the parity rate Your Answer

Plus or minus movement of exchange rate around the parity rate

Select The Blank Question "Flip-Flop', 'Mini-Max', 'Capped' 'Perpetual' are the various types of ________. Correct Answer Floating Rate Note (FRN) Your Answer

Floating Rate Note (FRN)

Multiple Choice Single Answer Question With whom can an exporter or importer book the forward contract? Correct Answer Authorised dealer/Authorised person Your Answer True/False Question

Authorised dealer/Authorised person

An Indian exporting goods to New York receives dollars in New York. Correct Answer True

Your Answer

False

Multiple Choice Multiple Answer Question Internationally only those currencies are in demand Correct Answer Which are convertible , Which have liquidity confirmed in Euro market operations Your Answer Which have liquidity confirmed in Euro market operations , Which are convertible Multiple Choice Single Answer Question What Preshipment facility is granted by EXIM Bank to an exporter? Correct Answer Preshipment credit beyond six months for construction and turnkey projects abroad Your Answer Preshipment credit beyond six months for construction and turnkey projects abroad True/False Question

The American Dollar market comprises of the domestic segment of USD the offshore segment of USD. Correct Answer True Your Answer

True

Multiple Choice Single Answer Question Who can open an EEFC account? Correct Answer Individuals , firms, companies, resident in India Your Answer

Individuals , firms, companies, resident in India

Select The Blank Question Over the years ________has been the most widely accepted currency in international capital market. Correct Answer American Dollar Your Answer

Euro Dollar

Multiple Choice Multiple Answer Question Under the liberalisation through FEMA, what can an AD offer to the clients, without the prior permission of RBI or Government? Correct Answer Hedges, interest rate swaps, currency swaps. , Caps and collars, and forward rate agreements Your Answer Hedges, interest rate swaps, currency swaps. , Caps and collars, and forward rate agreements True/False Question

Out of 15 member countries in European Union 12 countries commiteed themselves for a single currency. Correct Answer False

Your Answer

True

Select The Blank Question Global capital market can be understood from the study of ________. Correct Answer Yen,Euro Dollar, Swiss Francs, USD market, Govt Borrowings within and outside the country Your Answer IMF, World Bank and local markets Match The Following Question

Correct Answer

Your Answer

Investment by partnerships not in core activity

form ODI

form ODI

Investment under Automatic route

Form ODA

Form ODA

Proposal under non automatic route

RBI

Form NAR

Proposal under Normal route

Special Committee

OAB form

True/False Question

JV means Joint venture and WOS means World of Security.

Correct Answer False Your Answer

True

Select The Blank Question ________ is drawn by taking mean of the bid and offered rates. Correct Answer Middle rate Your Answer

Average rate

Match The Following Question

Correct Answer

Your Answer

IBRD

World Bank

World Bank

Collapse of IMF administered fixed exchange rates

Early 1970s

SDR

No International control on Exchange rates

Mid 80s

2002

Euro as a currency note/coin

2002

Mid 80s

Select The Blank Question An option on a future is ________. Correct Answer Derivative on a derivative Your Answer

Derivative on a derivative

Select The Blank Question Short term instruments normally issued by corporate borrowers, forhigh values, at a discount to face value are known as ________. Correct Answer Euro Commercial Papers Your Answer

FRNs

Multiple Choice Single Answer Question How does ECGC protect exporters against risks for export finance Correct Answer Issuing various insurance policies covering different risks connected with exports at nominal premium Your Answer Acting as agents for exporters and taking the entire risk of transit, recovery etc Multiple Choice Multiple Answer Question As regards the due date of an Option exercise which methods are in vogue? Correct Answer American option , European Option Your Answer

American option , European Option , Exercise date

Multiple Choice Multiple Answer Question Who can issue the Certificate of Inspection? Correct Answer Export Inspection Council , Person authorised by Importer. Your Answer

Export Inspection Council , Person authorised by Importer.

Multiple Choice Multiple Answer Question What were the primary objectives of the group of seven countries? Correct Answer Bring down the external value of US Dollar , Bring some stability to exchange rates of all the countries Your Answer Bring down the external value of US Dollar , Bring some stability to exchange rates of all the countries Multiple Choice Single Answer Question What is a bill of lading? Correct Answer Transport document representing movement of goods by water. Your Answer

Transport document representing movement of goods by water.

Multiple Choice Multiple Answer Question Why is Bill of Exchange a Quasi Negotiable Instrument? Correct Answer It represents title to goods , It is issued under Sale of Goods act Your Answer

It represents title to goods , It is issued under Sale of Goods act

Multiple Choice Single Answer Question Mr P has exported goods in May 2005 and is expecting payment of Euro 25000 by the last week of Sept 2005. Give details of the forward contract he can book :Correct Answer He can book a sale contract any time from May 2005 till Mid Sept 2005, for Euro 25000 or less, for maturity last week of Sept. Your Answer He can book a sale contract any time from May 2005 till Mid Sept 2005, for Euro 25000 or less, for maturity last week of Sept. Multiple Choice Single Answer Question Financial management is complex process for Correct Answer Companies which have international trade Your Answer

Companies which have international trade

Multiple Choice Single Answer Question What is the objective of INCOTERMS? Correct Answer Arrange for transfer of risk from seller to buyer at a convenient place Your Answer Arrange for transfer of risk from seller to buyer at a convenient place Multiple Choice Multiple Answer Question What do you understand by 'Foreign Exchange Risk'? Correct Answer The change in value of currency alters the home currency value of an asset , liability, , The change in value of currency alters the home currency value of profit, loss, or future cash flows Your Answer The change in value of currency alters the home currency value of an asset , liability, , The change in value of currency alters the home currency value of profit, loss, or future cash flows Multiple Choice Multiple Answer Question The various arms of World Bank are: Correct Answer International Development Agency , Multinational Investment Guarantee Agency Your Answer International Development Agency , Multinational Investment Guarantee Agency , Asian Development Bank Multiple Choice Multiple Answer Question Which of these are considered as miscellaneous documents?

Correct Answer Packing list , Weight certificate Your Answer

Packing list , Weight certificate

Select The Blank Question ECGC provides cover to an Indian contractor for construction work abroad under ________. Correct Answer Construction works policy Your Answer

Construction works policy

International Finance_5 LIST OF ATTEMPTED QUESTIONS AND ANSWERS Multiple Choice Single Answer Question Who issues guidelines for Direct Investments abroad by any Indian? Correct Answer Exchange Control Department of RBI Your Answer Exchange Control Department of RBI Multiple Choice Multiple Answer Question What does a speculator accept while dealing with an option? Correct Answer The exercise or strike price, the amount , The expiration date and time, the method of payment Your Answer The exercise or strike price, the amount , The expiration date and time, the method of payment Match The Following Question Correct Answer Your Answer Returning NRI RFC INVESTMENT IN MMMF NRI Deposits FCNR, NRE Investment in GDR, ADR NRIS AND OCBS INVESTMENT IN MMMF FCA EXPORTERS EEFC FCNR, NRE True/False Question The Negotiating bank examines the documents submitted by the seller and makes payment if they comply the terms and conditions of the LC. Correct Answer True Your Answer True True/False Question Cash flows generated from aforeign project may replace revenue-producing imports to the host country Correct Answer False Your Answer True Select The Blank Question Global capital market can be understood from the study of ________. Page 1

International Finance_5 Correct Answer Yen,Euro Dollar, Swiss Francs, USD market, Govt Borrowings within and outside the country Your Answer Yen,Euro Dollar, Swiss Francs, USD market, Govt Borrowings within and outside the country Select The Blank Question Under the ________ standard the Country's Central Bank promised to buy back the currency at certain rate of Gold. Correct Answer Gold Your Answer Gold Multiple Choice Single Answer Question What is the meaning of right to buy or sell the future contract? Correct Answer An Option Your Answer An Option Multiple Choice Single Answer Question Financial management is complex process for Correct Answer Companies which have international trade Your Answer Companies which have international trade True/False Question Bid rate is bidders buying rate and offered rate is bidders selling rate. Correct Answer True Your Answer True Multiple Choice Single Answer Question A Dealer Buys commodity in Bombay and sells it in Delhi due to price differentials. He is said to be resorting to which practice? Correct Answer Arbitrage Your Answer Cash market True/False Question Head office of ICC is in Paris because of its First President Eliance Clementel. Correct Answer True Your Answer True Multiple Choice Single Answer Question Under which category of exchange rate did the Indian Rupee came from June 92? Page 2

International Finance_5 Correct Answer Other managed Floating Your Answer Pegged to Single currency Select The Blank Question A Credit can be a confirmed credit only if it is also ________. Correct Answer An Irrevocable credit Your Answer Anticipatory Credit Select The Blank Question ________ is the price at which the market maker is prepared to borrow the money. Correct Answer Bid rate Your Answer Selling price True/False Question Sandby credis are introduced by US Banks because they were not permitted to issue on demand guarantees. Correct Answer True Your Answer True Multiple Choice Single Answer Question How does ECGC protect exporters against risks for export finance Correct Answer Issuing various insurance policies covering different risks connected with exports at nominal premium Your Answer Issuing various insurance policies covering different risks connected with exports at nominal premium Multiple Choice Single Answer Question What are the functions of Export Import Bank of India? Correct Answer Finance functions and development functions Your Answer Commercial Banking, development Banking functions Multiple Choice Multiple Answer Question What were the primary objectives of the group of seven countries? Correct Answer Bring down the external value of US Dollar , Bring some stability to exchange rates of all the countries Your Answer Bring down the external value of US Dollar , Bring some stability to exchange rates of all the countries Select The Blank Question ________ is the agency of ICC which deals with the maritime Page 3

International Finance_5 crime. Correct Answer International Maritime Bureau Your Answer International Maritime Bureau Multiple Choice Single Answer Question Samuraibonds are Correct Answer Foreign yen bonds issued to non resident entities in Japanese Market Your Answer Yen bonds issued in USA Multiple Choice Single Answer Question Name the members of Exchange Banks association :Correct Answer Foreign Banks at Bombay, Calcutta, Delhi, Madras, Amritsar, Your Answer Foreign Banks at Bombay, Calcutta, Delhi, Madras, Amritsar, Multiple Choice Multiple Answer Question Export finance Correct Answer Preshipment credit , Post shipment credit , Duty Draw back Your Answer Post shipment credit , Duty Draw back Multiple Choice Single Answer Question How many Countries participated in the third survey on Foreign Exchange, coordinated by BIS in 1992? Correct Answer 26 Your Answer 25 Select The Blank Question FRNs are issued for a maturity period of ________ and are normally redeemed in one shot. Correct Answer 5-7 years Your Answer 1 to 3 years Multiple Choice Single Answer Question What was the official foreign currency reserve of the group of seven central Banks taken togeather in mid 1992? Correct Answer USD 275 Billion Your Answer USD 275 Billion Multiple Choice Single Answer Page 4

International Finance_5 Question What is the volume of the world's total speculative transactions? Correct Answer Roughly 90 % of the total turnover Your Answer 70% of the total turnover Multiple Choice Single Answer Question Which Indian Co. is not eligible to make investment abroad under automatic route? Correct Answer Co which is caution listed by RBI Your Answer Co. which may not have proposed activity as core activity Multiple Choice Multiple Answer Question The maximum period for which a forward contract can be granted by the AD depends upon which RBI directives? Correct Answer Maximum Period of repatriation of export proceeds , Maximum Period of payment for Imports Your Answer Maximum Period of payment for Imports , The need of the importer Multiple Choice Multiple Answer Question As regards the due date of an Option exercise which methods are in vogue? Correct Answer American option , European Option Your Answer American option , European Option , Exercise date Multiple Choice Single Answer Question Which is the most actively traded currency in the international market today? Correct Answer US Dollar Your Answer US Dollar Multiple Choice Multiple Answer Question Mr A sells an option conferring the right to buy a kilogram of silver at Rs 8000/- for next six months means :Correct Answer The buyer of option can buy silver from Mr A at 8000/- per Kg, in next 6 months, even if the price goes up, , The buyer of option can sell silver to Mr A at 8000/- per Kg, in next 6 months, even if the price goes down. Your Answer The buyer of option can buy silver from Mr A at 8000/- per Kg, in next 6 months, even if the price goes up, , The buyer of option can sell silver to Mr A at 8000/- per Kg, in next 6 months, even if the price goes down. Page 5

International Finance_5 True/False Question Funding Options in domestic yen market are Futures, FRAs, Call and put options. Correct Answer True Your Answer True Multiple Choice Single Answer Question Whose permission was necessary for forming the Association of Foreign Exchange Dealers, in India? Correct Answer Reserve Bank of India Your Answer Reserve Bank of India Match The Following Question Correct Answer Your Answer 16 principles for good environmental conduct Business Charter of sustainable development Business Charter of sustainable development World Chambers Congress World Federation of Chambers World Federation of Chambers ICC Conference in 2004 Marrakesh Marrakesh 13 types of trade contracts INCOTERM Commercial crime Bureau Select The Blank Question Hotels and Airline companies are allowed to receive payments in Indian rupees from foreign tourist without insisiting on ________. Correct Answer Encashment certificate Your Answer Foreign currency Multiple Choice Single Answer Question Fixed, Floating or Limited flexibility are used in the context of Correct Answer Exchange Rates Your Answer Exchange Rates Multiple Choice Multiple Answer Question Internationally only those currencies are in demand Correct Answer Which are convertible , Which have liquidity confirmed in Euro market operations Your Answer Which are convertible , Which have greater intrinsic value Page 6

International Finance_5 Select The Blank Question Forward Margin makes a currency costlier then the currency is at ________. Correct Answer Premium Your Answer Loss Select The Blank Question The rate of interest on rupee finance for exports is not exceeding ________ less than PLR. Correct Answer 0.025 Your Answer 50 basis points over LIBOR Select The Blank Question The creditworthiness of futures exchange is maintained through imposition of ________. Correct Answer Margin Your Answer Penalty True/False Question The volume of trading of Derivatives in the world trade market is growing but still only 5% of the total. Correct Answer False Your Answer True True/False Question With the advent of futures and Options there is no need for additional instruments of risk hedging. Correct Answer False Your Answer True True/False Question Charter party bills of lading are generally not acceptable to bankers as security because Sea Charters are full of problems. Correct Answer True Your Answer True Multiple Choice Multiple Answer Question In which cases can a Resident Individual in India acquire foreign securities without prior approval? Correct Answer Gift from a person outside India, or Inheritance from a person resident in or outside India , Security issued under Cashless employees Page 7

International Finance_5 stock option scheme Your Answer Purchased from NRO/NRNR accounts , Purchase from borrowings from Authorised dealers Select The Blank Question Over the years ________has been the most widely accepted currency in international capital market. Correct Answer American Dollar Your Answer American Dollar True/False Question The American Dollar market comprises of the domestic segment of USD the offshore segment of USD. Correct Answer True Your Answer True

Page 8

Ât cellSpacing=0 cellPadding=0 width="100%" border=0>  Select The Blank   Question ÂThe stock of foreign reserves in India, is now around ________ % of the GDP and exceeds money circulation.

  Correct Answer 15    Your Answer 20 Â

  Multiple Choice Multiple Answer   Question ÂThe World Bank Group Consists of

  Correct Answer International  Development Association , International finance Corporation , Multinational Investment Guarantee Agency

  Your Answer International  finance Corporation , Multinational Insurance Guarantee Agency , International Development Association

  Multiple Choice Single Answer   Question ÂWhat do you call a 'decreasing principal swap'?   Correct Answer Amortizing  swap   Your Answer Amortizing  swap   Multiple Choice Single Answer   Question ÂWho can open an EEFC account?

  Correct Answer Individuals  , firms, companies, resident in India

  Your Answer Individuals  , firms, companies, resident in India   Select The Blank   Question ÂEUROCLEAR, CEDEL are the names of ________ related with the issue of GDR.

  Correct Answer Registrars    Your Answer Registrars    True/False   Question ÂAn Indian importing goods from USA pays dollars in India.   Correct Answer False    Your Answer True    Multiple Choice Multiple Answer   Question ÂHow does a company use swap to restructure debt, in its balance sheet?   Correct Answer By resorting  to floating rate from fixed rate , By resorting to fixed rate from floating rate

  Your Answer By resorting  to floating rate from fixed rate , By resorting to fixed rate from floating rate

  Multiple Choice Single Answer   Question ÂHow many Countries participated in the third survey on Foreign Exchange, coordinated by BIS in 1992?

  Correct Answer 26    Your Answer 26    True/False   Question ÂIn red clause credit some clauses are printed in Red ink.   Correct Answer True Â

  Your Answer False Â

  Select The Blank   Question ÂThe rate of interest on rupee finance for exports is not exceeding ________ less than PLR.

  Correct Answer 0.025    Your Answer 50 basis  points over LIBOR   Multiple Choice Multiple Answer   Question ÂFor issuing a Documentary credit The applicant is required to provide to the issuing bank details about

  Correct Answer The purchase-sales  contract, the mode of payment, the tenor of payment , The documents required, the names of authorities who would issue the documents

  Your Answer TheÂpurchase-sales contract, the mode of payment, the tenor of payment , The track record of the buyer and seller

  Multiple Choice Single Answer   Question ÂExchange Rate means

  Correct Answer Value  of one currency in terms of another   Your Answer Value  of one currency in terms of another   Multiple Choice Single Answer

  Question ÂWhat do you understand by 'bill of lading in a quasi negotiable instrument'?

  Correct Answer It is a document  of title to goods and its negotiation may not be complete and free from qualification.

  Your Answer It isÂa document of title to goods and its negotiation may not be complete and free from qualification.

  Multiple Choice Single Answer   Question ÂWhere does the word derivative originate from?   Correct Answer Mathematics    Your Answer Mathematics    Multiple Choice Multiple Answer   Question ÂWhat options does the merchant have while booking a forward contract?   Correct Answer Delivery  on a Fixed date, within six months , Delivery on a date between two days separated by a period of maximum one month

  Your Answer Delivery  on a Fixed date, within six months , Delivery on a date between two days separated by a period of maximum one month , A day within next 1 year

  Select The Blank   Question ÂEEFC account holder is allowed to have ________ facility.

  Correct Answer Cheque  book and nomination

  Your Answer Cheque  book and nomination   True/False   Question ÂA small percentage of futures contracts actually result in delivery of the underlying commodity or security.

  Correct Answer True    Your Answer True    True/False   Question ÂLong in futures means a commitment to sell futures at a future date.

  Correct Answer False    Your Answer True    Multiple Choice Multiple Answer   Question ÂWhat are the categories of exposures, a person faces in International trade?

  Correct Answer Transaction  exposure , Translation exposure   Your Answer Transaction  exposure , Translation exposure   True/False

  Question ÂNaked position is when investor does not hedge against market risk.

  Correct Answer True    Your Answer True    Multiple Choice Multiple Answer   Question ÂThe participants in the Forex Markets are :-

  Correct Answer Non banking  entities wishing to exchange currencies, , Banks and their clients , Speculators, arbitragers

  Your Answer NonÂbanking entities wishing to exchange currencies, , Banks and their clients , Speculators, arbitragers

  Select The Blank   Question Â________ is the acid test for events to be included in country risk.

  Correct Answer Control  by Government'   Your Answer Predictability Â

  Multiple Choice Multiple Answer   Question ÂCriteria for Direct investment abroad under normal route :  Correct Answer Viability  of the proposal, and excellent track record of business , Experience and expertise of promoters, benefit to the country

  Your Answer Viability  of the proposal, and excellent track record of business , Experience and expertise of promoters, benefit to the country

Â

 Match The Following Question

Correct Answer

Your Answer

16 principles for good environmental conduct World Chambers Congress ICC Conference in 2004 13 types of trade contracts

Business Charter of sustainable development World Federation of Chambers Marrakesh

Business Charter of sustainable development

INCOTERM

INCOTERM

World Federation of Chambers Marrakesh

  True/False   Question ÂPP Form is required when exports are made to any country by post parcel on value payable basis.

  Correct Answer False    Your Answer True Â

  Multiple Choice Multiple Answer   Question ÂThe investor is in the money when :-

  Correct Answer The exercise  price is below the spot price for a call option , The exercise price is above the spot price for a put option

  Your Answer TheÂexercise price is above the spot price for a put option , The exercise price is below the spot price for a put option

  Multiple Choice Single Answer   Question ÂUnder which category of exchange rate did the Indian Rupee came from June 92?

  Correct Answer Other  managed Floating

  Your Answer Independently  floating

  Multiple Choice Single Answer   Question ÂFinancial management is complex process for   Correct Answer Companies  which have international trade   Your Answer Cooperative  societies abroad   Select The Blank   Question ÂThe Trans European Automated Real Time Gross Settlement Express Transfer is popular in ________.

  Correct Answer Brussels    Your Answer Frankfurt    Multiple Choice Multiple Answer   Question ÂWhat is the duty of the AD while booking a forward contract?   Correct Answer Verify  the suitable documentary evidence, and that the customer is exposed to risk , Ensure that a genuine transaction exists

  Your Answer Verify  the suitable documentary evidence, and that the customer is exposed to risk , Ensure that a genuine transaction exists , Protect the customer fully from exchange risk

  Select The Blank

  Question ÂEven after liberalisation of exchange controls, the Euro sterling market remained volatile because of uncertainties in UK and gyrations of sterling against major ________ countries

  Correct Answer OECD   Your Answer OECD    Multiple Choice Single Answer   Question ÂWith whom can an exporter or importer book the forward contract?

  Correct Answer Authorised  dealer/Authorised person

  Your Answer AnyÂbank in India   Select The Blank

  Question ÂTo make the export more competitive and remunerative, the Indian Govt extends ________ facility.   Correct Answer Cash Incentives    Your Answer LineÂof credit   Multiple Choice Single Answer   Question ÂPunjab National Bank Mumbai Branch quoted USD 1= Rs 50.5000/52.5050. Which is the bid rate for USD?

  Correct Answer USD 1=Rs  50.5000   Your Answer USD  1=Rs 50.5000 + margin   True/False

  Question ÂThe Euro Commercial paper is supported by a team-syndicated credit with both sides of the transaction combined into one operation.

  Correct Answer True    Your Answer True    Multiple Choice Multiple Answer   Question ÂAs regards the due date of an Option exercise which methods are in vogue?

  Correct Answer American  option , European Option   Your Answer American  option , European Option   Select The Blank   Question ÂIn mid 1992 ________ rejected the ratification of Maastricht treaty

  Correct Answer Denmark    Your Answer UK    Multiple Choice Single Answer   Question ÂHow does ECGC protect exporters against risks for export finance

  Correct Answer Issuing various insurance policies covering different risks connected with exports at nominal premium

  Your Answer Issuing  various guarantees covering different risks connected with exports at nominal premium,

  Match The Following Question Correct Answer

Your Answer

Swap price

Difference in Difference in prices of prices of two two legs of buying and legs of selling buying and selling Euro Market Market in Market in Euro Euro currencies currencies Making market Quoting rate Quoting rate for both for both buying and selling a buying and currency selling a currency Spread Price Price difference difference between bid and offer between bid and offer   True/False   Question ÂCountry risk analysis is a futile exercise in the background of cost of analysis.   Correct Answer True    Your Answer False    Multiple Choice Single Answer   Question ÂWhat is the volume of the world's average daily transactions in foreign exchange?   Correct Answer USD 1Âtrillion   Your Answer USD  1 trillion   Multiple Choice Multiple Answer   Question ÂIn DAF, the frontier means :  Correct Answer Frontier  of exporter's country , Frontier of importer's country   Your Answer Frontier  of exporter's country , Frontier of importer's country   True/False   Question ÂIn an interest rate swap if the payment schedules are identical, only diffrence between the two payments is delivered.

  Correct Answer True    Your Answer True    Select The Blank

  Question ÂThe option which can be exercised on the maturity date and not before it is known as________.

  Correct Answer European  Option   Your Answer European  Option   True/False

  Question ÂSandby credis are introduced by US Banks because they were not permitted to issue on demand guarantees.

  Correct Answer True    Your Answer True    Multiple Choice Multiple Answer   Question ÂAt the request of M/s Universal Traders Calcutta, State Bank of India opened a documentary credit in Favour of M/s Star Trading Co Dubai, for USD 25000/-.Here State Bank of India Calcutta is known as

  Correct Answer Issuing Bank , Opening Bank   Your Answer Issuing  Bank , Confirming Bank

  Select The Blank   Question ÂUnder the ________ standard the Country's Central Bank promised to buy back the currency at certain rate of Gold.

  Correct Answer Gold    Your Gold Answer  Multiple Choice Multiple Answer Âoice Multiple Answer   Question ÂThe World Bank Group Consists of

Â

  Correct Answer International  Development Association , International finance Corporation , Multinational Investment Guarantee Agency   Your Answer International  Development Association , International finance Corporation , Multinational Investment Guarantee Agency   Multiple Choice Single Answer   Question ÂEquity can be raised internationallly by a company through :  Correct Answer Fund raising,  underwriting, hedging , arbitration operations   Your Answer Floating  debt instruments Â

 Match The Following

Question Swap price

Euro Market

Correct Difference in prices of two legs of buying and selling Market in Euro currencies

Making market Quoting rate for both buying and selling a currency Spread Price difference between bid and offer

Your Answer Difference in prices of two legs of buying and selling

Market in Euro currencies Quoting rate for both buying and selling a currency

Price difference between bid and offer

  True/False   Question ÂIn an interest rate swap if the payment schedules are identical, only diffrence between the two payments is delivered.   Correct Answer True    Your Answer True Â

  Select The Blank   Question ÂThe large flow of money internationally was caused primarily by the oil price hikes engineered by ________.   Correct Answer OPECÂcountries   Your Answer European  Countries   Select The Blank   Question Â________ enables bankers to extend purchase, negotiation or discount of export bills.   Correct Answer Post shipment  credit guarantee   Your Answer Export  finance guarantee

  Multiple Choice Multiple Answer   Question ÂWhich non financial service does EXIM Bank give to an exporter?   Correct Answer Provide  Technical and administrtive support , Planning, promoting and developing export oriented Industries   Your Answer Provide  Technical and administrtive support , Planning, promoting and developing export oriented Industries   Multiple Choice Multiple Answer   Question ÂWhich debits are permitted to EEFC accounts?   Correct Answer Payment  of hotel bills abroad, payment for expenses on Heart operation abroad , Payment to EOUs in India for services rendered by them   Your Answer Payment  of hotel bills abroad, payment for expenses on Heart operation abroad , Payment to EOUs in India for services rendered by them   True/False   Question ÂTerms in an Irrevocable credit can not be alterred at all.   Correct Answer False    Your Answer True    Multiple Choice Single Answer   Question ÂHow many INCOTERMS are there in "Departure" Group?

  Correct Answer One Â

  Your Answer OneÂ

  Multiple Choice Single Answer   Question ÂHow does ICC access Govt and influence Governmental decisions?   Correct Answer Through  national committees   Your Answer Through  national committees   Multiple Choice Multiple Answer   Question ÂA Usance bill of exchange may be drawn in 2 ways. Which are they?   Correct Answer Documents  against payment , Documents against acceptance   Your Answer Documents  against payment , Documents against acceptance   Multiple Choice Single Answer   Question ÂName the single most important factor which may devastate the economic viability of an operation in international trade :  Correct Answer Unfavourable  fluctuation in the exchange rate   Your Answer Unfavourable  fluctuation in the exchange rate   True/False   Question ÂThe Euro Commercial paper is supported by a team-syndicated credit with both sides of the transaction combined into one operation.   Correct Answer True    Your Answer True    Select The Blank   Question ÂWhen interest rate changes by 0.05 per cent then it has changed ________ basis points.   Correct Answer 50 Â

  Your Answer 50 Â

  Select The Blank

  Question ÂIndia is a founder member of ________.   Correct Answer IMF    Your Answer BANKNET    Multiple Choice Single Answer   Question ÂHow many types of documents are required in the International trade?   Correct Answer Six    Your Answer Six    Multiple Choice Multiple Answer   Question ÂExport finance   Correct Answer Preshipment  credit , Post shipment credit , Duty Draw back   Your Answer Preshipment  credit , Post shipment credit , Buyers credit   Select The Blank   Question ÂWhen a credit is made to available to additional beneficiary wholly or partly it is called as ________.   Correct Answer Transferable  credit   Your Answer Transferable  credit   True/False   Question ÂA small percentage of futures contracts actually result in delivery of the underlying commodity or security.   Correct Answer True    Your Answer True    Select The Blank   Question ÂAn exchange rate for $ was48.2050, it is now 48.2075, then one says that it has moved ________ basis points   Correct Answer 25    Your Answer 0.0025    True/False   Question ÂTransfer of funds for investment abroad is allowed freely.   Correct Answer True Â

  Your Answer True    Select The Blank

  Question ÂThe rate of interest on rupee finance for exports is not exceeding ________ less than PLR.   Correct Answer 0.025    Your Answer 0.025    Multiple Choice Multiple Answer   Question ÂWhat do you understand by 'Foreign Exchange Risk'?   Correct Answer The change  in value of currency alters the home currency value of an asset , liability, , The change in value of currency alters the home currency value of profit, loss, or future cash flows   Your Answer Certainty  of an event which would happen in favour of the importer/ exporter , Inability to control various factors contributing to International trade   Multiple Choice Single Answer   Question ÂFinancial management is complex process for   Correct Answer Companies  which have international trade   Your Answer Companies  which have international trade   True/False   Question ÂImport finance is nomally available at rates of interest cheaper than domestic finance rates.   Correct Answer False    Your Answer False    Select The Blank   Question ÂThe stock of foreign reserves in India, is now around ________ % of the GDP and exceeds money circulation.   Correct Answer 15    Your Answer 12.5   Multiple Choice Single Answer   Question ÂWhich type of Convertibility of Rupee launched in March 1992 ?   Correct Answer Limited  convertibility   Your Answer Convertibility  on current account

  Multiple Choice Multiple Answer

  Question ÂHow does a company use swap to restructure debt, in its balance sheet?   Correct Answer By resorting  to floating rate from fixed rate , By resorting to fixed rate from floating rate   Your Answer By resorting  to fixed rate from floating rate , Prepayment of existing debts , Raising new loans Â

 Multiple Choice Single Answer   Question ÂPunjab National Bank Mumbai Branch quoted USD 1= Rs 50.5000/52.5050. Which is the offered rate for Rs?   Correct Answer USD 1=Rs  50.5000   Your Answer USD  1=Rs 50.5000 + margin   Multiple Choice Multiple Answer

  Question ÂWhat facilities are proposed by Sodhani Committee for NRIs?   Correct Answer Housing  Loan to NRI staff of Indian Companies, , NRIs be permitted to invest in MMMF   Your Answer Housing  Loan to NRI staff of Indian Companies, , NRIs be permitted to invest in MMMF , Permission to sale shares under Port folio investment be denied   Multiple Choice Multiple Answer   Question ÂWhich document is used for transportation by air, rail, road, marine?   Correct Answer Multimodal  transport document , Combined Transport document   Your Answer Multimodal  transport document , Combined Transport document Â

 Multiple Choice Single Answer   Question ÂUnder which contracts the seller pays the import duty for the buyer?

  Correct Answer DDP    Your Answer DDU    True/False   Question ÂDealing room is a place where the actual exchange of currencies takes place phsically at predetermined rates.   Correct Answer False    Your Answer True Â

  Multiple Choice Single Answer   Question ÂWhat was the reason for launching swap as an risk hedging instrument?   Correct Answer To avoid  the effects of currency control imposed in UK   Your Answer To avoid  the effects of currency control imposed in UK   Select The Blank   Question ÂThe creditworthiness of futures exchange is maintained through imposition of ________.   Correct Answer Margin   Your Answer Margin    True/False   Question ÂIn forward sales, the delivery refers to actual cash flows on due date.   Correct Answer True    Your Answer True    Multiple Choice Multiple Answer   Question ÂIn a documentary credit the word document means :  Correct Answer Specified  documents of title, transportation, insurance , Specified certificates, and other papers required as per the regulations in the buyers country

  Your Answer Specified  documents of title, transportation, insurance , Specified certificates, and other papers required as per the regulations in the buyers country , Credit is documented as an evidence   True/False   Question ÂThe amount in EEFC account can be deposited in FCNR account to get interest.   Correct Answer False    Your Answer True    True/False   Question ÂADR Level I, is the final step to enter US erquity market.   Correct Answer False    Your Answer False    Multiple Choice Multiple Answer   Question ÂAs per BIS, recovery from the recent crisis in global capital market was mainly due to   Correct Answer Rapid Âand appropriate policy response in US and other countries , Increase in Foreign Direct Investments , free flow of credit   Your Answer Rapid  and appropriate policy response in US and other countries , Increase in Foreign Direct Investments , free flow of credit , Devaluation of currencies   Multiple Choice Multiple Answer   Question ÂWhat is the effect of cross currency option on the customer?   Correct Answer He gets  the flexibility to exposure , He is exposed to additional risk of movement of rates between 3 currencies   Your Answer He  gets the flexibility to exposure , His profit increases   Multiple Choice Single Answer   Question ÂIn case the value date happens to be a holiday then the settlement is done :  Correct Answer On theÂnext working day

  Your Answer On  the next working day   Select The Blank   Question ÂAfter the second world war many countries adopted the route of ________ to give facelift to sagging economies.   Correct Answer Devaluation  of their currencies   Your Answer Devaluation  of their currencies   Select The Blank   Question Â________Bill of lading must specify the name of the ship.   Correct Answer "On Board"    Your Answer "OnÂBoard"   Multiple Choice Single Answer   Question ÂWhose permission was necessary for forming the Association of Foreign Exchange Dealers, in India?   Correct Answer Reserve  Bank of India   Your Answer Reserve  Bank of India   Match The Following Question Correct Your Answer Answer European On or after On or after maturity option maturity Out of money Loss Loss In the money Profit Profit American Before Before maturity option maturity

Â

Multiple Choice Multiple Answer Âoice Multiple Answer   Question The ÂWorld Bank Group Consists of

  CorrectInternational Answer  Development   Your Answer International  Development Association , International finance Corporation , Multinational Investment Guarantee Agency   Multiple Choice Single Answer   Question Equity  can be raised internationallly by a company through :  CorrectFund Answer raising,  underwriting, hedging , arbitration operations

  Your Answer FloatingÂdebt instruments

  Multiple Choice Single Answer   Question Financial  management is complex process for   CorrectCompanies Answer Âwhich have international trade   Your Answer Companies  which have international trade   True/False   Question Import  finance is nomally available at rates of interest cheaper than domestic finance rates.   CorrectFalse Answer Â

  Your Answer False Â

  Select The Blank   Question The Âstock of foreign reserves in India, is now around ________ % of the GDP and exceeds money circulation.   Correct15 Answer    Your Answer 12.5    Multiple Choice Single Answer   Question Which  type of Convertibility of Rupee launched in March 1992 ?   CorrectLimited Answer convertibility    Your Answer Convertibility  on current account   Multiple Choice Multiple Answer   Question HowÂdoes a company use swap to restructure debt, in its balance sheet?   CorrectBy Answer resorting  to floating rate from fixed rate , By resorting to fixed rate from floating rate

  Your Answer By resorting  to fixed rate from floating rate , Prepayment of existing debts , Raising new loans

  Multiple Choice Single Answer   Question Punjab  National Bank Mumbai Branch quoted USD 1= Rs 50.5000/52.5050. Which is the offered rate for Rs?

  CorrectUSD Answer 1=RsÂ50.5000   Your Answer USD 1=Rs  50.5000 + margin   Multiple Choice Multiple Answer   Question What  facilities are proposed by Sodhani Committee for NRIs?   CorrectHousing AnswerÂLoan  to NRI staff of Indian Companies, , NRIs be permitted to invest in MMMF

  Your Answer HousingÂLoan to NRI staff of Indian Companies, , NRIs be permitted to invest in MMMF , Permission to sale shares under Port folio investment be denied   Multiple Choice Multiple Answer   Question Which  document is used for transportation by air, rail, road, marine?

  CorrectMultimodal Answer Âtransport document , Combined Transport document   Your Answer Multimodal  transport document , Combined Transport document   Multiple Choice Single Answer   Question Under  which contracts the seller pays the import duty for the buyer?

  CorrectDDP Answer Â

  Your Answer DDU Â

  True/False   Question Dealing  room is a place where the actual exchange of currencies takes place phsically at predetermined rates.   CorrectFalse Answer Â

  Your Answer True Â

  Multiple Choice Single Answer

  Question What  was the reason for launching swap as an risk hedging instrument?

  CorrectTo Answer avoid the  effects of currency control imposed in UK

  Your Answer To avoidÂthe effects of currency control imposed in UK

  Select The Blank   Question The Âcreditworthiness of futures exchange is maintained through imposition of ________.   CorrectMargin Answer    Your Answer Margin    True/False   Question In forward  sales, the delivery refers to actual cash flows on due date.   CorrectTrue Answer Â

  Your Answer True Â

  Multiple Choice Multiple Answer   Question In a  documentary credit the word document means :-

  CorrectSpecified AnswerÂdocuments  of title, transportation, insurance , Specified certificates, and other papers required as per the regulations in the buyers country

  Your Specified documents of title, Answer transportation, insurance ,  Multiple Choice Single Answer ÂChoice Single Answer   Question In which  of the funding facilities the repayment period is between 3 to 5 years ?

  CorrectC.C.F.F. Answer    Your Answer Stand By  Arrangement   Multiple Choice Single Answer   Question Samuraibonds  are

  CorrectForeign Answer yen  bonds issued to non resident entities in Japanese Market   Your Answer Foreign  yen bonds issued to non resident entities in Japanese Market   Match The Following Question Correct Answer Your Answer

Returning RFC NRI

FCNR, NRE

NRI Deposits

INVESTM ENT IN MMMF RFC Investmen t in GDR, ADR

FCNR, NRE

NRIS AND INVESTMENT IN MMMF EXPORT EEFC ERS

 Ât cellSpacing=0 cellPadding=0 width="100%" border=0>

 Multiple Choice Single Answer   Question In which  of the funding facilities the repayment period is between 3 to 5 years ?   CorrectC.C.F.F. Answer    Your Answer Stand By  Arrangement Â

 Multiple Choice Single Answer

  Question Samuraibonds  are

  CorrectForeign Answer yen  bonds issued to non resident entities in Japanese Market   Your Answer Foreign  yen bonds issued to non resident entities in Japanese Market Â

 Multiple Choice Single Answer   Question In the  context of International economy ,the period between 1920s and 1930s is known as   CorrectGreat Answer Depression    Your Answer Great Depression  Â

 Multiple Choice Multiple Answer

  Question Sterling  Market is attractive for an international investor because

  CorrectGilts Answer offer higher  attractive margin , Choice of issuing medium term and long term bonds   Your Answer Gilts offer  higher attractive margin , Choice of issuing medium term and long term bonds , Linking of issues to currency swaps Â

 Select The Blank

  Question When  interest rate changes by 0.05 per cent then it has changed ________ basis points.   Correct50 Answer Â

  Your Answer 500 Â

  Multiple Choice Multiple Answer   Question Internationally  only those currencies are in demand

  CorrectWhich Answer are  convertible , Which have liquidity confirmed in Euro market operations   Your Answer Which are  convertible , Which have liquidity confirmed in Euro market operations , Which have greater intrinsic value   Select The Blank   Question AfterÂthe second world war many countries adopted the route of ________ to give facelift to sagging economies.   CorrectDevaluation Answer  of their currencies

  Your Answer Devaluation  of their currencies

  Multiple Choice Multiple Answer   Question HowÂdoes RBI control Exports?

  CorrectThrough AnswerÂRule  like "Full value of Export bills must be realised in 180 days," , Through Rule like " Export must be denominated In a currency of the Importing country or a freely convertible currency"

  Your Answer ThroughÂRule like "Full value of Export bills must be realised in 180 days," , Through Rule like " Export must be denominated In a currency of the Importing country or a freely convertible currency" , By Issuing a Export Code Number, and monitoring his exports through returns and statements filed by him   Select The Blank   Question OverÂthe years ________has been the most widely accepted currency in international capital market.   CorrectAmerican Answer Dollar Â

  Your Answer American  Dollar   Select The Blank   Question Any  company involved in foreign exchange transactions is exposed to ________ risk.

  CorrectCurrency Answer    Your Answer Payment   True/False   Question An AD  can book a forward contract for a part of the amount of Sale Contract   CorrectTrue Answer Â

  Your Answer True Â

  Multiple Choice Single Answer

  Question WhyÂdoes a person need protection from unfavourable movement of exchange rate?

  CorrectUnfavourable Answer  fluctuations in exchange rate can wipe out the normal business profit   Your Answer He wants  to make profit   Match The Following Question Correct Answer Your Answer

Merchant Portfolio management banking Securitizat Prime Traditional banking service

Issue of debt in the form of Highest credit rating Loans and cash credit against security

Portfolio managem ent Issue of Highest Loans and cash credit against security

Â

 Multiple Choice Single Answer   Question A Specific  date forward contract can be booked so as to be :  CorrectPayable Answer onÂa specific date of the calendar month   Your Answer PayableÂon a specific date of the calendar month Â

 Multiple Choice Multiple Answer   Question What  are the sources of raising funds for exports?   CorrectSuppliers AnswerÂcredit  , Forfaiting   Your Answer Suppliers  credit , Forfaiting , Buyers credit

Â

 Select The Blank   Question The Âcounter party in option is ________.   CorrectThe Answer exchange    Your Answer Trustworthy  corporation, who hedges the risks of market uncertainty Â

 True/False   Question Transfer  of funds for investment abroad is allowed freely.   CorrectTrue Answer    Your Answer True  Â

 Multiple Choice Single Answer   Question What  do you understand by a swap?   Correct2Answer counter parties  agree to exchange streams of payments over a period of time   Your Answer 2 counter  parties agree to exchange streams of payments over a period of time Â

 True/False

  Question The ÂOption period in the forward contract is at the option of the Banker, hence the bank has to be very conservative in quoting rates.

  CorrectFalse Answer    Your Answer True  Â

 Multiple Choice Single Answer

  Question What  are the powers of money changers?

  CorrectPurchase Answer and  sell foreign currency notes and travellers cheques

  Your Answer Purchase  and sell foreign currency notes and travellers cheques

  Select The Blank   Question ________  is drawn by taking mean of the bid and offered rates.

  CorrectMiddle Answer rate   Your Answer Basic rate    True/False   Question The Âcredit rating of World Bank is high in the world Capital market because it operates at the debt equity ratio of 1:2   CorrectFalse Answer    Your Answer True    Multiple Choice Multiple Answer   Question What  are the activities of ICC?   CorrectAribtration Answer  and dispute resolving , Canvass for open trade combat commercial crime   Your Answer Aribtration  and dispute resolving , Canvass for open trade combat commercial crime   Multiple Choice Single Answer   Question Whose  permission was necessary for forming the Association of Foreign Exchange Dealers, in India?

  CorrectReserve AnswerÂBank  of India   Your Answer ReserveÂBank of India   Match The Following

Question

Correct Answer

Returning RFC NRI FCNR, NRE NRIS AND INVESTMENT IN MMMF EXPORT EEFC ERS

Your Answer

FCNR, INVESTM RFC Investmen t in GDR, ADR

Â

 True/False   Question Tranche'  is a managed issue of securities in smaller denominations.   CorrectTrue Answer    Your Answer True  Â

 Select The Blank   Question A Documentary  credit issued on the strength of another documentary credit is known as ________.   CorrectBack Answer to Back  credit   Your Answer Back to  Back credit Â

 Multiple Choice Single Answer   Question Which  is the most actively traded currency in the international market today?   CorrectUS Answer Dollar    Your Answer US Dollar  Â

 True/False

  Question Till August  1994 India was following the Indirect Methodof Rate quotation.

  CorrectFalse Answer    Your Answer True  Â

 Multiple Choice Multiple Answer   Question Which  debits are permitted to EEFC accounts?   CorrectPayment AnswerÂofÂhotel bills abroad, payment for expenses on Heart operation abroad , Payment to EOUs in India for services rendered by them   Your Answer Payment of hotel bills abroad, payment for expenses on Heart operation abroad , Payment to EOUs in India for services rendered by them , Investment abroad in stocks and shares by an Individual   True/False

  Question Normally  the forward contracts are booked for making huge profits.   CorrectFalse Answer Â

  Your Answer True Â

  Multiple Choice Multiple Answer   Question Which  factors influence the valuation of cash flows?

  CorrectIncomplete Answer Âcapital market , Availability of projkect specific concessional finance,   Your Answer Incomplete  capital market , Rates of inflation, , volatility of exchange rates

  True/False   Question Indian  companies are now allowed to freely invest abroad up to $1.5 million, raised through GDE or EEFC without prior permission from RBI.   CorrectFalse Answer    Your Answer True    Multiple Choice Multiple Answer   Question Name  the services provided by 'Reuter 3000' in forex market:  CorrectQuotation Answer of  forex rates, interest rates , Electronic broking by matching the rates   Your Answer Quotation  of forex rates, interest rates , Electronic broking by matching the rates , Calculate and quote rates for transactions   Multiple Choice Multiple Answer   Question Underwriters  are more attracted to NIF issue because of :  CorrectLesser Answer risks  , Increased returns , Easy marketability features

  Your Answer Lesser risks  , Increased returns , High underwriting fees

  Multiple Choice Multiple Answer   Question In which  cases can a Resident Individual in India acquire foreign securities without prior approval?   CorrectGift Answer from aÂperson outside India, or Inheritance from a person resident in or outside India , Security issued under Cashless employees stock option scheme   Your Answer Gift fromÂa person outside India, or Inheritance from a person resident in or outside India , Security issued under Cashless employees stock option scheme , Purchased from NRO/NRNR accounts   Select The Blank   Question A ________  Bill of lading does not specify the terms and conditions of carriage.   CorrectShort Answer Form   Your Answer Dirty    Multiple Choice Multiple Answer   Question What  does a speculator accept while dealing with an option?

  CorrectThe Answer exercise  or strike price, the amount , The expiration date and time, the method of payment   Your Answer The exercise  or strike price, the amount , The expiration date and time, the method of payment   Multiple Choice Single Answer   Question For a  spot transaction carried out on Thursday, currencies will be exchanged on :  CorrectMonday Answer    Your Answer Friday    Multiple Choice Single Answer   Question Inhow  many groups are INCOTERMS classified?   CorrectFour Answer    Your Answer Four    Select The Blank   Question In ________  transaction the delivery is postponed from the agreed date.

  CorrectRollover Answer    Your Answer Future    True/False   Question Phytosanitary/  Radiation/Fumigation certificates are in the category of Health Certificates

  CorrectTrue Answer    Your Answer True    Multiple Choice Multiple Answer   Question What  is the objective of FEDAI?

  CorrectLaying Answer down  terms and conditions of exchange business between Customers and Bankers , Interbank understanding for transacting forex business and exchange rates   Your Answer Laying down  terms and conditions of exchange business between Customers and Bankers , Growth of Indian Exports , Ensure the profitability of Forex business to the banks   True/False

  Question Out  of 15 member countries in European Union 12 countries commiteed themselves for a single currency.   CorrectFalse Answer    Your Answer True    Select The Blank   Question For setting  up joint ventures abroad or equity participation abroad, EXIM bank provides funds under ________ Scheme.   CorrectOverseas Answer investment  facility   Your Answer Equity Participation  scheme   Multiple Choice Multiple Answer   Question HowÂis the entry into any conuntry's money market or capital market is restricted?   CorrectBecause AnswerÂofÂdifferent tax systems , Because of different credit standards   Your Answer BecauseÂof different tax systems , Because of different credit standards , Because of resistance by dealer's associations   Select The Blank   Question A Bill of exchange payable 45 days from date of shipment, is a means of ________ from seller to the buyer.   CorrectCredit Answer    Your Payment  Select The Blank Â>Select The Blank

  Question Forward  margin makes a currency cheaper then the currency is at ________.   CorrectDiscount Answer    Your Answer PAR    Multiple Choice Multiple Answer

  Question Which  of these are not considered as good securities by the banker?   CorrectHouse Answer Airway  bill , Postal Receipt   Your Answer Postal Receipt  , House Airway bill   Multiple Choice Single Answer   Question The ÂDirect Investment abroad through share swap arrangement under normal route is permitted in a :  CorrectJV/WOS Answer    Your Answer JV/WOS Â

 Multiple Choice Single Answer

  Question WhoÂintroduced the worlds first exchange traded currency futures?   CorrectChicago AnswerÂMercantile  Exchange   Your Answer ChicagoÂMercantile Exchange   Multiple Choice Multiple Answer   Question WithÂreference to ECB, fund raising activity includes :-

  CorrectFloating Answer rate  notes, bonds, debentures , Syndicate loans, issue of shares

  Your Answer FloatingÂrate notes, bonds, debentures , Syndicate loans, issue of shares   Select The Blank   Question An option  on a future is ________.   CorrectDerivative Answer  on a derivative   Your Answer Irrelevant    Multiple Choice Single Answer   Question HowÂdoes ICC help poor countries?   CorrectAttract Answer Foreign  Direct investments togeather with UNCTAD   Your Answer Attract Foreign  Direct investments togeather with UNCTAD   Multiple Choice Multiple Answer

  Question Which  functions does the Bill of Lading serve?   CorrectItAnswer is a document  of title to goods. , It is receipt of goods   Your Answer It is a document  of title to goods. , It is receipt of goods   Match The Following Question Correct Answer Stage III in Members to meet the

Your Members

LastStage The First stage in the European d and Monetary Union (EMU) Stage II in EMU

Form a single currency Free movement of capital in EC

Free Establish ment of European Monetary Institute

Establishment of European Monetary Institute

Establish EURO Bank

Â

 Multiple Choice Single Answer

  Question For a  spot transaction carried out on Tuesday, where both centers have holiday on Wednesday, currencies will be exchanged on :  CorrectThursday AnswerÂifÂit is a working day at both the centers   Your Answer Thursday  if it is a working day at both the centers

  Match The Following Question Correct Answer Intolerable Drop the investment project risk

Country risk Stock market analysis Weightag es %

Cost of evluation does not justify analysis Banks are ahead

Factors' values

Your Cost of evluation does not justify analysis Continue the project Banks are ahead Factors' values

  True/False   Question Bid rate  is bidders buying rate and offered rate is bidders selling rate.   CorrectTrue Answer    Your Answer True  Â

 True/False   Question Resident  Individuals in India can acquire foreign bonus shares on the foreign securities already held by them without prior approval of RBI   CorrectTrue Answer    Your Answer True    Multiple Choice Multiple Answer   Question What  do you understand by Commercial Risk?

  CorrectProtracted Answer Âdefault by the buyer , Insolvency of the buyer   Your Answer Protracted  default by the buyer , Insolvency of the buyer , Risk of strikes due to which commercial activity is haulted   Multiple Choice Single Answer   Question A billÂof exchange issued by a bank which pays specified sum of money to the holder on a specific date is called :  CorrectBankers' AnswerÂAcceptance  Bond   Your Answer Usance ÂBill of Exchange   Select The Blank   Question Association  of several scheduled commercial banks for forex business is ________.   CorrectForeign Answer Exchange  Dealers Association of India   Your Answer Foreign  Exchange Dealers Association of India   Select The Blank   Question The Âmajor industrial power whose economy was not affected after the II world war was ________.   CorrectUSA Answer    Your Answer USA    True/False   Question Sterling  FRNs are issued in Europe, USA ,Asia   CorrectFalse Answer Â

  Your Answer True Â

  Multiple Choice Single Answer   Question Mr M has imported some goods from Greece. What is the maximum period within which he must make the payment?   CorrectSix Answer monthsÂfrom date of shipment   Your Answer Six months  from date of bill of exchange   Multiple Choice Single Answer   Question Under  the automatic route of Direct Investment Abroad, what is the restriction on market purchases?   Correct50% Answer of theÂnet worth of the investing company   Your Answer 50% of the  net worth of the investing company   Select The Blank   Question For importing  goods in India RBI insists on ________ value.   CorrectCIF Answer Â

  Your Answer FOB Â

  Multiple Choice Single Answer   Question What  is the objective of INCOTERMS?   CorrectArrange Answer forÂtransfer of risk from seller to buyer at a convenient place

  Your Answer ArrangeÂfor transfer of risk from seller to buyer at a convenient place   Multiple Choice Single Answer   Question Name  the members of Exchange Banks association :  CorrectForeign Answer Banks  at Bombay, Calcutta, Delhi, Madras, Amritsar,

  Your Answer Foreign  Banks at Bombay, Calcutta, Delhi, Madras, Amritsar,   True/False   Question Yankee  bond is issued for domestic borrowers.

  CorrectFalse Answer    Your Answer True    True/False   Question In the  forward rate quotations between two currencies, when one currency is at discount the other currency is at premium.   CorrectTrue Answer    Your Answer False    Select The Blank   Question Fixed  interest rate Eurobond without the option to convert to equity is known as ________.   CorrectStraight Answer Debt    Your Answer Straight ÂDebt   True/False   Question Cause  of increase in cost of public issues Underwriting fees , selling commission   CorrectTrue Answer    Your Answer True    Multiple Choice Single Answer   Question In the  syndicated loan arrangement the borrower has to execute :  CorrectSingle Answer Principal  loan agreement   Your Answer Single Principal  loan agreement   Multiple Choice Multiple Answer   Question What  criteria of lending is adopted by The International Development agency while giving loans to a country?   CorrectThe Answer per capita  income of the borrwoing country , The population of the borrwoing country   Your Answer The per Âcapita income of the borrwoing country , The repayment capacity. , The Security offerred.   Select The Blank   Question As aÂresult of the world wars in the 20th century the ________ of most of the countries suffered heavily.   CorrectEconomies Answer Â

  Your Answer Currencies    Select The Blank   Question Rupee  was liberalised on ________ account in 1993, and on current account in 1994.   CorrectTrade Answer    Your Answer Capital    True/False   Question Integrated  branches of Commercial banks handle forex transactions as well as options, swaps, futures, arbitrage operations   CorrectTrue Answer    Your Answer True    True/False   Question Floating  rate notes are traded in secondary market also.   CorrectTrue Answer    Your Answer True    Multiple Choice Single Answer   Question What  maxim is followed by the forex dealer when indirect method of exchange rate quotation is used?   CorrectBuy Answer Low, sell  High   Your Answer Buy High,  Sell Low   Select The Blank   Question In ________  Contract seller is like a domestic seller in destination country   CorrectDDP Answer    Your Answer DDP    Select The Blank   Question ________Bill  of lading must specify the name of the ship.   Correct"On Answer Board"   Your Answer "On Board"    Select The Blank   Question Indian  Co. intending to make investment under Automatic route needs to submit form ________ to RBI with enclosures.   CorrectODA Answer    Your Answer ODA    Multiple Choice Multiple Answer   Question What  is a 'speculator'?   CorrectTrader Answer whoÂis neither hedger nor arbitragor , He takes risks for profit   Your Answer Trader who  is neither hedger nor arbitragor , He takes risks for profit , He buys or sells to hedge his risk Â

 Multiple Choice Multiple Answer   Question The Âcharacteristics of IDA Loan are:   CorrectItAnswer gives loans  to poorer countries , The rate of interest is zero percent , The repayment period is very long 40 years   Your Answer It gives loans  to poorer countries , The rate of interest is zero percent , The repayment period is very long 40 years   True/False   Question Head  office of ICC is in Paris because of its First President Eliance Clementel.   CorrectTrue Answer    Your Answer True    Multiple Choice Multiple Answer   Question What  does the word '"Tombstone" stand for?   CorrectAdvertisement Answer  which lists the managers, underwriters, , List of Providers of recently floated issue   Your Answer Advertisement  which lists the managers, underwriters, , List of Providers of recently floated issue , Issue without planning, for short duration   Multiple Choice Single Answer   Question What  is Risk Syndication facility?   CorrectSharing Answer of  risk of non payment by supplier   Your Answer Sharing Âof risk of non payment by supplier   Multiple Choice Multiple Answer   Question What  do you understand by 'Foreign Exchange Risk'?   CorrectThe Answer change  in value of currency alters the home currency value of an asset , liability, , The change in value of currency alters the home currency value of profit, loss, or future cash flows   Your Answer The change  in value of currency alters the home currency value of an asset , liability, , The change in value of currency alters the home currency value of profit, loss, or future cash flows   Multiple Choice Multiple Answer   Question The Âmaximum period for which a forward contract can be granted by the AD depends upon which RBI directives?   CorrectMaximum Answer Period  of repatriation of export proceeds , Maximum Period of payment for Imports   Your Answer Maximum  Period of repatriation of export proceeds , Maximum Period of payment for Imports Â

 Multiple Choice Multiple Answer   Question A Forward  purchase contract was booked with an authorised dealer and subsequently cancelled .It :  CorrectCan Answer be booked  again with another dealer , Can be booked again with the same dealer   Your Answer Can be booked  again with another dealer , Can be booked again with the same dealer , Can be booked only for un booked portion of the contract.   True/False   Question BothÂfutures and options are traded on a public exchange.   CorrectFalse Answer    Your Answer False    Multiple Choice Multiple Answer   Question The Âcharacteristics of an International documentary credit are :  CorrectItAnswer is an undertaking  given by the openers bank to pay certain sum of money , It is an undertaking given on behalf of a certain person to another person in different country   Your It is an undertaking given by the openers Answer bank to pay certain sum of money , It is an  undertaking given on behalf of a certain person to another person in different country , It is an unconditional undertaking Â

LIST OF ATTEMPTED QUESTIONS AND ANSWERS Multiple Choice Multiple Answer Question What are the features of Settlement price ? Correct Answer Price established by the clearing house at the end of each trading session , It is used to determine the net gain, loss, margin call and next days price limits Your Answer Price established by the clearing house at the end of each trading session , It is used to determine the net gain, loss, margin call and next days price limits Select The Blank Question Legalised invoice is also called ________ invoice. Correct Answer Visaed Your Answer

Visaed

True/False Question

The working of the IBRD and IMF in recent has blurred the difference between the two Correct Answer True Your Answer

False

Multiple Choice Multiple Answer Question Under the liberalisation through FEMA, what can an AD offer to the clients, without the prior permission of RBI or Government? Correct Answer Hedges, interest rate swaps, currency swaps. , Caps and collars, and forward rate agreements Your Answer Hedges, interest rate swaps, currency swaps. , Caps and collars, and forward rate agreements , Good and competitive exchange rates and cross currency rates Select The Blank Question ________Bill of lading must specify the name of the ship. Correct Answer "On Board" Your Answer

"On Board"

Multiple Choice Single Answer Question What is the immediate cause for the growth of futures and options in the last two decades? Correct Answer Increase in volatile trading conditions

Your Answer

General market investments

Multiple Choice Single Answer Question In the context of International economy ,the period between 1920s and 1930s is known as Correct Answer Great Depression Your Answer

Great Depression

Match The Following Question

Correct Answer

Your Answer

Departure

EXW

EXW

Main carriage paid

CFR

CFR

Main carriage unpaid

FAS

FAS

Arrival

DES

DES

Multiple Choice Single Answer Question What is the statutory requirement about receivables of exports from India? Correct Answer The export receivables must be repatriated within six months of the date of shipment. Your Answer The export receivables must be repatriated within six months of the date of shipment. True/False Question

An exporter expecting payment in$ next month will book a forward purchase contract. Correct Answer False Your Answer

True

Multiple Choice Multiple Answer Question The characteristics of IDA Loan are: Correct Answer It gives loans to poorer countries , The rate of interest is zero percent , The repayment period is very long 40 years Your Answer It gives loans to poorer countries , The rate of interest is zero percent , The repayment period is very long 40 years Multiple Choice Multiple Answer Question What is the duty of the AD while booking a forward contract?

Correct Answer Verify the suitable documentary evidence, and that the customer is exposed to risk , Ensure that a genuine transaction exists Your Answer Verify the suitable documentary evidence, and that the customer is exposed to risk , Ensure that a genuine transaction exists , Protect the customer fully from exchange risk Multiple Choice Multiple Answer Question Criteria for Direct investment abroad under normal route :Correct Answer Viability of the proposal, and excellent track record of business , Experience and expertise of promoters, benefit to the country Your Answer Viability of the proposal, and excellent track record of business , Experience and expertise of promoters, benefit to the country , Total Financial commitment in JV/WOS as per prescribed ceiling Multiple Choice Single Answer Question When was the first currency swap introduced in world market? Correct Answer 1970 Your Answer

1970

True/False Question

Integrated branches of Commercial banks handle forex transactions as well as options, swaps, futures, arbitrage operations Correct Answer True Your Answer

True

Multiple Choice Multiple Answer Question Who can issue the Certificate of Inspection? Correct Answer Export Inspection Council , Person authorised by Importer. Your Answer

Export Inspection Council , Person authorised by Importer. , Reserve Bank of India

Match The Following Question

Correct Answer

Your Answer

Cancellation of contracts above USD 500000

ECD

FEDAI

Forward Purchase contract currency at premium

Presume earliest delivery Presume earliest delivery

Forward purchase contract currency at discount

Presume latest delivery

Presume latest delivery

Exchange margins

True/False Question

FEDAI

ECD

Both futures and options are traded on a public exchange.

Correct Answer False Your Answer

True

Multiple Choice Multiple Answer Question Internationally only those currencies are in demand Correct Answer Which are convertible , Which have liquidity confirmed in Euro market operations Your Answer Which are convertible , Which have liquidity confirmed in Euro market operations , Which have greater intrinsic value Multiple Choice Single Answer Question What was the official foreign currency reserve of the group of seven central Banks taken togeather in mid 1992? Correct Answer USD 275 Billion Your Answer

GBP 300 billion

Multiple Choice Single Answer Question A Bank quotes interest rate of 9.50/10.00 for six months dollar deposit, here the bid rate is :Correct Answer 9.50% P.A. Your Answer

9.50 divided by 6 =1.58% P.A.

True/False Question

JV means Joint venture and WOS means World of Security.

Correct Answer False Your Answer

True

Multiple Choice Multiple Answer Question How is the entry into any conuntry's money market or capital market is restricted? Correct Answer Because of different tax systems , Because of different credit standards Your Answer Because of different tax systems , Because of different credit standards , Because of protectionism True/False

Question

The American Dollar market comprises of the domestic segment of USD the offshore segment of USD. Correct Answer True Your Answer

True

Multiple Choice Multiple Answer Question The major cost components of NIF are :Correct Answer Underwriters fees , Margins of notes Your Answer

Underwriters fees , Documentation charges , Issue registration fees

True/False Question

FRN's issued with ceiling on interest beyond which the issues is not required to pay interest is known as Mini-Max FRN Correct Answer True Your Answer

False

Multiple Choice Single Answer Question A weight list, a packing list used in international trade are which type of documents? Correct Answer Operational Documents Your Answer

Optional documents

Select The Blank Question The credit which does not constitute a legally binding undertaking between the bank and the beneficiary is ________. Correct Answer Revocable Credit Your Answer

Revocable Credit

Multiple Choice Multiple Answer Question Sterling Market is attractive for an international investor because Correct Answer Gilts offer higher attractive margin , Choice of issuing medium term and long term bonds Your Answer Gilts offer higher attractive margin , Choice of issuing medium term and long term bonds , Linking of issues to currency swaps Multiple Choice Single Answer Question European currency Unit Correct Answer Basket of Currencies of members of EMS

Your Answer

Basket of Currencies of members of EMS

True/False Question

At Least one principal Lender bank is essential for issue of the ADR or GDR. Correct Answer False Your Answer

True

True/False Question

In an already booked forward contract, a person may seek early delivery, extension in period, or cancellation. Correct Answer True Your Answer

True

Multiple Choice Single Answer Question A Run on gold gold by a country means Correct Answer Exchange of reserve currency in to gold Your Answer

Exchange of gold into Reserve Currency

Multiple Choice Single Answer Question Punjab National Bank Mumbai Branch quoted USD 1= Rs 50.5000/52.5050. Which is the bid rate for USD? Correct Answer USD 1=Rs 50.5000 Your Answer

USD 1=Rs 50.5000

Multiple Choice Single Answer Question The Direct Investment abroad through share swap arrangement under normal route is permitted in a :Correct Answer JV/WOS Your Answer

JV/WOS

Select The Blank Question FRNs are issued for a maturity period of ________ and are normally redeemed in one shot. Correct Answer 5-7 years Your Answer True/False

5-7 years

Question

Bid rate is bidders buying rate and offered rate is bidders selling rate. Correct Answer True Your Answer

True

True/False Question

Corporates are allowed to raise funds abroad up to $ 15 million as short term finance if they have EPCG license. Correct Answer True Your Answer

False

Select The Blank Question Global capital market can be understood from the study of ________. Correct Answer Yen,Euro Dollar, Swiss Francs, USD market, Govt Borrowings within and outside the country Your Answer IMF, World Bank and local markets Select The Blank Question Short term instruments normally issued by corporate borrowers, forhigh values, at a discount to face value are known as ________. Correct Answer Euro Commercial Papers Your Answer

Euro Commercial Papers

Select The Blank Question Even after liberalisation of exchange controls, the Euro sterling market remained volatile because of uncertainties in UK and gyrations of sterling against major ________ countries Correct Answer OECD Your Answer

OECD

Multiple Choice Multiple Answer Question For issuing a Documentary credit The applicant is required to provide to the issuing bank details about Correct Answer The purchase-sales contract, the mode of payment, the tenor of payment , The documents required, the names of authorities who would issue the documents Your Answer The purchase-sales contract, the mode of payment, the tenor of payment , The documents required, the names of authorities who would issue the documents , Authority who will issue the documents Multiple Choice Multiple Answer

Question

The crisis in global capital market in 2002-04 was due to

Correct Answer Collapse of Enron , Collapse of Argentina's currency board , The war against terrorism Your Answer Collapse of Enron , Collapse of Argentina's currency board , The war against terrorism Select The Blank Question Over the years ________has been the most widely accepted currency in international capital market. Correct Answer American Dollar Your Answer

American Dollar

Select The Blank Question "Flip-Flop', 'Mini-Max', 'Capped' 'Perpetual' are the various types of ________. Correct Answer Floating Rate Note (FRN) Your Answer

Floating Rate Note (FRN)

Select The Blank Question Combined Certificate of Origin and Value are required by ________. Correct Answer Commonwealth Countries Your Answer

Commonwealth Countries

Multiple Choice Single Answer Question Bank with lowest Bid for fees etc and which undertakes the responsibility of loan syndication is called :Correct Answer The lead bank- leadmanager- arranger Your Answer

The lead bank- leadmanager- arranger

LIST OF ATTEMPTED QUESTIONS AND ANSWERS Multiple Choice Multiple Answer Question Which functions does the Bill of Lading serve? Correct Answer It is a document of title to goods. , It is receipt of goods Your Answer It is a document of title to goods. , It is receipt of goods Multiple Choice Single Answer Question A Bank quotes interest rate of 9.50/10.00 for six months dollar deposit, here the bid rate is :Correct Answer 9.50% P.A. Your Answer 9.50% P.A.

Multiple Choice Single Answer Question What is Sovereign Risk? Correct Answer Risk of lending to the government of a country Your Answer Risk of lending to the government of a country Multiple Choice Single Answer Question How does ICC help poor countries? Correct Answer Attract Foreign Direct investments togeather with UNCTAD Your Answer Attract Foreign Direct investments togeather with UNCTAD Multiple Choice Multiple Answer Question As per BIS, recovery from the recent crisis in global capital market was mainly due to Correct Answer Rapid and appropriate policy response in US and other countries , Increase in Foreign Direct Investments , free flow of credit Your Answer Rapid and appropriate policy response in US and other countries , Increase in Foreign Direct Investments , free flow of credit True/False Question Out of 15 member countries in European Union 12 countries commiteed themselves for a single currency. Correct Answer False Your Answer True True/False Question In red clause credit some clauses are printed in Red ink. Correct Answer True Your Answer True True/False Question Transfer of funds for investment abroad is allowed freely. Correct Answer True Your Answer False Select The Blank Question EEFC account holder is allowed to have ________ facility. Correct Answer Cheque book and nomination Your Answer Credit card, safe custody, safe deposit locker without any fee Select The Blank Question Fixed interest rate Eurobond without the option to convert to equity is known as ________. Correct Answer Straight Debt Your Answer Interest rate swap True/False Question An Indian importing goods from USA pays dollars in India. Correct Answer False Your Answer True Match The Following Question Correct Answer Your Answer PP Form Export Declaration form Export Declaration form Health Certificate Fumigation certificate Export

Declaration form Certificate of Inspection As per importers requirement Export Declaration form Certifiate of Origin Export Promotion Council Export Declaration form Multiple Choice Multiple Answer Question For issuing a Documentary credit The applicant is required to provide to the issuing bank details about Correct Answer The purchase-sales contract, the mode of payment, the tenor of payment , The documents required, the names of authorities who would issue the documents Your Answer The purchase-sales contract, the mode of payment, the tenor of payment , The track record of the buyer and seller Multiple Choice Multiple Answer Question The characteristics of an International documentary credit are :Correct Answer It is an undertaking given by the openers bank to pay certain sum of money , It is an undertaking given on behalf of a certain person to another person in different country Your Answer It is an undertaking given on behalf of a certain person to another person in different country , It is a stamped agreement and is not valid unless it is signed by the issuer True/False Question CFR = FOB plus freight paid by seller +insurance paid by seller. Correct Answer False Your Answer True Select The Blank Question Even after liberalisation of exchange controls, the Euro sterling market remained volatile because of uncertainties in UK and gyrations of sterling against major ________ countries Correct Answer OECD Your Answer OPEC True/False Question Integrated branches of Commercial banks handle forex transactions as well as options, swaps, futures, arbitrage operations Correct Answer True Your Answer True Multiple Choice Single Answer Question Which of them are special funding facilities of IMF ? Correct Answer Buffer Stock Facility Your Answer Supplemental Reserve Policy Multiple Choice Multiple Answer Question What are the sources of raising funds for exports? Correct Answer Suppliers credit , Forfaiting Your Answer Suppliers credit , Forfaiting , Buyers credit , Foreign currency or Rupee finance

True/False Question Normally the forward contracts are booked for making huge profits.

Correct Answer False Your Answer False True/False Question Head office of ICC is in Paris because of its First President Eliance Clementel. Correct Answer True Your Answer False Select The Blank Question A ________ Bill of lading does not specify the terms and conditions of carriage. Correct Answer Short Form Your Answer Short Form Multiple Choice Single Answer Question What is the volume of the world's total speculative transactions? Correct Answer Roughly 90 % of the total turnover Your Answer 70% of the total turnover True/False Question Till August 1994 India was following the Indirect Methodof Rate quotation. Correct Answer False Your Answer False Select The Blank Question For setting up joint ventures abroad or equity participation abroad, EXIM bank provides funds under ________ Scheme. Correct Answer Overseas investment facility Your Answer Overseas investment facility Multiple Choice Single Answer Question Mr P has exported goods in May 2005 and is expecting payment of Euro 25000 by the last week of Sept 2005. Give details of the forward contract he can book :Correct Answer He can book a sale contract any time from May 2005 till Mid Sept 2005, for Euro 25000 or less, for maturity last week of Sept. Your Answer He can book a contract for Forward Purchase, any time from May 2005 till Mid Sept 2005, for Euro 25000 or less, for maturity last week of Sept. Select The Blank Question In US Capital Market a Company prepares all the necessary documents in advance of the public issue under the facility known as ________. Correct Answer Shelf Registration Your Answer Shell Registration Multiple Choice Multiple Answer Question Which common details will be available in any bill of Lading? Correct Answer Names of consigner, consignee , The port of loading and unloading, contract of shipment Your Answer Names of consigner, consignee , The port of loading and unloading, contract of shipment , The name of the vessel , The date of transportation and contract of transportation Select The Blank Question Under the ________ standard the Country's Central Bank promised to buy back the currency at certain rate of Gold. Correct Answer Gold Your Answer Interest

Select The Blank Question Short term instruments normally issued by corporate borrowers, forhigh values, at a discount to face value are known as ________. Correct Answer Euro Commercial Papers Your Answer ADRs Select The Blank Question The type of trade where a country sells orange juice in return for cars is known as ________. Correct Answer Counter Trade Your Answer Non money trade Multiple Choice Multiple Answer Question Which debits are permitted to EEFC accounts? Correct Answer Payment of hotel bills abroad, payment for expenses on Heart operation abroad , Payment to EOUs in India for services rendered by them Your Answer Payment to EOUs in India for services rendered by them , Payment of Gifts of GBP 5000 per year per donor Multiple Choice Multiple Answer Question Under the liberalisation through FEMA, what can an AD offer to the clients, without the prior permission of RBI or Government? Correct Answer Hedges, interest rate swaps, currency swaps. , Caps and collars, and forward rate agreements Your Answer Caps and collars, and forward rate agreements , Good and competitive exchange rates and cross currency rates Multiple Choice Multiple Answer Question Sterling Market is attractive for an international investor because Correct Answer Gilts offer higher attractive margin , Choice of issuing medium term and long term bonds Your Answer Choice of issuing medium term and long term bonds , Less controls and less formalities to be completed , Linking of issues to currency swaps Multiple Choice Single Answer Question Under which contracts the seller pays the import duty for the buyer? Correct Answer DDP Your Answer DDP True/False Question PP Form is required when exports are made to any country by post parcel on value payable basis. Correct Answer False Your Answer True Multiple Choice Multiple Answer Question What were the immediate causes of converting the group of 5 into group of 7 and convene their meeting in late 1985? Correct Answer Many Industries in USA had become uncompetitive with imports , In USA the Trade protectionnism had increased Your Answer Many Industries in USA had become uncompetitive with imports , In USA the Trade protectionnism had increased , The heads of G5 insisted to have a meeting , Failure of IMF to control the exchange rates

Multiple Choice Single Answer Question What are the powers of money changers? Correct Answer Purchase and sell foreign currency notes and travellers cheques Your Answer Purchase and sell foreign currency notes and travellers cheques, give import and export finance Multiple Choice Single Answer Question Who can open an EEFC account? Correct Answer Individuals , firms, companies, resident in India Your Answer Individuals , firms, companies, resident in India Multiple Choice Single Answer Question What is the role of ICC towards WTO? Correct Answer To provide world business recommendations to WTO Your Answer To further the cause of WTO Select The Blank Question Position in a currency is said to be ________, when sales equal purchases. Correct Answer Square Your Answer Awkward Select The Blank Question Hotels and Airline companies are allowed to receive payments in Indian rupees from foreign tourist without insisiting on ________. Correct Answer Encashment certificate Your Answer Encashment certificate Multiple Choice Multiple Answer Question Why is Bill of Exchange a Quasi Negotiable Instrument? Correct Answer It represents title to goods , It is issued under Sale of Goods act Your Answer It represents title to goods , It is issued under Transportation act , It is issued under Bill of exchange act Multiple Choice Single Answer Question Which details about forward contracts do the Exchange Controls verify ? Correct Answer Particulars of cancellation of forward cover for the equivalent of USD 500,000 Your Answer Details of all contracts booked and cancelled beyond a specified limit Multiple Choice Single Answer Question In the context of International economy,the period between 1920s and 1930s is known as Correct Answer Great Depression Your Answer Great Depression Multiple Choice Multiple Answer Question What do you understand by Commercial Risk? Correct Answer Protracted default by the buyer, Insolvency of the buyer Your Answer Protracted default by the buyer, Insolvency of the buyer

LIST OF ATTEMPTED QUESTIONS AND ANSWERS Multiple Choice Single Answer Question What is the statutory requirement about receivables of exports from India? Correct Answer The export receivables must be repatriated within six months of the date of shipment. Your Answer The export receivables must be repatriated within six months of the date of shipment. Match The Following Question

Correct Answer

Your Answer

Maturity of future

Time when actual commodity is delivered

Last date of exercising option

Note issuance facility

Euronotes

Facility of issuing bonds

Bearer security issued in Eurobonds currency other than that of the country of issue

Eurobonds

Margin on future

Peformance bond or security deposit

Peformance bond or security deposit

Select The Blank Question When an Indian Company earns at least 50% of its average turnover from a particular activity then that activity is considered as ________. Correct Answer Core activity Your Answer

Core activity

True/False Question

In developed markets, speculators play important role in determining the trend of currency rates. Correct Answer True Your Answer

True

True/False Question

Cash flows generated from aforeign project may replace revenueproducing imports to the host country Correct Answer False Your Answer

False

Select The Blank

Question

In US Capital Market a Company prepares all the necessary documents in advance of the public issue under the facility known as ________. Correct Answer Shelf Registration Your Answer

Shelf Registration

True/False Question

Cross currency forward contract is not meaningful if original contract is in Rupees to USD Correct Answer True Your Answer

True

Multiple Choice Single Answer Question What maxim is followed by the forex dealer when direct method of exchange rate quotation is used? Correct Answer Buy High, Sell Low Your Answer

Buy Low, sell High

True/False Question

Sterling FRNs are issued in Europe, USA ,Asia

Correct Answer False Your Answer

False

Multiple Choice Single Answer Question Under which category of exchange rate did the Indian Rupee came from June 92? Correct Answer Other managed Floating Your Answer

Other managed Floating

Multiple Choice Single Answer Question In which currency can the EEFC account be opened? Correct Answer Any Convertible currency Your Answer

Any Convertible currency

Multiple Choice Single Answer Question When is intermediation resorted to? Correct Answer When The Lender does not know or trust the creditwothiness of the borrower Your Answer When The Lender does not know or trust the creditwothiness of

the borrower Multiple Choice Multiple Answer Question What is the effect of cross currency option on the customer? Correct Answer He gets the flexibility to exposure , He is exposed to additional risk of movement of rates between 3 currencies Your Answer He gets the flexibility to exposure , He is exposed to additional risk of movement of rates between 3 currencies Multiple Choice Single Answer Question How is the exporter paid through 'import loan'? Correct Answer The Bank delivers goods to the importer against his 'trustee' receipt, exporter sells the goods and then pays for the same Your Answer The Bank delivers goods to the importer against his 'trustee' receipt, exporter sells the goods and then pays for the same True/False Question

The working of the IBRD and IMF in recent has blurred the difference between the two Correct Answer True Your Answer

True

Multiple Choice Multiple Answer Question Mr A an Indian exporter is expecting to receive payment for exports in Hong kong dollar, after one month; How can he cover the exchange rate fluctuation risk? Correct Answer By booking a A forward contract in HKD itself , By booking a Cross currency option between USD and HKD and / or USD and Rs Your Answer By booking a A forward contract in HKD itself , By booking a Cross currency option between USD and HKD and / or USD and Rs Match The Following Question

Correct Answer

Your Answer

Forfaiting

Discounting export receivables

Discounting export receivables

Buyer's Credit

Available to Buyer abroad Available to Buyer abroad

Seller's Credit

Available to seller abroad

Available to seller abroad

Usance LC

Seller paid only on maturity date

Seller paid only on maturity date

Multiple Choice Single Answer Question Which currency is traded in one leg of about 70% of the world's transactions? Correct Answer US Dollar Your Answer

US Dollar

Select The Blank Question After the second world war many countries adopted the route of ________ to give facelift to sagging economies. Correct Answer Devaluation of their currencies Your Answer

Devaluation of their currencies

Multiple Choice Single Answer Question Which is the most actively traded currency in the international market today? Correct Answer US Dollar Your Answer

EURO

Multiple Choice Multiple Answer Question The integrated treasury branches of commercial banks handle which type of business? Correct Answer Forex transactions, Derivatives transactions , Mney market and bond market transactions Your Answer Forex transactions, Derivatives transactions , Mney market and bond market transactions , International loans, and borrowings Multiple Choice Multiple Answer Question The participants in the Forex Markets are :Correct Answer Non banking entities wishing to exchange currencies, , Banks and their clients , Speculators, arbitragers Your Answer Non banking entities wishing to exchange currencies, , Banks and their clients , Speculators, arbitragers Multiple Choice Multiple Answer Question Name the services provided by 'Reuter 3000' in forex market:Correct Answer Quotation of forex rates, interest rates , Electronic broking by matching the rates Your Answer Quotation of forex rates, interest rates , Electronic broking by matching the rates , Demat securities Multiple Choice Single Answer Question What is the volume of the world's average daily transactions in foreign exchange?

Correct Answer USD 1 trillion Your Answer

USD 1 trillion

Select The Blank Question EUROCLEAR, CEDEL are the names of ________ related with the issue of GDR. Correct Answer Registrars Your Answer

Registrars

Select The Blank Question Under the ________ standard the Country's Central Bank promised to buy back the currency at certain rate of Gold. Correct Answer Gold Your Answer

Gold

True/False Question

Multimodal transport document is also known as "IntermobileTransport Document" Correct Answer True Your Answer

True

Multiple Choice Multiple Answer Question Export finance Correct Answer Preshipment credit , Post shipment credit , Duty Draw back Your Answer

Preshipment credit , Post shipment credit , Duty Draw back

Select The Blank Question Value of USDollar is ________ in the world market but RBI is trying to match the Rupee value with it. Correct Answer Depreciating Your Answer

Depreciating

Multiple Choice Single Answer Question Why did Govt of India initially exercise controls on foreign receipts and payments? Correct Answer Conserve and mobilise foreign exchange for war effort Your Answer

Conserve and mobilise foreign exchange for war effort

Multiple Choice Single Answer

Question

What do firms face while evaluating risks in cross border investment? Correct Answer The issues are different, unique to each project, and solutions are varied Your Answer The issues are different, unique to each project, and solutions are varied Select The Blank Question EEFC account holder is allowed to have ________ facility. Correct Answer Cheque book and nomination Your Answer

Cheque book and nomination

True/False Question

A small percentage of futures contracts actually result in delivery of the underlying commodity or security. Correct Answer True Your Answer

False

True/False Question

The Option period in the forward contract is at the option of the Banker, hence the bank has to be very conservative in quoting rates. Correct Answer False Your Answer

True

Multiple Choice Multiple Answer Question Which general risks are covered by any insurance policy for transit? Correct Answer Theft, loss , Damage, sea perils, non delivery Your Answer

Theft, loss , Damage, sea perils, non delivery

Multiple Choice Multiple Answer Question Mr A sells an option conferring the right to buy a kilogram of silver at Rs 8000/- for next six months means :Correct Answer The buyer of option can buy silver from Mr A at 8000/- per Kg, in next 6 months, even if the price goes up, , The buyer of option can sell silver to Mr A at 8000/- per Kg, in next 6 months, even if the price goes down. Your Answer The buyer of option can buy silver from Mr A at 8000/- per Kg, in next 6 months, even if the price goes up, , The buyer of option can sell silver to Mr A at 8000/- per Kg, in next 6 months, even if the price goes down.

Select The Blank Question In mid 1992 ________ rejected the ratification of Maastricht treaty Correct Answer Denmark Your Answer

Denmark

Multiple Choice Multiple Answer Question What non funded facilities are granted by Export Import Bank of India? Correct Answer Guarantee for Bid bond, execution of export project, for borrowings abroad, for retention of money , Advance payment Guarantee, performance guarantee Your Answer Guarantee for Bid bond, execution of export project, for borrowings abroad, for retention of money , Advance payment Guarantee, performance guarantee Select The Blank Question FRNs are issued for a maturity period of ________ and are normally redeemed in one shot. Correct Answer 5-7 years Your Answer

5-7 years

Multiple Choice Multiple Answer Question Which factors affect the exchange rates in short time period? Correct Answer Demand and supply, interest rates, inflation, , Sentiment, Speculation Your Answer Demand and supply, interest rates, inflation, , Sentiment, Speculation , GDP,Balance of trade and payment,Unemployment Select The Blank Question ________ is the acid test for events to be included in country risk. Correct Answer Control by Government' Your Answer

Control by Government'

Multiple Choice Single Answer Question How many types of documents are required in the International trade? Correct Answer Six Your Answer

Six

Multiple Choice Multiple Answer Question What are the main causes of fluctuations in Exchange rates?

Correct Answer Government regulatios, fiscal policies, political instabilities , Market demand and supply, sentiments Your Answer Government regulatios, fiscal policies, political instabilities , Market demand and supply, sentiments , The purchasing power of currency changes every second Multiple Choice Multiple Answer Question Which two countries were included to convert the group of five into the group of seven? Correct Answer Canada , Italy Your Answer

Canada , Italy

Multiple Choice Single Answer Question What do you call a 'decreasing principal swap'? Correct Answer Amortizing swap Your Answer

Amortizing swap

Multiple Choice Single Answer Question Who are the members of the Special committee on Indian Direct Investments abroad, for normal route? Correct Answer Representatives of Ministry of finance, Ministry of Economic affairs, Ministry of Commerce and RBI Your Answer Representatives of Ministry of finance, Ministry of Economic affairs, Ministry of Commerce and RBI Multiple Choice Single Answer Question What are the functions of Export Import Bank of India? Correct Answer Finance functions and development functions Your Answer

Finance functions and development functions

LIST OF ATTEMPTED QUESTIONS AND ANSWERS True/False Question

Services of a Paying bank are taken when currencies of exporters country or the importers country is not involved in the transaction. Correct Answer False Your Answer

False

Select The Blank Question To make the export more competitive and remunerative, the Indian Govt extends ________ facility. Correct Answer Cash Incentives Your Answer

Line of credit

Multiple Choice Multiple Answer Question For issuing a Documentary credit The applicant is required to provide to the issuing bank details about Correct Answer The purchase-sales contract, the mode of payment, the tenor of payment , The documents required, the names of authorities who would issue the documents Your Answer The purchase-sales contract, the mode of payment, the tenor of payment , The documents required, the names of authorities who would issue the documents , The track record of the buyer and seller Multiple Choice Multiple Answer Question Which of these are not considered as good securities by the banker? Correct Answer House Airway bill , Postal Receipt Your Answer

House Airway bill , Postal Receipt , On board bill of exchange

Multiple Choice Single Answer Question What is the statutory requirement about receivables of exports from India? Correct Answer The export receivables must be repatriated within six months of the date of shipment. Your Answer The export receivables must be repatriated within six months of the date of shipment. Multiple Choice Single Answer Question What do firms face while evaluating risks in cross border investment? Correct Answer The issues are different, unique to each project, and solutions are varied Your Answer

The issues are different, unique to each project, and solutions are varied

True/False Question

An Indian importing goods from USA pays dollars in India.

Correct Answer False Your Answer

True

Multiple Choice Multiple Answer Question Name the services provided by 'Reuter 3000' in forex market:Correct Answer Quotation of forex rates, interest rates , Electronic broking by matching the rates Your Answer Quotation of forex rates, interest rates , Electronic broking by matching the rates , Calculate and quote rates for transactions Match The Following Question

Correct Answer

Your Answer

16 principles for good environmental conduct

Business Charter of sustainable development

Business Charter of sustainable development

World Chambers Congress

World Federation of Chambers

Doha

ICC Conference in 2004

Marrakesh

Marrakesh

13 types of trade contracts

INCOTERM

World Federation of Chambers

Multiple Choice Single Answer Question What is the cause of Volatility in the exchange market? Correct Answer Interest rates are changing, Interest rates affect and are affected by exchange rates Your Answer Interest rates are changing, Interest rates affect and are affected by exchange rates Multiple Choice Single Answer Question How many Countries participated in the third survey on Foreign Exchange, coordinated by BIS in 1992? Correct Answer 26 Your Answer

26

True/False Question

Cash flows generated from aforeign project may replace revenueproducing imports to the host country Correct Answer False Your Answer

True

Multiple Choice Multiple Answer Question Internationally only those currencies are in demand Correct Answer Which are convertible , Which have liquidity confirmed in Euro market operations

Your Answer

Which are convertible , Which have liquidity confirmed in Euro market operations , which have greater gold parity

True/False Question

Out of 15 member countries in European Union 12 countries commiteed themselves for a single currency. Correct Answer False Your Answer

False

Multiple Choice Multiple Answer Question Major investors in NIF are :Correct Answer Commercial Banks , Non banking Finance companies , Insurance Companies Your Answer Non banking Finance companies , Commercial Banks Select The Blank Question The major industrial power whose economy was not affected after the II world war was ________. Correct Answer USA Your Answer

Germany

Multiple Choice Multiple Answer Question What are the main causes of fluctuations in Exchange rates? Correct Answer Government regulatios, fiscal policies, political instabilities , Market demand and supply, sentiments Your Answer Government regulatios, fiscal policies, political instabilities , Market demand and supply, sentiments , The purchasing power of currency changes every second Multiple Choice Single Answer Question The severemost problem faced by the Fixed exchange rate system was Correct Answer Inability of US to change gold for dollars Your Answer

Measurement of world liquidity

Multiple Choice Multiple Answer Question What schemes have been floated by RBI for facilitating Inter bank transactions? Correct Answer Clearing house for settlement and clearance for the operations of Ads , Set up a market intelligence cell to study and monitor developments in Forex Market Your Answer Clearing house for settlement and clearance for the operations of Ads , Set up a market intelligence cell to study and monitor developments in Forex Market , Purchase excess foreign curency from Ads at higher rates Select The Blank

Question

Over the years ________has been the most widely accepted currency in international capital market. Correct Answer American Dollar Your Answer

Euro Dollar

Match The Following Question

Correct Answer

Your Answer

Departure

EXW

EXW

Main carriage paid

CFR

FAS

Main carriage unpaid

FAS

FAS

Arrival

DES

DES

True/False Question

The American Dollar market comprises of the domestic segment of USD the offshore segment of USD. Correct Answer True Your Answer

True

True/False Question

Tranche' is a managed issue of securities in smaller denominations.

Correct Answer True Your Answer

True

Multiple Choice Single Answer Question Which is the most actively traded currency in the international market today? Correct Answer US Dollar Your Answer

EURO

True/False Question

Prior approval of RBI is needed for payment of all imports.

Correct Answer False Your Answer

True

Select The Blank Question Mostly, countries in the Middle east require ________ invoice, for facilitating import duties .

Correct Answer Legalised Your Answer

Customs

Select The Blank Question A ________ Bill of lading does not specify the terms and conditions of carriage. Correct Answer Short Form Your Answer

Short Form

True/False Question

Cause of increase in cost of public issues Underwriting fees , selling commission Correct Answer True Your Answer

True

Multiple Choice Single Answer Question How many types of documents are required in the International trade? Correct Answer Six Your Answer

Six

Multiple Choice Single Answer Question If Investment is out of EEFC accounts then :Correct Answer Exemption is granted from the criteria of Core activity stipulation under automatic route Your Answer Exemption is granted from the criteria of Core activity stipulation under automatic route Select The Blank Question When a credit is made to available to additional beneficiary wholly or partly it is called as ________. Correct Answer Transferable credit Your Answer

Transferable credit

Select The Blank Question India is a founder member of ________. Correct Answer IMF Your Answer

IMF

True/False Question

Bid rate is bidders buying rate and offered rate is bidders selling rate.

Correct Answer True Your Answer

True

Select The Blank Question When interest rate changes by 0.05 per cent then it has changed ________ basis points. Correct Answer 50 Your Answer

5

Select The Blank Question Partnership firm not eligible for Overseas Direct Investment needs to apply to RBI Approval in form ________. Correct Answer ODI Your Answer

ODA

Multiple Choice Multiple Answer Question With reference to ECB, fund raising activity includes :Correct Answer Floating rate notes, bonds, debentures , Syndicate loans, issue of shares Your Answer

Floating rate notes, bonds, debentures , Syndicate loans, issue of shares

Select The Blank Question Rupee was liberalised on ________ account in 1993, and on current account in 1994. Correct Answer Trade Your Answer

Capital

Select The Blank Question FRNs are issued for a maturity period of ________ and are normally redeemed in one shot. Correct Answer 5-7 years Your Answer

Up to 1 year

Multiple Choice Multiple Answer Question What are the objectives of International Chamber of Commerce? Correct Answer Champion the global economy as a force of economic growth , Creating of job and prosperity Your Answer Champion the global economy as a force of economic growth , Creating of job and prosperity Multiple Choice Multiple Answer Question Which contracts are used for transportation by road?

Correct Answer FCA , CPT Your Answer

FCA , CPT

Multiple Choice Single Answer Question Financial management is complex process for Correct Answer Companies which have international trade Your Answer

Companies which have international trade

True/False Question

The volume of trading of Derivatives in the world trade market is growing but still only 5% of the total. Correct Answer False Your Answer

True

Multiple Choice Multiple Answer Question What are the functions of International Finance Corporation? Correct Answer Help private enterprise by financing projects , Provide technical assistance to private enterprise Your Answer Help private enterprise by financing projects , Provide technical assistance to private enterprise , Help to maintain the exchange rate stability Multiple Choice Multiple Answer Question What does a speculator accept while dealing with an option? Correct Answer The exercise or strike price, the amount , The expiration date and time, the method of payment Your Answer The exercise or strike price, the amount , The expiration date and time, the method of payment , The Market report of the exchange which trades the option Select The Blank Question The ________is a Floating rate index for swaps. Correct Answer US Commercial paper rate Your Answer

US Commercial paper rate

Multiple Choice Single Answer Question What is the role of ICC towards WTO? Correct Answer To provide world business recommendations to WTO Your Answer

To provide world business recommendations to WTO

LIST OF ATTEMPTED QUESTIONS AND ANSWERS True/False Question

In a coupon swap the receiver is the one who receives fixed rate of interest. Correct Answer True Your Answer

False

Multiple Choice Single Answer Question What is the volume of the world's average daily transactions in foreign exchange? Correct Answer USD 1 trillion Your Answer

USD 1 trillion

Multiple Choice Multiple Answer Question What are the main causes of fluctuations in Exchange rates? Correct Answer Government regulatios, fiscal policies, political instabilities , Market demand and supply, sentiments Your Answer Government regulatios, fiscal policies, political instabilities , Market demand and supply, sentiments , The purchasing power of currency changes every second Multiple Choice Single Answer Question Why have futures and options come into existence? Correct Answer To avoid business risk. Your Answer

To avoid business risk.

Multiple Choice Single Answer Question Whose permission was necessary for forming the Association of Foreign Exchange Dealers, in India? Correct Answer Reserve Bank of India Your Answer

Exchange Banks' Association

Select The Blank Question A Dealer bought USD at 47.5000, in morning and the USD is quoted in the noon at 1 USD =48.2000/49.7000, then he has the opportunity for ________. Correct Answer Arbitrage Your Answer

Arbitrage

Select The Blank Question Partnership firm not eligible for Overseas Direct Investment needs to apply to RBI Approval in form ________. Correct Answer ODI Your Answer

ODA

Select The Blank Question A ________ Bill of lading does not specify the terms and conditions of carriage. Correct Answer Short Form Your Answer

Dirty

Multiple Choice Single Answer Question Who arranges for Insurance and transportation under EXW contract? Correct Answer Buyer arranges and pays Your Answer

Seller

True/False Question

JV means Joint venture and WOS means World of Security.

Correct Answer False Your Answer

False

Match The Following Question

Correct Answer

Your Answer

European option

On or after maturity

Before maturity

Out of money

Loss

Before maturity

In the money

Profit

no profit no loss

American option

Before maturity

Buy the contract

Multiple Choice Multiple Answer Question What were the primary objectives of the group of seven countries? Correct Answer Bring down the external value of US Dollar , Bring some stability to exchange rates of all the countries Your Answer Bring down the external value of US Dollar , Bring some stability

to exchange rates of all the countries , Stop the process of protectionism in USA Select The Blank Question As a result of the world wars in the 20th century the ________ of most of the countries suffered heavily. Correct Answer Economies Your Answer

Economies

Multiple Choice Single Answer Question What are the powers of restricted money changers? Correct Answer Purchase foreign currencies Your Answer

Purchase and sell foreign currency notes and travellers cheques

Multiple Choice Multiple Answer Question Why does a Transaction exposure arise? Correct Answer Trading flows , Dividend or capital flows Your Answer

Trading flows , Valuation of assets and liabilities abroad in financial statements

Multiple Choice Single Answer Question What was the purpose of reviewing FERA 1973 in the year 1993? Correct Answer Economic Liberalisation relating to Foreign trade and Foreign investments Your Answer Economic Liberalisation relating to Foreign trade and Foreign investments True/False Question

Till August 1994 India was following the Indirect Methodof Rate quotation. Correct Answer False Your Answer

True

True/False Question

Services of a Paying bank are taken when currencies of exporters country or the importers country is not involved in the transaction. Correct Answer False Your Answer

True

Multiple Choice Multiple Answer

Question

What are the sources of raising funds for exports?

Correct Answer Suppliers credit , Forfaiting Your Answer

Forfaiting , Buyers credit , Foreign currency or Rupee finance

True/False Question

An Indian importing goods from USA pays dollars in India.

Correct Answer False Your Answer

True

Multiple Choice Single Answer Question In case the value date happens to be a holiday then the settlement is done :Correct Answer On the next working day Your Answer

On the next working day

Select The Blank Question After the second world war many countries adopted the route of ________ to give facelift to sagging economies. Correct Answer Devaluation of their currencies Your Answer

Devaluation of their currencies

Select The Blank Question The rate of interest on rupee finance for exports is not exceeding ________ less than PLR. Correct Answer 0.025 Your Answer

50 Basis points

Multiple Choice Multiple Answer Question With reference to ECB, fund raising activity includes :Correct Answer Floating rate notes, bonds, debentures , Syndicate loans, issue of shares Your Answer Floating rate notes, bonds, debentures , Syndicate loans, issue of shares , Futures and Forward Rate agreements Select The Blank Question Mostly, countries in the Middle east require ________ invoice, for facilitating import duties . Correct Answer Legalised Your Answer

Customs

Multiple Choice Single Answer Question Which invoice is drawn in specific form to be supplied by the consular office of the importer's country, for treatment of preferential tariff rates? Correct Answer Customs invoice Your Answer

Customs invoice

Multiple Choice Single Answer Question Under which method of payments is the buyer not required to pay till he gets the possession of the documents? Correct Answer Documents against payment terms specified in the bill of exchange Your Answer Documents against payment terms specified in the bill of exchange Multiple Choice Single Answer Question What is the volume of the world's total speculative transactions? Correct Answer Roughly 90 % of the total turnover Your Answer

70% of the total turnover

Select The Blank Question ________ enables bankers to extend purchase, negotiation or discount of export bills. Correct Answer Post shipment credit guarantee Your Answer

Export finance guarantee

Multiple Choice Multiple Answer Question Which two countries were included to convert the group of five into the group of seven? Correct Answer Italy , Canada Your Answer

Italy , China

True/False Question

The working of the IBRD and IMF in recent has blurred the difference between the two Correct Answer True Your Answer

True/False Question

True

Cash flows generated from aforeign project may replace revenue-producing imports to the host country

Correct Answer False Your Answer

False

Multiple Choice Single Answer Question What do you call a 'decreasing principal swap'? Correct Answer Amortizing swap Your Answer

Amortizing swap

Multiple Choice Multiple Answer Question These are the parties to the documentary credit :Correct Answer Negotiating Bank, advising bank , Reimbursing bank, Confirming bank Your Answer Negotiating Bank, advising bank , Reimbursing bank, Confirming bank , The customs and excise authorities Multiple Choice Multiple Answer Question Which of these are considered as miscellaneous documents? Correct Answer Packing list , Weight certificate Your Answer

Packing list , Weight certificate , PP Form

Select The Blank Question ECGC provides cover to an Indian contractor for construction work abroad under ________. Correct Answer Construction works policy Your Answer

Services Policy

Multiple Choice Single Answer Question What was the reason for launching swap as an risk hedging instrument? Correct Answer To avoid the effects of currency control imposed in UK Your Answer

TO exchange currency debts

Multiple Choice Multiple Answer Question In a documentary credit the word document means :Correct Answer Specified documents of title, transportation, insurance , Specified certificates, and other papers required as per the regulations in the buyers country Your Answer Specified documents of title, transportation, insurance , Specified certificates, and other papers required as per the regulations in the buyers country , Credit is documented as an

evidence Select The Blank Question EEFC account holder is allowed to have ________ facility. Correct Answer Cheque book and nomination Your Answer

Loan against the EEFC balance

True/False Question

Resident Individuals in India can acquire foreign bonus shares on the foreign securities already held by them without prior approval of RBI Correct Answer True Your Answer

True

Match The Following Question Issues linked with LIBOR,LIBID,LIMEAN

Correct Answer

Your Answer

FRNs

American or Global Depository Receipts

Issue of bonds without FloatingRate Note fixed rate of interest, linked to short term interest rate in the global market

Mergers & Amalgamations

Issue of loans Syndicated Loans internationally,through arranger, underwriter,co manager, for a large amount for sharing risk of lending

FRNs

Equity issues in the American or Global foreign markets either as Depository Receipts direct investments or portfolio investments.

Syndicated Loans

Multiple Choice Multiple Answer Question In DAF, the frontier means :Correct Answer Frontier of exporter's country , Frontier of importer's country Your Answer

Frontier of exporter's country , Frontier of importer's country , Frontier of the intermediate country where goods are off loaded

Multiple Choice Multiple Answer

Question

What do you understand by 'Foreign Exchange Risk'?

Correct Answer The change in value of currency alters the home currency value of an asset , liability, , The change in value of currency alters the home currency value of profit, loss, or future cash flows Your Answer The change in value of currency alters the home currency value of an asset , liability, , The change in value of currency alters the home currency value of profit, loss, or future cash flows , Certainty of an event which would happen in favour of the importer/ exporter Multiple Choice Multiple Answer Question Which contracts are used for transportation by road? Correct Answer FCA , CPT Your Answer

FCA , FOB , CFR

True/False Question

Cross currency forward contract is not meaningful if original contract is in Rupees to USD Correct Answer True Your Answer

False

Select The Blank Question Fixed interest rate Eurobond without the option to convert to equity is known as ________. Correct Answer Straight Debt Your Answer

hybrid

True/False Question

With the advent of futures and Options there is no need for additional instruments of risk hedging. Correct Answer False Your Answer

False

LIST OF ATTEMPTED QUESTIONS AND ANSWERS Select The Blank Question Profitability of a project in China to an Indian may be enhanced by ________. Correct Answer Concessionary finance available in China Your Answer

Subsidiaries and Tax concessions in India

Select The Blank Question The currency swaps used ________. Correct Answer To lower the risk of currency exposure Your Answer

To lower the risk of currency exposure

Match The Following Question

Correct Answer

Your Answer

Treasury management

Profit making

Banks and multinationals

Arbitraging

Rate disparity

Speculation

Futures, Options

Hedging instruments

Hedging instruments

Exchange Rate volatility

Speculation

Rate disparity

Multiple Choice Multiple Answer Question At the request of M/s Universal Traders Calcutta, State Bank of India opened a documentary credit in Favour of M/s Star Trading Co Dubai, for USD 25000/-.Here State Bank of India Calcutta is known as Correct Answer Issuing Bank , Opening Bank Your Answer

Issuing Bank , Exporters Bank

True/False Question

In UK in london Market the Indirect Quote Method is still adopted. Correct Answer True Your Answer

True

True/False Question

The volume of trading of Derivatives in the world trade market is growing but still only 5% of the total. Correct Answer False Your Answer

True

Multiple Choice Multiple Answer Question How does RBI control Exports? Correct Answer Through Rule like "Full value of Export bills must be realised in 180 days," , Through Rule like " Export must be denominated In

Your Answer

a currency of the Importing country or a freely convertible currency" Through Rule like "Full value of Export bills must be realised in 180 days," , Through Rule like " Export must be denominated In a currency of the Importing country or a freely convertible currency" , By Issuing a Export Code Number, and monitoring his exports through returns and statements filed by him

Select The Blank Question The large flow of money internationally was caused primarily by the oil price hikes engineered by ________. Correct Answer OPEC countries Your Answer

OPEC countries

True/False Question

The Negotiating bank examines the documents submitted by the seller and makes payment if they comply the terms and conditions of the LC. Correct Answer True Your Answer

True

Multiple Choice Single Answer Question What is the primary concern of the seller in the International Trade? Correct Answer To be sure to get the payment for goods, in time and in the required currency. Your Answer To be sure to get the payment for goods, in time and in the required currency. Multiple Choice Multiple Answer Question The characteristics of an International documentary credit are :Correct Answer It is an undertaking given by the openers bank to pay certain sum of money , It is an undertaking given on behalf of a certain person to another person in different country Your Answer It is an undertaking given by the openers bank to pay certain sum of money , It is an undertaking given on behalf of a certain person to another person in different country , It is a stamped agreement and is not valid unless it is signed by the issuer Select The Blank Question To make the export more competitive and remunerative, the Indian Govt extends ________ facility. Correct Answer Cash Incentives Your Answer

Cash Incentives

Multiple Choice Single Answer Question Name the method of payment where the seller neither gets the guarantee of payment from the bank, nor the possibility of negotiation of documents :Correct Answer Bills for Collection Your Answer

Bills for Collection

Multiple Choice Multiple Answer Question Which of these are not considered as good securities by the banker? Correct Answer House Airway bill , Postal Receipt Your Answer

House Airway bill , Postal Receipt

True/False Question

FRN's issued with ceiling on interest beyond which the issues is not required to pay interest is known as Mini-Max FRN Correct Answer True Your Answer

True

Select The Blank Question EUROCLEAR, CEDEL are the names of ________ related with the issue of GDR. Correct Answer Registrars Your Answer

Registrars

True/False Question

Phytosanitary/ Radiation/Fumigation certificates are in the category of Health Certificates Correct Answer True Your Answer

True

Multiple Choice Single Answer Question Financial management is complex process for Correct Answer Companies which have international trade Your Answer

Companies which have international trade

Select The Blank Question Global capital market can be understood from the study of ________.

Correct Answer Yen,Euro Dollar, Swiss Francs, USD market, Govt Borrowings within and outside the country Your Answer IMF, World Bank and local markets Multiple Choice Single Answer Question Exchange Rate means Correct Answer Value of one currency in terms of another Your Answer

Value of one currency in terms of another

Multiple Choice Multiple Answer Question The crisis in global capital market in 2002-04 was due to Correct Answer Collapse of Enron , Collapse of Argentina's currency board , The war against terrorism Your Answer Collapse of Enron , Collapse of Argentina's currency board , The war against terrorism Select The Blank Question In Indian interbank market forward rates are generally quoted on the basis of ________. Correct Answer Calendar month Your Answer

Calendar month

Multiple Choice Single Answer Question What is the Indian Govt's obligation of becoming member of IMF? Correct Answer Ensure orderly exchange arrangement, promote stable exchange rate Your Answer Ensure orderly exchange arrangement, promote stable exchange rate Select The Blank Question Over the years ________has been the most widely accepted currency in international capital market. Correct Answer American Dollar Your Answer

American Dollar

True/False Question

The American Dollar market comprises of the domestic segment of USD the offshore segment of USD. Correct Answer True Your Answer

True

Multiple Choice Multiple Answer Question The various arms of World Bank are: Correct Answer International Development Agency , Multinational Investment Guarantee Agency Your Answer International Development Agency , Multinational Investment Guarantee Agency , Asian Development Bank Multiple Choice Single Answer Question Under which category of exchange rate did the Indian Rupee came from June 92? Correct Answer Other m anaged Floating Your Answer

Other managed Floating

Multiple Choice Single Answer Question The need for global finance for a company is mainly because of :Correct Answer Availability of cheaper funds Your Answer

Increased opportunities of investment abroad

True/False Question

Cash flows generated from aforeign project may replace revenue-producing imports to the host country Correct Answer False Your Answer

True

True/False Question

In an interest rate swap if the payment schedules are identical, only diffrence between the two payments is delivered. Correct Answer True Your Answer

True

Multiple Choice Multiple Answer Question Sterling Market is attractive for an international investor because Correct Answer Gilts offer higher attractive margin , Choice of issuing medium term and long term bonds Your Answer Gilts offer higher attractive margin , Choice of issuing medium term and long term bonds , Linking of issues to currency swaps Multiple Choice Single Answer Question In case of syndication of loans the amount of loan and currency is finalised by :Correct Answer Borrower

Your Answer

Leader of the syndicate

True/False Question

Yankee bond is issued for domestic borrowers.

Correct Answer False Your Answer

True

Select The Blank Question A Bill of exchange payable 45 days from date of shipment, is a means of ________ from seller to the buyer. Correct Answer Credit Your Answer

Credit

True/False Question

Country risk analysis is a futile exercise in the background of cost of analysis. Correct Answer True Your Answer

True

Multiple Choice Single Answer Question The 'On Board' bill of lading means bill of lading evidencing that :Correct Answer The goods are put on board a ship for shipment Your Answer

The goods are put on board a ship for shipment

Multiple Choice Multiple Answer Question Which factors influence the valuation of cash flows? Correct Answer Incomplete capital market , Availability of projkect specific concessional finance, Your Answer Availability of projkect specific concessional finance, , Rates of inflation, , volatility of exchange rates Match The Following Question

Correct Answer

Your Answer

Forfaiting

Discounting export receivables

Discounting export receivables

Buyer's Credit

Available to Buyer abroad Available to Buyer abroad

Seller's Credit

Available to seller abroad Available to seller abroad

Usance LC

Seller paid only on maturity date

Seller paid only on maturity date

Multiple Choice Multiple Answer Question What criteria of lending is adopted by The International Development agency while giving loans to a country? Correct Answer The per capita income of the borrwoing country , The population of the borrwoing country Your Answer The per capita income of the borrwoing country , The population of the borrwoing country Multiple Choice Single Answer Question How many types of documents are required in the International trade? Correct Answer Six Your Answer

Six

Select The Blank Question ________Bill of lading must specify the name of the ship. Correct Answer "On Board" Your Answer

"On Board"

Select The Blank Question In Overseas market the forward rate is generally quoted on the basis of ________. Correct Answer Fixed date Your Answer

Fixed date

Multiple Choice Single Answer Question Revolving underwriting facility, short term note issuance facilty, transferabla revolving undertaking facility are the names of :Correct Answer Note issuance facility Your Answer

Note issuance facility

Select The Blank Question The international liquidity problem was tackled effectively by a reserve asset known as ________. Correct Answer Special drawing right Your Answer

Gold

Multiple Choice Single Answer

Question

As per article IV of IMF for fixed exchange rates member countries agreed to Correct Answer Change exchange rates only for correcting fundamental disequilibrium in external payments with IMF approval Your Answer Change exchange rates only for correcting fundamental disequilibrium in external payments with IMF approval Multiple Choice Multiple Answer Question As regards the due date of an Option exercise which methods are in vogue? Correct Answer American option , European Option Your Answer

American option , European Option , Exercise date

Multiple Choice Single Answer Question What was the collective assessment of International financial market about the country risk of many cnuntries in the last decade? Correct Answer Whatever it was, was disastrously wrong. Your Answer

Whatever it was, was disastrously wrong.

LIST OF ATTEMPTED QUESTIONS AND ANSWERS Select The Blank Question The counter party in option is ________. Correct Answer The exchange Your Answer The exchange Select The Blank Question An exchange rate for $ was48.2050, it is now 48.2075, then one says that it has moved ________ basis points Correct Answer 25 Your Answer 25 True/False Question Cross currency forward contract is not meaningful if original contract is in Rupees to USD Correct Answer True Your Answer True Multiple Choice Single Answer Question What do you understand by a swap? Correct Answer 2 counter parties agree to exchange streams of payments over a period of time Your Answer 2 counter parties agree to exchange streams of payments over a period of time True/False Question ECGC offers insurance protection to Indian exporters against payment risks. Correct Answer True Your Answer True Select The Blank Question Value of US Dollar is ________ in the world market but RBI is trying to match the Rupee value with it. Correct Answer Depreciating Your Answer Depreciating Multiple Choice Single Answer Question Financial management is complex process for Correct Answer Companies which have international trade Your Answer Companies which have international trade True/False Question The American Dollar market comprises of the domestic segment of USD the offshore segment of USD. Correct Answer True Your Answer True Multiple Choice Multiple Answer

Question What do you understand by 'Foreign Exchange Risk'? Correct Answer The change in value of currency alters the home currency value of an asset , liability, , The change in value of currency alters the home currency value of profit, loss, or future cash flows Your Answer The change in value of currency alters the home currency value of an asset , liability, , The change in value of currency alters the home currency value of profit, loss, or future cash flows Multiple Choice Single Answer Question Borrower from a developing country can borrow from US market Correct Answer Only if it has high credit rating and the country's Govt gives a guarantee for the same. Your Answer Only if it has high credit rating and the country's Govt gives a guarantee for the same. Multiple Choice Single Answer Question What was the purpose of reviewing FERA 1973 in the year 1993? Correct Answer Economic Liberalisation relating to Foreign trade and Foreign investments Your Answer Economic Liberalisation relating to Foreign trade and Foreign investments Select The Blank Question Legalised invoice is also called ________ invoice. Correct Answer Visaed Your Answer Visaed Multiple Choice Single Answer Question In Import financing through trust receipts, with whom does the ownership of goods remain? Correct Answer Bank releasing the import documents Your Answer Exporter Multiple Choice Single Answer Question Mr P has exported goods in May 2005 and is expecting payment of Euro 25000 by the last week of Sept 2005. Give details of the forward contract he can book :Correct Answer He can book a sale contract any time from May 2005 till Mid Sept 2005, for Euro 25000 or less, for maturity last week of Sept. Your Answer He can book a sale contract any time from May 2005 till Mid Sept 2005, for Euro 25000 or less, for maturity last week of Sept. True/False Question An Indian exporting goods to New York receives dollars in New York. Correct Answer True Your Answer True Multiple Choice Multiple Answer Question At the request of M/s Universal Traders Calcutta, State Bank of India opened a documentary credit in Favour of M/s Star Trading Co Dubai, for USD 25000/-.Here State Bank of India Calcutta is known as

Correct Answer Issuing Bank , Opening Bank Your Answer Issuing Bank , Opening Bank Multiple Choice Multiple Answer Question The integrated treasury branches of commercial banks handle which type of business? Correct Answer Forex transactions, Derivatives transactions , Mney market and bond market transactions Your Answer Forex transactions, Derivatives transactions , Mney market and bond market transactions Select The Blank Question For setting up joint ventures abroad or equity participation abroad, EXIM bank provides funds under ________ Scheme. Correct Answer Overseas investment facility Your Answer Overseas investment facility Match The Following Question Correct Answer Your Answer LastStage Form a single currency Form a single currency The First stage in the Europeand and Monetary Union (EMU) Free movement of capital in EC Free movement of capital in EC Stage II in EMU Establishment of European Monetary Institute Establishment of European Monetary Institute Stage III in EMU Members to meet the convergence criteria and fix the exchange rates Members to meet the convergence criteria and fix the exchange rates Multiple Choice Single Answer Question What do you call a 'decreasing principal swap'? Correct Answer Amortizing swap Your Answer Amortizing swap Multiple Choice Multiple Answer Question What does the word '"Tombstone" stand for? Correct Answer Advertisement which lists the managers, underwriters, , List of Providers of recently floated issue Your Answer Advertisement which lists the managers, underwriters, , List of Providers of recently floated issue Multiple Choice Single Answer Question Which of them are special funding facilities of IMF ? Correct Answer Buffer Stock Facility

Your Answer Buffer Stock Facility Multiple Choice Multiple Answer Question Mr A an Indian exporter is expecting to receive payment for exports in Hong kong dollar, after one month; How can he cover the exchange rate fluctuation risk? Correct Answer By booking a A forward contract in HKD itself , By booking a Cross currency option between USD and HKD and / or USD and Rs Your Answer By booking a A forward contract in HKD itself , By booking a Cross currency option between USD and HKD and / or USD and Rs True/False Question Bridge Loans are granted for construction of large size bridges, and flyovers. Correct Answer False Your Answer True True/False Question Integrated branches of Commercial banks handle forex transactions as well as options, swaps, futures, arbitrage operations Correct Answer True Your Answer True Match The Following Question

Correct Answer

Your Answer

Investment under Automatic route

Form ODA

Form ODA

Proposal under non-automatic route

RBI

RBI

Proposal under Normal route

Special Committee

Special Committee

Investment by partnerships not in core activity form ODI

form ODI

Multiple Choice Multiple Answer Question Name the services provided by 'Reuter 3000' in forex market:Correct Answer Quotation of forex rates, interest rates , Electronic broking by matching the rates Your Answer Quotation of forex rates, interest rates , Electronic broking by matching the rates Multiple Choice Single Answer Question Before abandoning the Fixed exchange rate system, the Dollar gold parity was fixed by USA at Correct Answer USD 35 per Ounce of gold Your Answer USD 35 per 10 grams of gold Select The Blank

Question After the Second World War many countries adopted the route of ________ to give facelift to sagging economies. Correct Answer Devaluation of their currencies Your Answer Devaluation of their currencies True/False Question Country risk analysis is a futile exercise in the background of cost of analysis. Correct Answer True Your Answer True Multiple Choice Multiple Answer Question Which factors affect the exchange rates in short time period? Correct Answer Demand and supply, interest rates, inflation, , Sentiment, Speculation Your Answer Demand and supply, interest rates, inflation, , Sentiment, Speculation , Literacy, Health of people Multiple Choice Single Answer Question A country needs external commercial borrowings to :Correct Answer Meet the adverse balance of payments position Your Answer Meet the adverse balance of payments position Multiple Choice Single Answer Question With whom can an exporter or importer book the forward contract? Correct Answer Authorised dealer/Authorised person Your Answer Authorised dealer/Authorised person Multiple Choice Multiple Answer Question World Bank gives loans for Correct Answer Structural Adjustments, Infrastructure projects, BOP problems Your Answer Structural Adjustments, Infrastructure projects, BOP problems Multiple Choice Multiple Answer Question Who can issue the Certificate of Inspection? Correct Answer Export Inspection Council, Person authorized by Importer. Your Answer Export Inspection Council, Person authorized by Importer. True/False Question In red clause credit some clauses are printed in Red ink. Correct Answer True Your Answer True True/False Question Out of 15 member countries in European Union 12 countries committed themselves for a single currency. Correct Answer False Your Answer False

Select The Blank Question FRNs are issued for a maturity period of ________ and are normally redeemed in one shot. Correct Answer 5-7 years Your Answer 5-7 years Multiple Choice Multiple Answer Question Criteria for investment abroad under automatic route :Correct Answer Activity for investment is core activity of Indian Company. , Total Financial commitment in JV/WOS as per prescribed ceiling Your Answer Activity for investment is core activity of Indian Company. , Total Financial commitment in JV/WOS as per prescribed ceiling Select The Blank Question ECGC provides cover to an Indian contractor for construction work abroad under ________. Correct Answer Construction works policy Your Answer Construction works policy Select The Blank Question A Credit can be a confirmed credit only if it is also ________. Correct Answer An Irrevocable credit Your Answer Redclause credit Multiple Choice Multiple Answer Question What is the criteria for an Indian Company for Direct Investment abroad? Correct Answer Activity proposed need to be core activity , Financial commitment in JV/WOS maximum $ 100 million or equivalent Your Answer Activity proposed need to be core activity , Financial commitment in JV/WOS maximum $ 100 million or equivalent , Indian co should not have been black listed Select The Blank Question A ________ Bill of lading does not specify the terms and conditions of carriage. Correct Answer Short Form Your Answer Copy of Multiple Choice Single Answer Question How does ECGC protect exporters against risks for export finance Correct Answer Issuing various insurance policies covering different risks connected with exports at nominal premium Your Answer Issuing various guarantees covering different risks connected with exports at nominal premium, True/False Question Sterling FRNs are issued in Europe, USA ,Asia

Correct Answer False Your Answer True Select The Blank Question Forward Margin makes a currency costlier then the currency is at ________. Correct Answer Premium Your Answer Premium Multiple Choice Multiple Answer Question Transaction exposures arises because of: Correct Answer The cost of settlement of future payment changes, Receipt denominated in a foreign currency varies due to exchange rate fluctuation Your Answer The cost of settlement of future payment changes, Receipt denominated in a foreign currency varies due to exchange rate fluctuation , Pricing of assets and liabilities in different countries

LIST OF ATTEMPTED QUESTIONS AND ANSWERS True/False Question

In a coupon swap the receiver is the one who receives fixed rate of interest. Correct Answer True Your Answer

True

Multiple Choice Multiple Answer Question The characteristics of an International documentary credit are :Correct Answer It is an undertaking given by the openers bank to pay certain sum of money , It is an undertaking given on behalf of a certain person to another person in different country Your Answer It is an undertaking given on behalf of a certain person to another person in different country , It is a stamped agreement and is not valid unless it is signed by the issuer , It is an undertaking given by the openers bank to pay certain sum of money True/False Question

Normally the forward contracts are booked for making huge profits. Correct Answer False Your Answer

True

Multiple Choice Single Answer Question What do you understand by a forex option? Correct Answer Right and not the obligation to take or give future delivery of an agreed quantity at certain price Your Answer Right and not the obligation to take or give future delivery of an agreed quantity at certain price Multiple Choice Single Answer Question In case the value date happens to be a holiday then the settlement is done :Correct Answer On the next working day Your Answer

On the next working day

True/False Question

Postal receipt is an acknowledgement of receipt of goods, a quasi negotiable instrument. Correct Answer False Your Answer

True/False

True

Question

Open Insurance policy is for any amount for specific period and specific number of shipments. Correct Answer False Your Answer

False

Multiple Choice Multiple Answer Question What schemes have been floated by RBI for facilitating Inter bank transactions? Correct Answer Clearing house for settlement and clearance for the operations of Ads , Set up a market intelligence cell to study and monitor developments in Forex Market Your Answer Clearing house for settlement and clearance for the operations of Ads , Set up a market intelligence cell to study and monitor developments in Forex Market Multiple Choice Single Answer Question Whose permission was necessary for forming the Association of Foreign Exchange Dealers, in India? Correct Answer Reserve Bank of India Your Answer

Reserve Bank of India

Multiple Choice Single Answer Question What was the collective assessment of International financial market about the country risk of many cnuntries in the last decade? Correct Answer Whatever it was, was disastrously wrong. Your Answer

Whatever it was, was disastrously wrong.

Select The Blank Question The international liquidity problem was tackled effectively by a reserve asset known as ________. Correct Answer Special drawing right Your Answer

Special drawing right

Select The Blank Question During the fixed rate exchange system prevailing in India, the intervention currency was ________. Correct Answer Sterling Pound Your Answer True/False Question

USDollar

Resident Individuals in India can acquire foreign bonus shares on the foreign securities already held by them without prior approval of RBI Correct Answer True

Your Answer

False

Multiple Choice Multiple Answer Question These are the parties to the documentary credit :Correct Answer Negotiating Bank, advising bank , Reimbursing bank, Confirming bank Your Answer Negotiating Bank, advising bank , Reimbursing bank, Confirming bank Multiple Choice Multiple Answer Question What are the functions of International Finance Corporation? Correct Answer Help private enterprise by financing projects , Provide technical assistance to private enterprise Your Answer Help private enterprise by financing projects , Provide technical assistance to private enterprise True/False Question

Both futures and options are traded on a public exchange.

Correct Answer False Your Answer

False

Multiple Choice Single Answer Question "Currency Snake" Correct Answer Plus or minus movement of exchange rate around the parity rate Your Answer

Plus or minus movement of exchange rate around the parity rate

Select The Blank Question "Flip-Flop', 'Mini-Max', 'Capped' 'Perpetual' are the various types of ________. Correct Answer Floating Rate Note (FRN) Your Answer

Floating Rate Note (FRN)

Multiple Choice Single Answer Question With whom can an exporter or importer book the forward contract? Correct Answer Authorised dealer/Authorised person Your Answer True/False Question

Authorised dealer/Authorised person

An Indian exporting goods to New York receives dollars in New York. Correct Answer True

Your Answer

False

Multiple Choice Multiple Answer Question Internationally only those currencies are in demand Correct Answer Which are convertible , Which have liquidity confirmed in Euro market operations Your Answer Which have liquidity confirmed in Euro market operations , Which are convertible Multiple Choice Single Answer Question What Preshipment facility is granted by EXIM Bank to an exporter? Correct Answer Preshipment credit beyond six months for construction and turnkey projects abroad Your Answer Preshipment credit beyond six months for construction and turnkey projects abroad True/False Question

The American Dollar market comprises of the domestic segment of USD the offshore segment of USD. Correct Answer True Your Answer

True

Multiple Choice Single Answer Question Who can open an EEFC account? Correct Answer Individuals , firms, companies, resident in India Your Answer

Individuals , firms, companies, resident in India

Select The Blank Question Over the years ________has been the most widely accepted currency in international capital market. Correct Answer American Dollar Your Answer

Euro Dollar

Multiple Choice Multiple Answer Question Under the liberalisation through FEMA, what can an AD offer to the clients, without the prior permission of RBI or Government? Correct Answer Hedges, interest rate swaps, currency swaps. , Caps and collars, and forward rate agreements Your Answer Hedges, interest rate swaps, currency swaps. , Caps and collars, and forward rate agreements True/False Question

Out of 15 member countries in European Union 12 countries commiteed themselves for a single currency. Correct Answer False

Your Answer

True

Select The Blank Question Global capital market can be understood from the study of ________. Correct Answer Yen,Euro Dollar, Swiss Francs, USD market, Govt Borrowings within and outside the country Your Answer IMF, World Bank and local markets Match The Following Question

Correct Answer

Your Answer

Investment by partnerships not in core activity

form ODI

form ODI

Investment under Automatic route

Form ODA

Form ODA

Proposal under non automatic route

RBI

Form NAR

Proposal under Normal route

Special Committee

OAB form

True/False Question

JV means Joint venture and WOS means World of Security.

Correct Answer False Your Answer

True

Select The Blank Question ________ is drawn by taking mean of the bid and offered rates. Correct Answer Middle rate Your Answer

Average rate

Match The Following Question

Correct Answer

Your Answer

IBRD

World Bank

World Bank

Collapse of IMF administered fixed exchange rates

Early 1970s

SDR

No International control on Exchange rates

Mid 80s

2002

Euro as a currency note/coin

2002

Mid 80s

Select The Blank Question An option on a future is ________. Correct Answer Derivative on a derivative Your Answer

Derivative on a derivative

Select The Blank Question Short term instruments normally issued by corporate borrowers, forhigh values, at a discount to face value are known as ________. Correct Answer Euro Commercial Papers Your Answer

FRNs

Multiple Choice Single Answer Question How does ECGC protect exporters against risks for export finance Correct Answer Issuing various insurance policies covering different risks connected with exports at nominal premium Your Answer Acting as agents for exporters and taking the entire risk of transit, recovery etc Multiple Choice Multiple Answer Question As regards the due date of an Option exercise which methods are in vogue? Correct Answer American option , European Option Your Answer

American option , European Option , Exercise date

Multiple Choice Multiple Answer Question Who can issue the Certificate of Inspection? Correct Answer Export Inspection Council , Person authorised by Importer. Your Answer

Export Inspection Council , Person authorised by Importer.

Multiple Choice Multiple Answer Question What were the primary objectives of the group of seven countries? Correct Answer Bring down the external value of US Dollar , Bring some stability to exchange rates of all the countries Your Answer Bring down the external value of US Dollar , Bring some stability to exchange rates of all the countries Multiple Choice Single Answer Question What is a bill of lading? Correct Answer Transport document representing movement of goods by water. Your Answer

Transport document representing movement of goods by water.

Multiple Choice Multiple Answer Question Why is Bill of Exchange a Quasi Negotiable Instrument? Correct Answer It represents title to goods , It is issued under Sale of Goods act Your Answer

It represents title to goods , It is issued under Sale of Goods act

Multiple Choice Single Answer Question Mr P has exported goods in May 2005 and is expecting payment of Euro 25000 by the last week of Sept 2005. Give details of the forward contract he can book :Correct Answer He can book a sale contract any time from May 2005 till Mid Sept 2005, for Euro 25000 or less, for maturity last week of Sept. Your Answer He can book a sale contract any time from May 2005 till Mid Sept 2005, for Euro 25000 or less, for maturity last week of Sept. Multiple Choice Single Answer Question Financial management is complex process for Correct Answer Companies which have international trade Your Answer

Companies which have international trade

Multiple Choice Single Answer Question What is the objective of INCOTERMS? Correct Answer Arrange for transfer of risk from seller to buyer at a convenient place Your Answer Arrange for transfer of risk from seller to buyer at a convenient place Multiple Choice Multiple Answer Question What do you understand by 'Foreign Exchange Risk'? Correct Answer The change in value of currency alters the home currency value of an asset , liability, , The change in value of currency alters the home currency value of profit, loss, or future cash flows Your Answer The change in value of currency alters the home currency value of an asset , liability, , The change in value of currency alters the home currency value of profit, loss, or future cash flows Multiple Choice Multiple Answer Question The various arms of World Bank are: Correct Answer International Development Agency , Multinational Investment Guarantee Agency Your Answer International Development Agency , Multinational Investment Guarantee Agency , Asian Development Bank Multiple Choice Multiple Answer Question Which of these are considered as miscellaneous documents?

Correct Answer Packing list , Weight certificate Your Answer

Packing list , Weight certificate

Select The Blank Question ECGC provides cover to an Indian contractor for construction work abroad under ________. Correct Answer Construction works policy Your Answer

Construction works policy

Ât cellSpacing=0 cellPadding=0 width="100%" border=0>  Select The Blank   Question ÂThe stock of foreign reserves in India, is now around ________ % of the GDP and exceeds money circulation.

  Correct Answer 15    Your Answer 20 Â

  Multiple Choice Multiple Answer   Question ÂThe World Bank Group Consists of

  Correct Answer International  Development Association , International finance Corporation , Multinational Investment Guarantee Agency

  Your Answer International  finance Corporation , Multinational Insurance Guarantee Agency , International Development Association

  Multiple Choice Single Answer   Question ÂWhat do you call a 'decreasing principal swap'?   Correct Answer Amortizing  swap   Your Answer Amortizing  swap   Multiple Choice Single Answer   Question ÂWho can open an EEFC account?

  Correct Answer Individuals  , firms, companies, resident in India

  Your Answer Individuals  , firms, companies, resident in India   Select The Blank   Question ÂEUROCLEAR, CEDEL are the names of ________ related with the issue of GDR.

  Correct Answer Registrars    Your Answer Registrars    True/False   Question ÂAn Indian importing goods from USA pays dollars in India.   Correct Answer False    Your Answer True    Multiple Choice Multiple Answer   Question ÂHow does a company use swap to restructure debt, in its balance sheet?   Correct Answer By resorting  to floating rate from fixed rate , By resorting to fixed rate from floating rate

  Your Answer By resorting  to floating rate from fixed rate , By resorting to fixed rate from floating rate

  Multiple Choice Single Answer   Question ÂHow many Countries participated in the third survey on Foreign Exchange, coordinated by BIS in 1992?

  Correct Answer 26    Your Answer 26    True/False   Question ÂIn red clause credit some clauses are printed in Red ink.   Correct Answer True Â

  Your Answer False Â

  Select The Blank   Question ÂThe rate of interest on rupee finance for exports is not exceeding ________ less than PLR.

  Correct Answer 0.025    Your Answer 50 basis  points over LIBOR   Multiple Choice Multiple Answer   Question ÂFor issuing a Documentary credit The applicant is required to provide to the issuing bank details about

  Correct Answer The purchase-sales  contract, the mode of payment, the tenor of payment , The documents required, the names of authorities who would issue the documents

  Your Answer TheÂpurchase-sales contract, the mode of payment, the tenor of payment , The track record of the buyer and seller

  Multiple Choice Single Answer   Question ÂExchange Rate means

  Correct Answer Value  of one currency in terms of another   Your Answer Value  of one currency in terms of another   Multiple Choice Single Answer

  Question ÂWhat do you understand by 'bill of lading in a quasi negotiable instrument'?

  Correct Answer It is a document  of title to goods and its negotiation may not be complete and free from qualification.

  Your Answer It isÂa document of title to goods and its negotiation may not be complete and free from qualification.

  Multiple Choice Single Answer   Question ÂWhere does the word derivative originate from?   Correct Answer Mathematics    Your Answer Mathematics    Multiple Choice Multiple Answer   Question ÂWhat options does the merchant have while booking a forward contract?   Correct Answer Delivery  on a Fixed date, within six months , Delivery on a date between two days separated by a period of maximum one month

  Your Answer Delivery  on a Fixed date, within six months , Delivery on a date between two days separated by a period of maximum one month , A day within next 1 year

  Select The Blank   Question ÂEEFC account holder is allowed to have ________ facility.

  Correct Answer Cheque  book and nomination

  Your Answer Cheque  book and nomination   True/False   Question ÂA small percentage of futures contracts actually result in delivery of the underlying commodity or security.

  Correct Answer True    Your Answer True    True/False   Question ÂLong in futures means a commitment to sell futures at a future date.

  Correct Answer False    Your Answer True    Multiple Choice Multiple Answer   Question ÂWhat are the categories of exposures, a person faces in International trade?

  Correct Answer Transaction  exposure , Translation exposure   Your Answer Transaction  exposure , Translation exposure   True/False

  Question ÂNaked position is when investor does not hedge against market risk.

  Correct Answer True    Your Answer True    Multiple Choice Multiple Answer   Question ÂThe participants in the Forex Markets are :-

  Correct Answer Non banking  entities wishing to exchange currencies, , Banks and their clients , Speculators, arbitragers

  Your Answer NonÂbanking entities wishing to exchange currencies, , Banks and their clients , Speculators, arbitragers

  Select The Blank   Question Â________ is the acid test for events to be included in country risk.

  Correct Answer Control  by Government'   Your Answer Predictability Â

  Multiple Choice Multiple Answer   Question ÂCriteria for Direct investment abroad under normal route :  Correct Answer Viability  of the proposal, and excellent track record of business , Experience and expertise of promoters, benefit to the country

  Your Answer Viability  of the proposal, and excellent track record of business , Experience and expertise of promoters, benefit to the country

Â

 Match The Following Question

Correct Answer

Your Answer

16 principles for good environmental conduct World Chambers Congress ICC Conference in 2004 13 types of trade contracts

Business Charter of sustainable development World Federation of Chambers Marrakesh

Business Charter of sustainable development

INCOTERM

INCOTERM

World Federation of Chambers Marrakesh

  True/False   Question ÂPP Form is required when exports are made to any country by post parcel on value payable basis.

  Correct Answer False    Your Answer True Â

  Multiple Choice Multiple Answer   Question ÂThe investor is in the money when :-

  Correct Answer The exercise  price is below the spot price for a call option , The exercise price is above the spot price for a put option

  Your Answer TheÂexercise price is above the spot price for a put option , The exercise price is below the spot price for a put option

  Multiple Choice Single Answer   Question ÂUnder which category of exchange rate did the Indian Rupee came from June 92?

  Correct Answer Other  managed Floating

  Your Answer Independently  floating

  Multiple Choice Single Answer   Question ÂFinancial management is complex process for   Correct Answer Companies  which have international trade   Your Answer Cooperative  societies abroad   Select The Blank   Question ÂThe Trans European Automated Real Time Gross Settlement Express Transfer is popular in ________.

  Correct Answer Brussels    Your Answer Frankfurt    Multiple Choice Multiple Answer   Question ÂWhat is the duty of the AD while booking a forward contract?   Correct Answer Verify  the suitable documentary evidence, and that the customer is exposed to risk , Ensure that a genuine transaction exists

  Your Answer Verify  the suitable documentary evidence, and that the customer is exposed to risk , Ensure that a genuine transaction exists , Protect the customer fully from exchange risk

  Select The Blank

  Question ÂEven after liberalisation of exchange controls, the Euro sterling market remained volatile because of uncertainties in UK and gyrations of sterling against major ________ countries

  Correct Answer OECD   Your Answer OECD    Multiple Choice Single Answer   Question ÂWith whom can an exporter or importer book the forward contract?

  Correct Answer Authorised  dealer/Authorised person

  Your Answer AnyÂbank in India   Select The Blank

  Question ÂTo make the export more competitive and remunerative, the Indian Govt extends ________ facility.   Correct Answer Cash Incentives    Your Answer LineÂof credit   Multiple Choice Single Answer   Question ÂPunjab National Bank Mumbai Branch quoted USD 1= Rs 50.5000/52.5050. Which is the bid rate for USD?

  Correct Answer USD 1=Rs  50.5000   Your Answer USD  1=Rs 50.5000 + margin   True/False

  Question ÂThe Euro Commercial paper is supported by a team-syndicated credit with both sides of the transaction combined into one operation.

  Correct Answer True    Your Answer True    Multiple Choice Multiple Answer   Question ÂAs regards the due date of an Option exercise which methods are in vogue?

  Correct Answer American  option , European Option   Your Answer American  option , European Option   Select The Blank   Question ÂIn mid 1992 ________ rejected the ratification of Maastricht treaty

  Correct Answer Denmark    Your Answer UK    Multiple Choice Single Answer   Question ÂHow does ECGC protect exporters against risks for export finance

  Correct Answer Issuing various insurance policies covering different risks connected with exports at nominal premium

  Your Answer Issuing  various guarantees covering different risks connected with exports at nominal premium,

  Match The Following Question Correct Answer

Your Answer

Swap price

Difference in Difference in prices of prices of two two legs of buying and legs of selling buying and selling Euro Market Market in Market in Euro Euro currencies currencies Making market Quoting rate Quoting rate for both for both buying and selling a buying and currency selling a currency Spread Price Price difference difference between bid and offer between bid and offer   True/False   Question ÂCountry risk analysis is a futile exercise in the background of cost of analysis.   Correct Answer True    Your Answer False    Multiple Choice Single Answer   Question ÂWhat is the volume of the world's average daily transactions in foreign exchange?   Correct Answer USD 1Âtrillion   Your Answer USD  1 trillion   Multiple Choice Multiple Answer   Question ÂIn DAF, the frontier means :  Correct Answer Frontier  of exporter's country , Frontier of importer's country   Your Answer Frontier  of exporter's country , Frontier of importer's country   True/False   Question ÂIn an interest rate swap if the payment schedules are identical, only diffrence between the two payments is delivered.

  Correct Answer True    Your Answer True    Select The Blank

  Question ÂThe option which can be exercised on the maturity date and not before it is known as________.

  Correct Answer European  Option   Your Answer European  Option   True/False

  Question ÂSandby credis are introduced by US Banks because they were not permitted to issue on demand guarantees.

  Correct Answer True    Your Answer True    Multiple Choice Multiple Answer   Question ÂAt the request of M/s Universal Traders Calcutta, State Bank of India opened a documentary credit in Favour of M/s Star Trading Co Dubai, for USD 25000/-.Here State Bank of India Calcutta is known as

  Correct Answer Issuing Bank , Opening Bank   Your Answer Issuing  Bank , Confirming Bank

  Select The Blank   Question ÂUnder the ________ standard the Country's Central Bank promised to buy back the currency at certain rate of Gold.

  Correct Answer Gold    Your Gold Answer  Multiple Choice Multiple Answer Âoice Multiple Answer   Question ÂThe World Bank Group Consists of

Â

  Correct Answer International  Development Association , International finance Corporation , Multinational Investment Guarantee Agency   Your Answer International  Development Association , International finance Corporation , Multinational Investment Guarantee Agency   Multiple Choice Single Answer   Question ÂEquity can be raised internationallly by a company through :  Correct Answer Fund raising,  underwriting, hedging , arbitration operations   Your Answer Floating  debt instruments Â

 Match The Following

Question Swap price

Euro Market

Correct Difference in prices of two legs of buying and selling Market in Euro currencies

Making market Quoting rate for both buying and selling a currency Spread Price difference between bid and offer

Your Answer Difference in prices of two legs of buying and selling

Market in Euro currencies Quoting rate for both buying and selling a currency

Price difference between bid and offer

  True/False   Question ÂIn an interest rate swap if the payment schedules are identical, only diffrence between the two payments is delivered.   Correct Answer True    Your Answer True Â

  Select The Blank   Question ÂThe large flow of money internationally was caused primarily by the oil price hikes engineered by ________.   Correct Answer OPECÂcountries   Your Answer European  Countries   Select The Blank   Question Â________ enables bankers to extend purchase, negotiation or discount of export bills.   Correct Answer Post shipment  credit guarantee   Your Answer Export  finance guarantee

  Multiple Choice Multiple Answer   Question ÂWhich non financial service does EXIM Bank give to an exporter?   Correct Answer Provide  Technical and administrtive support , Planning, promoting and developing export oriented Industries   Your Answer Provide  Technical and administrtive support , Planning, promoting and developing export oriented Industries   Multiple Choice Multiple Answer   Question ÂWhich debits are permitted to EEFC accounts?   Correct Answer Payment  of hotel bills abroad, payment for expenses on Heart operation abroad , Payment to EOUs in India for services rendered by them   Your Answer Payment  of hotel bills abroad, payment for expenses on Heart operation abroad , Payment to EOUs in India for services rendered by them   True/False   Question ÂTerms in an Irrevocable credit can not be alterred at all.   Correct Answer False    Your Answer True    Multiple Choice Single Answer   Question ÂHow many INCOTERMS are there in "Departure" Group?

  Correct Answer One Â

  Your Answer OneÂ

  Multiple Choice Single Answer   Question ÂHow does ICC access Govt and influence Governmental decisions?   Correct Answer Through  national committees   Your Answer Through  national committees   Multiple Choice Multiple Answer   Question ÂA Usance bill of exchange may be drawn in 2 ways. Which are they?   Correct Answer Documents  against payment , Documents against acceptance   Your Answer Documents  against payment , Documents against acceptance   Multiple Choice Single Answer   Question ÂName the single most important factor which may devastate the economic viability of an operation in international trade :  Correct Answer Unfavourable  fluctuation in the exchange rate   Your Answer Unfavourable  fluctuation in the exchange rate   True/False   Question ÂThe Euro Commercial paper is supported by a team-syndicated credit with both sides of the transaction combined into one operation.   Correct Answer True    Your Answer True    Select The Blank   Question ÂWhen interest rate changes by 0.05 per cent then it has changed ________ basis points.   Correct Answer 50 Â

  Your Answer 50 Â

  Select The Blank

  Question ÂIndia is a founder member of ________.   Correct Answer IMF    Your Answer BANKNET    Multiple Choice Single Answer   Question ÂHow many types of documents are required in the International trade?   Correct Answer Six    Your Answer Six    Multiple Choice Multiple Answer   Question ÂExport finance   Correct Answer Preshipment  credit , Post shipment credit , Duty Draw back   Your Answer Preshipment  credit , Post shipment credit , Buyers credit   Select The Blank   Question ÂWhen a credit is made to available to additional beneficiary wholly or partly it is called as ________.   Correct Answer Transferable  credit   Your Answer Transferable  credit   True/False   Question ÂA small percentage of futures contracts actually result in delivery of the underlying commodity or security.   Correct Answer True    Your Answer True    Select The Blank   Question ÂAn exchange rate for $ was48.2050, it is now 48.2075, then one says that it has moved ________ basis points   Correct Answer 25    Your Answer 0.0025    True/False   Question ÂTransfer of funds for investment abroad is allowed freely.   Correct Answer True Â

  Your Answer True    Select The Blank

  Question ÂThe rate of interest on rupee finance for exports is not exceeding ________ less than PLR.   Correct Answer 0.025    Your Answer 0.025    Multiple Choice Multiple Answer   Question ÂWhat do you understand by 'Foreign Exchange Risk'?   Correct Answer The change  in value of currency alters the home currency value of an asset , liability, , The change in value of currency alters the home currency value of profit, loss, or future cash flows   Your Answer Certainty  of an event which would happen in favour of the importer/ exporter , Inability to control various factors contributing to International trade   Multiple Choice Single Answer   Question ÂFinancial management is complex process for   Correct Answer Companies  which have international trade   Your Answer Companies  which have international trade   True/False   Question ÂImport finance is nomally available at rates of interest cheaper than domestic finance rates.   Correct Answer False    Your Answer False    Select The Blank   Question ÂThe stock of foreign reserves in India, is now around ________ % of the GDP and exceeds money circulation.   Correct Answer 15    Your Answer 12.5   Multiple Choice Single Answer   Question ÂWhich type of Convertibility of Rupee launched in March 1992 ?   Correct Answer Limited  convertibility   Your Answer Convertibility  on current account

  Multiple Choice Multiple Answer

  Question ÂHow does a company use swap to restructure debt, in its balance sheet?   Correct Answer By resorting  to floating rate from fixed rate , By resorting to fixed rate from floating rate   Your Answer By resorting  to fixed rate from floating rate , Prepayment of existing debts , Raising new loans Â

 Multiple Choice Single Answer   Question ÂPunjab National Bank Mumbai Branch quoted USD 1= Rs 50.5000/52.5050. Which is the offered rate for Rs?   Correct Answer USD 1=Rs  50.5000   Your Answer USD  1=Rs 50.5000 + margin   Multiple Choice Multiple Answer

  Question ÂWhat facilities are proposed by Sodhani Committee for NRIs?   Correct Answer Housing  Loan to NRI staff of Indian Companies, , NRIs be permitted to invest in MMMF   Your Answer Housing  Loan to NRI staff of Indian Companies, , NRIs be permitted to invest in MMMF , Permission to sale shares under Port folio investment be denied   Multiple Choice Multiple Answer   Question ÂWhich document is used for transportation by air, rail, road, marine?   Correct Answer Multimodal  transport document , Combined Transport document   Your Answer Multimodal  transport document , Combined Transport document Â

 Multiple Choice Single Answer   Question ÂUnder which contracts the seller pays the import duty for the buyer?

  Correct Answer DDP    Your Answer DDU    True/False   Question ÂDealing room is a place where the actual exchange of currencies takes place phsically at predetermined rates.   Correct Answer False    Your Answer True Â

  Multiple Choice Single Answer   Question ÂWhat was the reason for launching swap as an risk hedging instrument?   Correct Answer To avoid  the effects of currency control imposed in UK   Your Answer To avoid  the effects of currency control imposed in UK   Select The Blank   Question ÂThe creditworthiness of futures exchange is maintained through imposition of ________.   Correct Answer Margin   Your Answer Margin    True/False   Question ÂIn forward sales, the delivery refers to actual cash flows on due date.   Correct Answer True    Your Answer True    Multiple Choice Multiple Answer   Question ÂIn a documentary credit the word document means :  Correct Answer Specified  documents of title, transportation, insurance , Specified certificates, and other papers required as per the regulations in the buyers country

  Your Answer Specified  documents of title, transportation, insurance , Specified certificates, and other papers required as per the regulations in the buyers country , Credit is documented as an evidence   True/False   Question ÂThe amount in EEFC account can be deposited in FCNR account to get interest.   Correct Answer False    Your Answer True    True/False   Question ÂADR Level I, is the final step to enter US erquity market.   Correct Answer False    Your Answer False    Multiple Choice Multiple Answer   Question ÂAs per BIS, recovery from the recent crisis in global capital market was mainly due to   Correct Answer Rapid Âand appropriate policy response in US and other countries , Increase in Foreign Direct Investments , free flow of credit   Your Answer Rapid  and appropriate policy response in US and other countries , Increase in Foreign Direct Investments , free flow of credit , Devaluation of currencies   Multiple Choice Multiple Answer   Question ÂWhat is the effect of cross currency option on the customer?   Correct Answer He gets  the flexibility to exposure , He is exposed to additional risk of movement of rates between 3 currencies   Your Answer He  gets the flexibility to exposure , His profit increases   Multiple Choice Single Answer   Question ÂIn case the value date happens to be a holiday then the settlement is done :  Correct Answer On theÂnext working day

  Your Answer On  the next working day   Select The Blank   Question ÂAfter the second world war many countries adopted the route of ________ to give facelift to sagging economies.   Correct Answer Devaluation  of their currencies   Your Answer Devaluation  of their currencies   Select The Blank   Question Â________Bill of lading must specify the name of the ship.   Correct Answer "On Board"    Your Answer "OnÂBoard"   Multiple Choice Single Answer   Question ÂWhose permission was necessary for forming the Association of Foreign Exchange Dealers, in India?   Correct Answer Reserve  Bank of India   Your Answer Reserve  Bank of India   Match The Following Question Correct Your Answer Answer European On or after On or after maturity option maturity Out of money Loss Loss In the money Profit Profit American Before Before maturity option maturity

Â

Multiple Choice Multiple Answer Âoice Multiple Answer   Question The ÂWorld Bank Group Consists of

  CorrectInternational Answer  Development   Your Answer International  Development Association , International finance Corporation , Multinational Investment Guarantee Agency   Multiple Choice Single Answer   Question Equity  can be raised internationallly by a company through :  CorrectFund Answer raising,  underwriting, hedging , arbitration operations

  Your Answer FloatingÂdebt instruments

  Multiple Choice Single Answer   Question Financial  management is complex process for   CorrectCompanies Answer Âwhich have international trade   Your Answer Companies  which have international trade   True/False   Question Import  finance is nomally available at rates of interest cheaper than domestic finance rates.   CorrectFalse Answer Â

  Your Answer False Â

  Select The Blank   Question The Âstock of foreign reserves in India, is now around ________ % of the GDP and exceeds money circulation.   Correct15 Answer    Your Answer 12.5    Multiple Choice Single Answer   Question Which  type of Convertibility of Rupee launched in March 1992 ?   CorrectLimited Answer convertibility    Your Answer Convertibility  on current account   Multiple Choice Multiple Answer   Question HowÂdoes a company use swap to restructure debt, in its balance sheet?   CorrectBy Answer resorting  to floating rate from fixed rate , By resorting to fixed rate from floating rate

  Your Answer By resorting  to fixed rate from floating rate , Prepayment of existing debts , Raising new loans

  Multiple Choice Single Answer   Question Punjab  National Bank Mumbai Branch quoted USD 1= Rs 50.5000/52.5050. Which is the offered rate for Rs?

  CorrectUSD Answer 1=RsÂ50.5000   Your Answer USD 1=Rs  50.5000 + margin   Multiple Choice Multiple Answer   Question What  facilities are proposed by Sodhani Committee for NRIs?   CorrectHousing AnswerÂLoan  to NRI staff of Indian Companies, , NRIs be permitted to invest in MMMF

  Your Answer HousingÂLoan to NRI staff of Indian Companies, , NRIs be permitted to invest in MMMF , Permission to sale shares under Port folio investment be denied   Multiple Choice Multiple Answer   Question Which  document is used for transportation by air, rail, road, marine?

  CorrectMultimodal Answer Âtransport document , Combined Transport document   Your Answer Multimodal  transport document , Combined Transport document   Multiple Choice Single Answer   Question Under  which contracts the seller pays the import duty for the buyer?

  CorrectDDP Answer Â

  Your Answer DDU Â

  True/False   Question Dealing  room is a place where the actual exchange of currencies takes place phsically at predetermined rates.   CorrectFalse Answer Â

  Your Answer True Â

  Multiple Choice Single Answer

  Question What  was the reason for launching swap as an risk hedging instrument?

  CorrectTo Answer avoid the  effects of currency control imposed in UK

  Your Answer To avoidÂthe effects of currency control imposed in UK

  Select The Blank   Question The Âcreditworthiness of futures exchange is maintained through imposition of ________.   CorrectMargin Answer    Your Answer Margin    True/False   Question In forward  sales, the delivery refers to actual cash flows on due date.   CorrectTrue Answer Â

  Your Answer True Â

  Multiple Choice Multiple Answer   Question In a  documentary credit the word document means :-

  CorrectSpecified AnswerÂdocuments  of title, transportation, insurance , Specified certificates, and other papers required as per the regulations in the buyers country

  Your Specified documents of title, Answer transportation, insurance ,  Multiple Choice Single Answer ÂChoice Single Answer   Question In which  of the funding facilities the repayment period is between 3 to 5 years ?

  CorrectC.C.F.F. Answer    Your Answer Stand By  Arrangement   Multiple Choice Single Answer   Question Samuraibonds  are

  CorrectForeign Answer yen  bonds issued to non resident entities in Japanese Market   Your Answer Foreign  yen bonds issued to non resident entities in Japanese Market   Match The Following Question Correct Answer Your Answer

Returning RFC NRI

FCNR, NRE

NRI Deposits

INVESTM ENT IN MMMF RFC Investmen t in GDR, ADR

FCNR, NRE

NRIS AND INVESTMENT IN MMMF EXPORT EEFC ERS

 Ât cellSpacing=0 cellPadding=0 width="100%" border=0>

 Multiple Choice Single Answer   Question In which  of the funding facilities the repayment period is between 3 to 5 years ?   CorrectC.C.F.F. Answer    Your Answer Stand By  Arrangement Â

 Multiple Choice Single Answer

  Question Samuraibonds  are

  CorrectForeign Answer yen  bonds issued to non resident entities in Japanese Market   Your Answer Foreign  yen bonds issued to non resident entities in Japanese Market Â

 Multiple Choice Single Answer   Question In the  context of International economy ,the period between 1920s and 1930s is known as   CorrectGreat Answer Depression    Your Answer Great Depression  Â

 Multiple Choice Multiple Answer

  Question Sterling  Market is attractive for an international investor because

  CorrectGilts Answer offer higher  attractive margin , Choice of issuing medium term and long term bonds   Your Answer Gilts offer  higher attractive margin , Choice of issuing medium term and long term bonds , Linking of issues to currency swaps Â

 Select The Blank

  Question When  interest rate changes by 0.05 per cent then it has changed ________ basis points.   Correct50 Answer Â

  Your Answer 500 Â

  Multiple Choice Multiple Answer   Question Internationally  only those currencies are in demand

  CorrectWhich Answer are  convertible , Which have liquidity confirmed in Euro market operations   Your Answer Which are  convertible , Which have liquidity confirmed in Euro market operations , Which have greater intrinsic value   Select The Blank   Question AfterÂthe second world war many countries adopted the route of ________ to give facelift to sagging economies.   CorrectDevaluation Answer  of their currencies

  Your Answer Devaluation  of their currencies

  Multiple Choice Multiple Answer   Question HowÂdoes RBI control Exports?

  CorrectThrough AnswerÂRule  like "Full value of Export bills must be realised in 180 days," , Through Rule like " Export must be denominated In a currency of the Importing country or a freely convertible currency"

  Your Answer ThroughÂRule like "Full value of Export bills must be realised in 180 days," , Through Rule like " Export must be denominated In a currency of the Importing country or a freely convertible currency" , By Issuing a Export Code Number, and monitoring his exports through returns and statements filed by him   Select The Blank   Question OverÂthe years ________has been the most widely accepted currency in international capital market.   CorrectAmerican Answer Dollar Â

  Your Answer American  Dollar   Select The Blank   Question Any  company involved in foreign exchange transactions is exposed to ________ risk.

  CorrectCurrency Answer    Your Answer Payment   True/False   Question An AD  can book a forward contract for a part of the amount of Sale Contract   CorrectTrue Answer Â

  Your Answer True Â

  Multiple Choice Single Answer

  Question WhyÂdoes a person need protection from unfavourable movement of exchange rate?

  CorrectUnfavourable Answer  fluctuations in exchange rate can wipe out the normal business profit   Your Answer He wants  to make profit   Match The Following Question Correct Answer Your Answer

Merchant Portfolio management banking Securitizat Prime Traditional banking service

Issue of debt in the form of Highest credit rating Loans and cash credit against security

Portfolio managem ent Issue of Highest Loans and cash credit against security

Â

 Multiple Choice Single Answer   Question A Specific  date forward contract can be booked so as to be :  CorrectPayable Answer onÂa specific date of the calendar month   Your Answer PayableÂon a specific date of the calendar month Â

 Multiple Choice Multiple Answer   Question What  are the sources of raising funds for exports?   CorrectSuppliers AnswerÂcredit  , Forfaiting   Your Answer Suppliers  credit , Forfaiting , Buyers credit

Â

 Select The Blank   Question The Âcounter party in option is ________.   CorrectThe Answer exchange    Your Answer Trustworthy  corporation, who hedges the risks of market uncertainty Â

 True/False   Question Transfer  of funds for investment abroad is allowed freely.   CorrectTrue Answer    Your Answer True  Â

 Multiple Choice Single Answer   Question What  do you understand by a swap?   Correct2Answer counter parties  agree to exchange streams of payments over a period of time   Your Answer 2 counter  parties agree to exchange streams of payments over a period of time Â

 True/False

  Question The ÂOption period in the forward contract is at the option of the Banker, hence the bank has to be very conservative in quoting rates.

  CorrectFalse Answer    Your Answer True  Â

 Multiple Choice Single Answer

  Question What  are the powers of money changers?

  CorrectPurchase Answer and  sell foreign currency notes and travellers cheques

  Your Answer Purchase  and sell foreign currency notes and travellers cheques

  Select The Blank   Question ________  is drawn by taking mean of the bid and offered rates.

  CorrectMiddle Answer rate   Your Answer Basic rate    True/False   Question The Âcredit rating of World Bank is high in the world Capital market because it operates at the debt equity ratio of 1:2   CorrectFalse Answer    Your Answer True    Multiple Choice Multiple Answer   Question What  are the activities of ICC?   CorrectAribtration Answer  and dispute resolving , Canvass for open trade combat commercial crime   Your Answer Aribtration  and dispute resolving , Canvass for open trade combat commercial crime   Multiple Choice Single Answer   Question Whose  permission was necessary for forming the Association of Foreign Exchange Dealers, in India?

  CorrectReserve AnswerÂBank  of India   Your Answer ReserveÂBank of India   Match The Following

Question

Correct Answer

Returning RFC NRI FCNR, NRE NRIS AND INVESTMENT IN MMMF EXPORT EEFC ERS

Your Answer

FCNR, INVESTM RFC Investmen t in GDR, ADR

Â

 True/False   Question Tranche'  is a managed issue of securities in smaller denominations.   CorrectTrue Answer    Your Answer True  Â

 Select The Blank   Question A Documentary  credit issued on the strength of another documentary credit is known as ________.   CorrectBack Answer to Back  credit   Your Answer Back to  Back credit Â

 Multiple Choice Single Answer   Question Which  is the most actively traded currency in the international market today?   CorrectUS Answer Dollar    Your Answer US Dollar  Â

 True/False

  Question Till August  1994 India was following the Indirect Methodof Rate quotation.

  CorrectFalse Answer    Your Answer True  Â

 Multiple Choice Multiple Answer   Question Which  debits are permitted to EEFC accounts?   CorrectPayment AnswerÂofÂhotel bills abroad, payment for expenses on Heart operation abroad , Payment to EOUs in India for services rendered by them   Your Answer Payment of hotel bills abroad, payment for expenses on Heart operation abroad , Payment to EOUs in India for services rendered by them , Investment abroad in stocks and shares by an Individual   True/False

  Question Normally  the forward contracts are booked for making huge profits.   CorrectFalse Answer Â

  Your Answer True Â

  Multiple Choice Multiple Answer   Question Which  factors influence the valuation of cash flows?

  CorrectIncomplete Answer Âcapital market , Availability of projkect specific concessional finance,   Your Answer Incomplete  capital market , Rates of inflation, , volatility of exchange rates

  True/False   Question Indian  companies are now allowed to freely invest abroad up to $1.5 million, raised through GDE or EEFC without prior permission from RBI.   CorrectFalse Answer    Your Answer True    Multiple Choice Multiple Answer   Question Name  the services provided by 'Reuter 3000' in forex market:  CorrectQuotation Answer of  forex rates, interest rates , Electronic broking by matching the rates   Your Answer Quotation  of forex rates, interest rates , Electronic broking by matching the rates , Calculate and quote rates for transactions   Multiple Choice Multiple Answer   Question Underwriters  are more attracted to NIF issue because of :  CorrectLesser Answer risks  , Increased returns , Easy marketability features

  Your Answer Lesser risks  , Increased returns , High underwriting fees

  Multiple Choice Multiple Answer   Question In which  cases can a Resident Individual in India acquire foreign securities without prior approval?   CorrectGift Answer from aÂperson outside India, or Inheritance from a person resident in or outside India , Security issued under Cashless employees stock option scheme   Your Answer Gift fromÂa person outside India, or Inheritance from a person resident in or outside India , Security issued under Cashless employees stock option scheme , Purchased from NRO/NRNR accounts   Select The Blank   Question A ________  Bill of lading does not specify the terms and conditions of carriage.   CorrectShort Answer Form   Your Answer Dirty    Multiple Choice Multiple Answer   Question What  does a speculator accept while dealing with an option?

  CorrectThe Answer exercise  or strike price, the amount , The expiration date and time, the method of payment   Your Answer The exercise  or strike price, the amount , The expiration date and time, the method of payment   Multiple Choice Single Answer   Question For a  spot transaction carried out on Thursday, currencies will be exchanged on :  CorrectMonday Answer    Your Answer Friday    Multiple Choice Single Answer   Question Inhow  many groups are INCOTERMS classified?   CorrectFour Answer    Your Answer Four    Select The Blank   Question In ________  transaction the delivery is postponed from the agreed date.

  CorrectRollover Answer    Your Answer Future    True/False   Question Phytosanitary/  Radiation/Fumigation certificates are in the category of Health Certificates

  CorrectTrue Answer    Your Answer True    Multiple Choice Multiple Answer   Question What  is the objective of FEDAI?

  CorrectLaying Answer down  terms and conditions of exchange business between Customers and Bankers , Interbank understanding for transacting forex business and exchange rates   Your Answer Laying down  terms and conditions of exchange business between Customers and Bankers , Growth of Indian Exports , Ensure the profitability of Forex business to the banks   True/False

  Question Out  of 15 member countries in European Union 12 countries commiteed themselves for a single currency.   CorrectFalse Answer    Your Answer True    Select The Blank   Question For setting  up joint ventures abroad or equity participation abroad, EXIM bank provides funds under ________ Scheme.   CorrectOverseas Answer investment  facility   Your Answer Equity Participation  scheme   Multiple Choice Multiple Answer   Question HowÂis the entry into any conuntry's money market or capital market is restricted?   CorrectBecause AnswerÂofÂdifferent tax systems , Because of different credit standards   Your Answer BecauseÂof different tax systems , Because of different credit standards , Because of resistance by dealer's associations   Select The Blank   Question A Bill of exchange payable 45 days from date of shipment, is a means of ________ from seller to the buyer.   CorrectCredit Answer    Your Payment  Select The Blank Â>Select The Blank

  Question Forward  margin makes a currency cheaper then the currency is at ________.   CorrectDiscount Answer    Your Answer PAR    Multiple Choice Multiple Answer

  Question Which  of these are not considered as good securities by the banker?   CorrectHouse Answer Airway  bill , Postal Receipt   Your Answer Postal Receipt  , House Airway bill   Multiple Choice Single Answer   Question The ÂDirect Investment abroad through share swap arrangement under normal route is permitted in a :  CorrectJV/WOS Answer    Your Answer JV/WOS Â

 Multiple Choice Single Answer

  Question WhoÂintroduced the worlds first exchange traded currency futures?   CorrectChicago AnswerÂMercantile  Exchange   Your Answer ChicagoÂMercantile Exchange   Multiple Choice Multiple Answer   Question WithÂreference to ECB, fund raising activity includes :-

  CorrectFloating Answer rate  notes, bonds, debentures , Syndicate loans, issue of shares

  Your Answer FloatingÂrate notes, bonds, debentures , Syndicate loans, issue of shares   Select The Blank   Question An option  on a future is ________.   CorrectDerivative Answer  on a derivative   Your Answer Irrelevant    Multiple Choice Single Answer   Question HowÂdoes ICC help poor countries?   CorrectAttract Answer Foreign  Direct investments togeather with UNCTAD   Your Answer Attract Foreign  Direct investments togeather with UNCTAD   Multiple Choice Multiple Answer

  Question Which  functions does the Bill of Lading serve?   CorrectItAnswer is a document  of title to goods. , It is receipt of goods   Your Answer It is a document  of title to goods. , It is receipt of goods   Match The Following Question Correct Answer Stage III in Members to meet the

Your Members

LastStage The First stage in the European d and Monetary Union (EMU) Stage II in EMU

Form a single currency Free movement of capital in EC

Free Establish ment of European Monetary Institute

Establishment of European Monetary Institute

Establish EURO Bank

Â

 Multiple Choice Single Answer

  Question For a  spot transaction carried out on Tuesday, where both centers have holiday on Wednesday, currencies will be exchanged on :  CorrectThursday AnswerÂifÂit is a working day at both the centers   Your Answer Thursday  if it is a working day at both the centers

  Match The Following Question Correct Answer Intolerable Drop the investment project risk

Country risk Stock market analysis Weightag es %

Cost of evluation does not justify analysis Banks are ahead

Factors' values

Your Cost of evluation does not justify analysis Continue the project Banks are ahead Factors' values

  True/False   Question Bid rate  is bidders buying rate and offered rate is bidders selling rate.   CorrectTrue Answer    Your Answer True  Â

 True/False   Question Resident  Individuals in India can acquire foreign bonus shares on the foreign securities already held by them without prior approval of RBI   CorrectTrue Answer    Your Answer True    Multiple Choice Multiple Answer   Question What  do you understand by Commercial Risk?

  CorrectProtracted Answer Âdefault by the buyer , Insolvency of the buyer   Your Answer Protracted  default by the buyer , Insolvency of the buyer , Risk of strikes due to which commercial activity is haulted   Multiple Choice Single Answer   Question A billÂof exchange issued by a bank which pays specified sum of money to the holder on a specific date is called :  CorrectBankers' AnswerÂAcceptance  Bond   Your Answer Usance ÂBill of Exchange   Select The Blank   Question Association  of several scheduled commercial banks for forex business is ________.   CorrectForeign Answer Exchange  Dealers Association of India   Your Answer Foreign  Exchange Dealers Association of India   Select The Blank   Question The Âmajor industrial power whose economy was not affected after the II world war was ________.   CorrectUSA Answer    Your Answer USA    True/False   Question Sterling  FRNs are issued in Europe, USA ,Asia   CorrectFalse Answer Â

  Your Answer True Â

  Multiple Choice Single Answer   Question Mr M has imported some goods from Greece. What is the maximum period within which he must make the payment?   CorrectSix Answer monthsÂfrom date of shipment   Your Answer Six months  from date of bill of exchange   Multiple Choice Single Answer   Question Under  the automatic route of Direct Investment Abroad, what is the restriction on market purchases?   Correct50% Answer of theÂnet worth of the investing company   Your Answer 50% of the  net worth of the investing company   Select The Blank   Question For importing  goods in India RBI insists on ________ value.   CorrectCIF Answer Â

  Your Answer FOB Â

  Multiple Choice Single Answer   Question What  is the objective of INCOTERMS?   CorrectArrange Answer forÂtransfer of risk from seller to buyer at a convenient place

  Your Answer ArrangeÂfor transfer of risk from seller to buyer at a convenient place   Multiple Choice Single Answer   Question Name  the members of Exchange Banks association :  CorrectForeign Answer Banks  at Bombay, Calcutta, Delhi, Madras, Amritsar,

  Your Answer Foreign  Banks at Bombay, Calcutta, Delhi, Madras, Amritsar,   True/False   Question Yankee  bond is issued for domestic borrowers.

  CorrectFalse Answer    Your Answer True    True/False   Question In the  forward rate quotations between two currencies, when one currency is at discount the other currency is at premium.   CorrectTrue Answer    Your Answer False    Select The Blank   Question Fixed  interest rate Eurobond without the option to convert to equity is known as ________.   CorrectStraight Answer Debt    Your Answer Straight ÂDebt   True/False   Question Cause  of increase in cost of public issues Underwriting fees , selling commission   CorrectTrue Answer    Your Answer True    Multiple Choice Single Answer   Question In the  syndicated loan arrangement the borrower has to execute :  CorrectSingle Answer Principal  loan agreement   Your Answer Single Principal  loan agreement   Multiple Choice Multiple Answer   Question What  criteria of lending is adopted by The International Development agency while giving loans to a country?   CorrectThe Answer per capita  income of the borrwoing country , The population of the borrwoing country   Your Answer The per Âcapita income of the borrwoing country , The repayment capacity. , The Security offerred.   Select The Blank   Question As aÂresult of the world wars in the 20th century the ________ of most of the countries suffered heavily.   CorrectEconomies Answer Â

  Your Answer Currencies    Select The Blank   Question Rupee  was liberalised on ________ account in 1993, and on current account in 1994.   CorrectTrade Answer    Your Answer Capital    True/False   Question Integrated  branches of Commercial banks handle forex transactions as well as options, swaps, futures, arbitrage operations   CorrectTrue Answer    Your Answer True    True/False   Question Floating  rate notes are traded in secondary market also.   CorrectTrue Answer    Your Answer True    Multiple Choice Single Answer   Question What  maxim is followed by the forex dealer when indirect method of exchange rate quotation is used?   CorrectBuy Answer Low, sell  High   Your Answer Buy High,  Sell Low   Select The Blank   Question In ________  Contract seller is like a domestic seller in destination country   CorrectDDP Answer    Your Answer DDP    Select The Blank   Question ________Bill  of lading must specify the name of the ship.   Correct"On Answer Board"   Your Answer "On Board"    Select The Blank   Question Indian  Co. intending to make investment under Automatic route needs to submit form ________ to RBI with enclosures.   CorrectODA Answer    Your Answer ODA    Multiple Choice Multiple Answer   Question What  is a 'speculator'?   CorrectTrader Answer whoÂis neither hedger nor arbitragor , He takes risks for profit   Your Answer Trader who  is neither hedger nor arbitragor , He takes risks for profit , He buys or sells to hedge his risk Â

 Multiple Choice Multiple Answer   Question The Âcharacteristics of IDA Loan are:   CorrectItAnswer gives loans  to poorer countries , The rate of interest is zero percent , The repayment period is very long 40 years   Your Answer It gives loans  to poorer countries , The rate of interest is zero percent , The repayment period is very long 40 years   True/False   Question Head  office of ICC is in Paris because of its First President Eliance Clementel.   CorrectTrue Answer    Your Answer True    Multiple Choice Multiple Answer   Question What  does the word '"Tombstone" stand for?   CorrectAdvertisement Answer  which lists the managers, underwriters, , List of Providers of recently floated issue   Your Answer Advertisement  which lists the managers, underwriters, , List of Providers of recently floated issue , Issue without planning, for short duration   Multiple Choice Single Answer   Question What  is Risk Syndication facility?   CorrectSharing Answer of  risk of non payment by supplier   Your Answer Sharing Âof risk of non payment by supplier   Multiple Choice Multiple Answer   Question What  do you understand by 'Foreign Exchange Risk'?   CorrectThe Answer change  in value of currency alters the home currency value of an asset , liability, , The change in value of currency alters the home currency value of profit, loss, or future cash flows   Your Answer The change  in value of currency alters the home currency value of an asset , liability, , The change in value of currency alters the home currency value of profit, loss, or future cash flows   Multiple Choice Multiple Answer   Question The Âmaximum period for which a forward contract can be granted by the AD depends upon which RBI directives?   CorrectMaximum Answer Period  of repatriation of export proceeds , Maximum Period of payment for Imports   Your Answer Maximum  Period of repatriation of export proceeds , Maximum Period of payment for Imports Â

 Multiple Choice Multiple Answer   Question A Forward  purchase contract was booked with an authorised dealer and subsequently cancelled .It :  CorrectCan Answer be booked  again with another dealer , Can be booked again with the same dealer   Your Answer Can be booked  again with another dealer , Can be booked again with the same dealer , Can be booked only for un booked portion of the contract.   True/False   Question BothÂfutures and options are traded on a public exchange.   CorrectFalse Answer    Your Answer False    Multiple Choice Multiple Answer   Question The Âcharacteristics of an International documentary credit are :  CorrectItAnswer is an undertaking  given by the openers bank to pay certain sum of money , It is an undertaking given on behalf of a certain person to another person in different country   Your It is an undertaking given by the openers Answer bank to pay certain sum of money , It is an  undertaking given on behalf of a certain person to another person in different country , It is an unconditional undertaking Â

LIST OF ATTEMPTED QUESTIONS AND ANSWERS Multiple Choice Multiple Answer Question What are the features of Settlement price ? Correct Answer Price established by the clearing house at the end of each trading session , It is used to determine the net gain, loss, margin call and next days price limits Your Answer Price established by the clearing house at the end of each trading session , It is used to determine the net gain, loss, margin call and next days price limits Select The Blank Question Legalised invoice is also called ________ invoice. Correct Answer Visaed Your Answer

Visaed

True/False Question

The working of the IBRD and IMF in recent has blurred the difference between the two Correct Answer True Your Answer

False

Multiple Choice Multiple Answer Question Under the liberalisation through FEMA, what can an AD offer to the clients, without the prior permission of RBI or Government? Correct Answer Hedges, interest rate swaps, currency swaps. , Caps and collars, and forward rate agreements Your Answer Hedges, interest rate swaps, currency swaps. , Caps and collars, and forward rate agreements , Good and competitive exchange rates and cross currency rates Select The Blank Question ________Bill of lading must specify the name of the ship. Correct Answer "On Board" Your Answer

"On Board"

Multiple Choice Single Answer Question What is the immediate cause for the growth of futures and options in the last two decades? Correct Answer Increase in volatile trading conditions

Your Answer

General market investments

Multiple Choice Single Answer Question In the context of International economy ,the period between 1920s and 1930s is known as Correct Answer Great Depression Your Answer

Great Depression

Match The Following Question

Correct Answer

Your Answer

Departure

EXW

EXW

Main carriage paid

CFR

CFR

Main carriage unpaid

FAS

FAS

Arrival

DES

DES

Multiple Choice Single Answer Question What is the statutory requirement about receivables of exports from India? Correct Answer The export receivables must be repatriated within six months of the date of shipment. Your Answer The export receivables must be repatriated within six months of the date of shipment. True/False Question

An exporter expecting payment in$ next month will book a forward purchase contract. Correct Answer False Your Answer

True

Multiple Choice Multiple Answer Question The characteristics of IDA Loan are: Correct Answer It gives loans to poorer countries , The rate of interest is zero percent , The repayment period is very long 40 years Your Answer It gives loans to poorer countries , The rate of interest is zero percent , The repayment period is very long 40 years Multiple Choice Multiple Answer Question What is the duty of the AD while booking a forward contract?

Correct Answer Verify the suitable documentary evidence, and that the customer is exposed to risk , Ensure that a genuine transaction exists Your Answer Verify the suitable documentary evidence, and that the customer is exposed to risk , Ensure that a genuine transaction exists , Protect the customer fully from exchange risk Multiple Choice Multiple Answer Question Criteria for Direct investment abroad under normal route :Correct Answer Viability of the proposal, and excellent track record of business , Experience and expertise of promoters, benefit to the country Your Answer Viability of the proposal, and excellent track record of business , Experience and expertise of promoters, benefit to the country , Total Financial commitment in JV/WOS as per prescribed ceiling Multiple Choice Single Answer Question When was the first currency swap introduced in world market? Correct Answer 1970 Your Answer

1970

True/False Question

Integrated branches of Commercial banks handle forex transactions as well as options, swaps, futures, arbitrage operations Correct Answer True Your Answer

True

Multiple Choice Multiple Answer Question Who can issue the Certificate of Inspection? Correct Answer Export Inspection Council , Person authorised by Importer. Your Answer

Export Inspection Council , Person authorised by Importer. , Reserve Bank of India

Match The Following Question

Correct Answer

Your Answer

Cancellation of contracts above USD 500000

ECD

FEDAI

Forward Purchase contract currency at premium

Presume earliest delivery Presume earliest delivery

Forward purchase contract currency at discount

Presume latest delivery

Presume latest delivery

Exchange margins

True/False Question

FEDAI

ECD

Both futures and options are traded on a public exchange.

Correct Answer False Your Answer

True

Multiple Choice Multiple Answer Question Internationally only those currencies are in demand Correct Answer Which are convertible , Which have liquidity confirmed in Euro market operations Your Answer Which are convertible , Which have liquidity confirmed in Euro market operations , Which have greater intrinsic value Multiple Choice Single Answer Question What was the official foreign currency reserve of the group of seven central Banks taken togeather in mid 1992? Correct Answer USD 275 Billion Your Answer

GBP 300 billion

Multiple Choice Single Answer Question A Bank quotes interest rate of 9.50/10.00 for six months dollar deposit, here the bid rate is :Correct Answer 9.50% P.A. Your Answer

9.50 divided by 6 =1.58% P.A.

True/False Question

JV means Joint venture and WOS means World of Security.

Correct Answer False Your Answer

True

Multiple Choice Multiple Answer Question How is the entry into any conuntry's money market or capital market is restricted? Correct Answer Because of different tax systems , Because of different credit standards Your Answer Because of different tax systems , Because of different credit standards , Because of protectionism True/False

Question

The American Dollar market comprises of the domestic segment of USD the offshore segment of USD. Correct Answer True Your Answer

True

Multiple Choice Multiple Answer Question The major cost components of NIF are :Correct Answer Underwriters fees , Margins of notes Your Answer

Underwriters fees , Documentation charges , Issue registration fees

True/False Question

FRN's issued with ceiling on interest beyond which the issues is not required to pay interest is known as Mini-Max FRN Correct Answer True Your Answer

False

Multiple Choice Single Answer Question A weight list, a packing list used in international trade are which type of documents? Correct Answer Operational Documents Your Answer

Optional documents

Select The Blank Question The credit which does not constitute a legally binding undertaking between the bank and the beneficiary is ________. Correct Answer Revocable Credit Your Answer

Revocable Credit

Multiple Choice Multiple Answer Question Sterling Market is attractive for an international investor because Correct Answer Gilts offer higher attractive margin , Choice of issuing medium term and long term bonds Your Answer Gilts offer higher attractive margin , Choice of issuing medium term and long term bonds , Linking of issues to currency swaps Multiple Choice Single Answer Question European currency Unit Correct Answer Basket of Currencies of members of EMS

Your Answer

Basket of Currencies of members of EMS

True/False Question

At Least one principal Lender bank is essential for issue of the ADR or GDR. Correct Answer False Your Answer

True

True/False Question

In an already booked forward contract, a person may seek early delivery, extension in period, or cancellation. Correct Answer True Your Answer

True

Multiple Choice Single Answer Question A Run on gold gold by a country means Correct Answer Exchange of reserve currency in to gold Your Answer

Exchange of gold into Reserve Currency

Multiple Choice Single Answer Question Punjab National Bank Mumbai Branch quoted USD 1= Rs 50.5000/52.5050. Which is the bid rate for USD? Correct Answer USD 1=Rs 50.5000 Your Answer

USD 1=Rs 50.5000

Multiple Choice Single Answer Question The Direct Investment abroad through share swap arrangement under normal route is permitted in a :Correct Answer JV/WOS Your Answer

JV/WOS

Select The Blank Question FRNs are issued for a maturity period of ________ and are normally redeemed in one shot. Correct Answer 5-7 years Your Answer True/False

5-7 years

Question

Bid rate is bidders buying rate and offered rate is bidders selling rate. Correct Answer True Your Answer

True

True/False Question

Corporates are allowed to raise funds abroad up to $ 15 million as short term finance if they have EPCG license. Correct Answer True Your Answer

False

Select The Blank Question Global capital market can be understood from the study of ________. Correct Answer Yen,Euro Dollar, Swiss Francs, USD market, Govt Borrowings within and outside the country Your Answer IMF, World Bank and local markets Select The Blank Question Short term instruments normally issued by corporate borrowers, forhigh values, at a discount to face value are known as ________. Correct Answer Euro Commercial Papers Your Answer

Euro Commercial Papers

Select The Blank Question Even after liberalisation of exchange controls, the Euro sterling market remained volatile because of uncertainties in UK and gyrations of sterling against major ________ countries Correct Answer OECD Your Answer

OECD

Multiple Choice Multiple Answer Question For issuing a Documentary credit The applicant is required to provide to the issuing bank details about Correct Answer The purchase-sales contract, the mode of payment, the tenor of payment , The documents required, the names of authorities who would issue the documents Your Answer The purchase-sales contract, the mode of payment, the tenor of payment , The documents required, the names of authorities who would issue the documents , Authority who will issue the documents Multiple Choice Multiple Answer

Question

The crisis in global capital market in 2002-04 was due to

Correct Answer Collapse of Enron , Collapse of Argentina's currency board , The war against terrorism Your Answer Collapse of Enron , Collapse of Argentina's currency board , The war against terrorism Select The Blank Question Over the years ________has been the most widely accepted currency in international capital market. Correct Answer American Dollar Your Answer

American Dollar

Select The Blank Question "Flip-Flop', 'Mini-Max', 'Capped' 'Perpetual' are the various types of ________. Correct Answer Floating Rate Note (FRN) Your Answer

Floating Rate Note (FRN)

Select The Blank Question Combined Certificate of Origin and Value are required by ________. Correct Answer Commonwealth Countries Your Answer

Commonwealth Countries

Multiple Choice Single Answer Question Bank with lowest Bid for fees etc and which undertakes the responsibility of loan syndication is called :Correct Answer The lead bank- leadmanager- arranger Your Answer

The lead bank- leadmanager- arranger

LIST OF ATTEMPTED QUESTIONS AND ANSWERS Multiple Choice Multiple Answer Question Which functions does the Bill of Lading serve? Correct Answer It is a document of title to goods. , It is receipt of goods Your Answer It is a document of title to goods. , It is receipt of goods Multiple Choice Single Answer Question A Bank quotes interest rate of 9.50/10.00 for six months dollar deposit, here the bid rate is :Correct Answer 9.50% P.A. Your Answer 9.50% P.A.

Multiple Choice Single Answer Question What is Sovereign Risk? Correct Answer Risk of lending to the government of a country Your Answer Risk of lending to the government of a country Multiple Choice Single Answer Question How does ICC help poor countries? Correct Answer Attract Foreign Direct investments togeather with UNCTAD Your Answer Attract Foreign Direct investments togeather with UNCTAD Multiple Choice Multiple Answer Question As per BIS, recovery from the recent crisis in global capital market was mainly due to Correct Answer Rapid and appropriate policy response in US and other countries , Increase in Foreign Direct Investments , free flow of credit Your Answer Rapid and appropriate policy response in US and other countries , Increase in Foreign Direct Investments , free flow of credit True/False Question Out of 15 member countries in European Union 12 countries commiteed themselves for a single currency. Correct Answer False Your Answer True True/False Question In red clause credit some clauses are printed in Red ink. Correct Answer True Your Answer True True/False Question Transfer of funds for investment abroad is allowed freely. Correct Answer True Your Answer False Select The Blank Question EEFC account holder is allowed to have ________ facility. Correct Answer Cheque book and nomination Your Answer Credit card, safe custody, safe deposit locker without any fee Select The Blank Question Fixed interest rate Eurobond without the option to convert to equity is known as ________. Correct Answer Straight Debt Your Answer Interest rate swap True/False Question An Indian importing goods from USA pays dollars in India. Correct Answer False Your Answer True Match The Following Question Correct Answer Your Answer PP Form Export Declaration form Export Declaration form Health Certificate Fumigation certificate Export

Declaration form Certificate of Inspection As per importers requirement Export Declaration form Certifiate of Origin Export Promotion Council Export Declaration form Multiple Choice Multiple Answer Question For issuing a Documentary credit The applicant is required to provide to the issuing bank details about Correct Answer The purchase-sales contract, the mode of payment, the tenor of payment , The documents required, the names of authorities who would issue the documents Your Answer The purchase-sales contract, the mode of payment, the tenor of payment , The track record of the buyer and seller Multiple Choice Multiple Answer Question The characteristics of an International documentary credit are :Correct Answer It is an undertaking given by the openers bank to pay certain sum of money , It is an undertaking given on behalf of a certain person to another person in different country Your Answer It is an undertaking given on behalf of a certain person to another person in different country , It is a stamped agreement and is not valid unless it is signed by the issuer True/False Question CFR = FOB plus freight paid by seller +insurance paid by seller. Correct Answer False Your Answer True Select The Blank Question Even after liberalisation of exchange controls, the Euro sterling market remained volatile because of uncertainties in UK and gyrations of sterling against major ________ countries Correct Answer OECD Your Answer OPEC True/False Question Integrated branches of Commercial banks handle forex transactions as well as options, swaps, futures, arbitrage operations Correct Answer True Your Answer True Multiple Choice Single Answer Question Which of them are special funding facilities of IMF ? Correct Answer Buffer Stock Facility Your Answer Supplemental Reserve Policy Multiple Choice Multiple Answer Question What are the sources of raising funds for exports? Correct Answer Suppliers credit , Forfaiting Your Answer Suppliers credit , Forfaiting , Buyers credit , Foreign currency or Rupee finance

True/False Question Normally the forward contracts are booked for making huge profits.

Correct Answer False Your Answer False True/False Question Head office of ICC is in Paris because of its First President Eliance Clementel. Correct Answer True Your Answer False Select The Blank Question A ________ Bill of lading does not specify the terms and conditions of carriage. Correct Answer Short Form Your Answer Short Form Multiple Choice Single Answer Question What is the volume of the world's total speculative transactions? Correct Answer Roughly 90 % of the total turnover Your Answer 70% of the total turnover True/False Question Till August 1994 India was following the Indirect Methodof Rate quotation. Correct Answer False Your Answer False Select The Blank Question For setting up joint ventures abroad or equity participation abroad, EXIM bank provides funds under ________ Scheme. Correct Answer Overseas investment facility Your Answer Overseas investment facility Multiple Choice Single Answer Question Mr P has exported goods in May 2005 and is expecting payment of Euro 25000 by the last week of Sept 2005. Give details of the forward contract he can book :Correct Answer He can book a sale contract any time from May 2005 till Mid Sept 2005, for Euro 25000 or less, for maturity last week of Sept. Your Answer He can book a contract for Forward Purchase, any time from May 2005 till Mid Sept 2005, for Euro 25000 or less, for maturity last week of Sept. Select The Blank Question In US Capital Market a Company prepares all the necessary documents in advance of the public issue under the facility known as ________. Correct Answer Shelf Registration Your Answer Shell Registration Multiple Choice Multiple Answer Question Which common details will be available in any bill of Lading? Correct Answer Names of consigner, consignee , The port of loading and unloading, contract of shipment Your Answer Names of consigner, consignee , The port of loading and unloading, contract of shipment , The name of the vessel , The date of transportation and contract of transportation Select The Blank Question Under the ________ standard the Country's Central Bank promised to buy back the currency at certain rate of Gold. Correct Answer Gold Your Answer Interest

Select The Blank Question Short term instruments normally issued by corporate borrowers, forhigh values, at a discount to face value are known as ________. Correct Answer Euro Commercial Papers Your Answer ADRs Select The Blank Question The type of trade where a country sells orange juice in return for cars is known as ________. Correct Answer Counter Trade Your Answer Non money trade Multiple Choice Multiple Answer Question Which debits are permitted to EEFC accounts? Correct Answer Payment of hotel bills abroad, payment for expenses on Heart operation abroad , Payment to EOUs in India for services rendered by them Your Answer Payment to EOUs in India for services rendered by them , Payment of Gifts of GBP 5000 per year per donor Multiple Choice Multiple Answer Question Under the liberalisation through FEMA, what can an AD offer to the clients, without the prior permission of RBI or Government? Correct Answer Hedges, interest rate swaps, currency swaps. , Caps and collars, and forward rate agreements Your Answer Caps and collars, and forward rate agreements , Good and competitive exchange rates and cross currency rates Multiple Choice Multiple Answer Question Sterling Market is attractive for an international investor because Correct Answer Gilts offer higher attractive margin , Choice of issuing medium term and long term bonds Your Answer Choice of issuing medium term and long term bonds , Less controls and less formalities to be completed , Linking of issues to currency swaps Multiple Choice Single Answer Question Under which contracts the seller pays the import duty for the buyer? Correct Answer DDP Your Answer DDP True/False Question PP Form is required when exports are made to any country by post parcel on value payable basis. Correct Answer False Your Answer True Multiple Choice Multiple Answer Question What were the immediate causes of converting the group of 5 into group of 7 and convene their meeting in late 1985? Correct Answer Many Industries in USA had become uncompetitive with imports , In USA the Trade protectionnism had increased Your Answer Many Industries in USA had become uncompetitive with imports , In USA the Trade protectionnism had increased , The heads of G5 insisted to have a meeting , Failure of IMF to control the exchange rates

Multiple Choice Single Answer Question What are the powers of money changers? Correct Answer Purchase and sell foreign currency notes and travellers cheques Your Answer Purchase and sell foreign currency notes and travellers cheques, give import and export finance Multiple Choice Single Answer Question Who can open an EEFC account? Correct Answer Individuals , firms, companies, resident in India Your Answer Individuals , firms, companies, resident in India Multiple Choice Single Answer Question What is the role of ICC towards WTO? Correct Answer To provide world business recommendations to WTO Your Answer To further the cause of WTO Select The Blank Question Position in a currency is said to be ________, when sales equal purchases. Correct Answer Square Your Answer Awkward Select The Blank Question Hotels and Airline companies are allowed to receive payments in Indian rupees from foreign tourist without insisiting on ________. Correct Answer Encashment certificate Your Answer Encashment certificate Multiple Choice Multiple Answer Question Why is Bill of Exchange a Quasi Negotiable Instrument? Correct Answer It represents title to goods , It is issued under Sale of Goods act Your Answer It represents title to goods , It is issued under Transportation act , It is issued under Bill of exchange act Multiple Choice Single Answer Question Which details about forward contracts do the Exchange Controls verify ? Correct Answer Particulars of cancellation of forward cover for the equivalent of USD 500,000 Your Answer Details of all contracts booked and cancelled beyond a specified limit Multiple Choice Single Answer Question In the context of International economy,the period between 1920s and 1930s is known as Correct Answer Great Depression Your Answer Great Depression Multiple Choice Multiple Answer Question What do you understand by Commercial Risk? Correct Answer Protracted default by the buyer, Insolvency of the buyer Your Answer Protracted default by the buyer, Insolvency of the buyer

LIST OF ATTEMPTED QUESTIONS AND ANSWERS Multiple Choice Single Answer Question What is the statutory requirement about receivables of exports from India? Correct Answer The export receivables must be repatriated within six months of the date of shipment. Your Answer The export receivables must be repatriated within six months of the date of shipment. Match The Following Question

Correct Answer

Your Answer

Maturity of future

Time when actual commodity is delivered

Last date of exercising option

Note issuance facility

Euronotes

Facility of issuing bonds

Bearer security issued in Eurobonds currency other than that of the country of issue

Eurobonds

Margin on future

Peformance bond or security deposit

Peformance bond or security deposit

Select The Blank Question When an Indian Company earns at least 50% of its average turnover from a particular activity then that activity is considered as ________. Correct Answer Core activity Your Answer

Core activity

True/False Question

In developed markets, speculators play important role in determining the trend of currency rates. Correct Answer True Your Answer

True

True/False Question

Cash flows generated from aforeign project may replace revenueproducing imports to the host country Correct Answer False Your Answer

False

Select The Blank

Question

In US Capital Market a Company prepares all the necessary documents in advance of the public issue under the facility known as ________. Correct Answer Shelf Registration Your Answer

Shelf Registration

True/False Question

Cross currency forward contract is not meaningful if original contract is in Rupees to USD Correct Answer True Your Answer

True

Multiple Choice Single Answer Question What maxim is followed by the forex dealer when direct method of exchange rate quotation is used? Correct Answer Buy High, Sell Low Your Answer

Buy Low, sell High

True/False Question

Sterling FRNs are issued in Europe, USA ,Asia

Correct Answer False Your Answer

False

Multiple Choice Single Answer Question Under which category of exchange rate did the Indian Rupee came from June 92? Correct Answer Other managed Floating Your Answer

Other managed Floating

Multiple Choice Single Answer Question In which currency can the EEFC account be opened? Correct Answer Any Convertible currency Your Answer

Any Convertible currency

Multiple Choice Single Answer Question When is intermediation resorted to? Correct Answer When The Lender does not know or trust the creditwothiness of the borrower Your Answer When The Lender does not know or trust the creditwothiness of

the borrower Multiple Choice Multiple Answer Question What is the effect of cross currency option on the customer? Correct Answer He gets the flexibility to exposure , He is exposed to additional risk of movement of rates between 3 currencies Your Answer He gets the flexibility to exposure , He is exposed to additional risk of movement of rates between 3 currencies Multiple Choice Single Answer Question How is the exporter paid through 'import loan'? Correct Answer The Bank delivers goods to the importer against his 'trustee' receipt, exporter sells the goods and then pays for the same Your Answer The Bank delivers goods to the importer against his 'trustee' receipt, exporter sells the goods and then pays for the same True/False Question

The working of the IBRD and IMF in recent has blurred the difference between the two Correct Answer True Your Answer

True

Multiple Choice Multiple Answer Question Mr A an Indian exporter is expecting to receive payment for exports in Hong kong dollar, after one month; How can he cover the exchange rate fluctuation risk? Correct Answer By booking a A forward contract in HKD itself , By booking a Cross currency option between USD and HKD and / or USD and Rs Your Answer By booking a A forward contract in HKD itself , By booking a Cross currency option between USD and HKD and / or USD and Rs Match The Following Question

Correct Answer

Your Answer

Forfaiting

Discounting export receivables

Discounting export receivables

Buyer's Credit

Available to Buyer abroad Available to Buyer abroad

Seller's Credit

Available to seller abroad

Available to seller abroad

Usance LC

Seller paid only on maturity date

Seller paid only on maturity date

Multiple Choice Single Answer Question Which currency is traded in one leg of about 70% of the world's transactions? Correct Answer US Dollar Your Answer

US Dollar

Select The Blank Question After the second world war many countries adopted the route of ________ to give facelift to sagging economies. Correct Answer Devaluation of their currencies Your Answer

Devaluation of their currencies

Multiple Choice Single Answer Question Which is the most actively traded currency in the international market today? Correct Answer US Dollar Your Answer

EURO

Multiple Choice Multiple Answer Question The integrated treasury branches of commercial banks handle which type of business? Correct Answer Forex transactions, Derivatives transactions , Mney market and bond market transactions Your Answer Forex transactions, Derivatives transactions , Mney market and bond market transactions , International loans, and borrowings Multiple Choice Multiple Answer Question The participants in the Forex Markets are :Correct Answer Non banking entities wishing to exchange currencies, , Banks and their clients , Speculators, arbitragers Your Answer Non banking entities wishing to exchange currencies, , Banks and their clients , Speculators, arbitragers Multiple Choice Multiple Answer Question Name the services provided by 'Reuter 3000' in forex market:Correct Answer Quotation of forex rates, interest rates , Electronic broking by matching the rates Your Answer Quotation of forex rates, interest rates , Electronic broking by matching the rates , Demat securities Multiple Choice Single Answer Question What is the volume of the world's average daily transactions in foreign exchange?

Correct Answer USD 1 trillion Your Answer

USD 1 trillion

Select The Blank Question EUROCLEAR, CEDEL are the names of ________ related with the issue of GDR. Correct Answer Registrars Your Answer

Registrars

Select The Blank Question Under the ________ standard the Country's Central Bank promised to buy back the currency at certain rate of Gold. Correct Answer Gold Your Answer

Gold

True/False Question

Multimodal transport document is also known as "IntermobileTransport Document" Correct Answer True Your Answer

True

Multiple Choice Multiple Answer Question Export finance Correct Answer Preshipment credit , Post shipment credit , Duty Draw back Your Answer

Preshipment credit , Post shipment credit , Duty Draw back

Select The Blank Question Value of USDollar is ________ in the world market but RBI is trying to match the Rupee value with it. Correct Answer Depreciating Your Answer

Depreciating

Multiple Choice Single Answer Question Why did Govt of India initially exercise controls on foreign receipts and payments? Correct Answer Conserve and mobilise foreign exchange for war effort Your Answer

Conserve and mobilise foreign exchange for war effort

Multiple Choice Single Answer

Question

What do firms face while evaluating risks in cross border investment? Correct Answer The issues are different, unique to each project, and solutions are varied Your Answer The issues are different, unique to each project, and solutions are varied Select The Blank Question EEFC account holder is allowed to have ________ facility. Correct Answer Cheque book and nomination Your Answer

Cheque book and nomination

True/False Question

A small percentage of futures contracts actually result in delivery of the underlying commodity or security. Correct Answer True Your Answer

False

True/False Question

The Option period in the forward contract is at the option of the Banker, hence the bank has to be very conservative in quoting rates. Correct Answer False Your Answer

True

Multiple Choice Multiple Answer Question Which general risks are covered by any insurance policy for transit? Correct Answer Theft, loss , Damage, sea perils, non delivery Your Answer

Theft, loss , Damage, sea perils, non delivery

Multiple Choice Multiple Answer Question Mr A sells an option conferring the right to buy a kilogram of silver at Rs 8000/- for next six months means :Correct Answer The buyer of option can buy silver from Mr A at 8000/- per Kg, in next 6 months, even if the price goes up, , The buyer of option can sell silver to Mr A at 8000/- per Kg, in next 6 months, even if the price goes down. Your Answer The buyer of option can buy silver from Mr A at 8000/- per Kg, in next 6 months, even if the price goes up, , The buyer of option can sell silver to Mr A at 8000/- per Kg, in next 6 months, even if the price goes down.

Select The Blank Question In mid 1992 ________ rejected the ratification of Maastricht treaty Correct Answer Denmark Your Answer

Denmark

Multiple Choice Multiple Answer Question What non funded facilities are granted by Export Import Bank of India? Correct Answer Guarantee for Bid bond, execution of export project, for borrowings abroad, for retention of money , Advance payment Guarantee, performance guarantee Your Answer Guarantee for Bid bond, execution of export project, for borrowings abroad, for retention of money , Advance payment Guarantee, performance guarantee Select The Blank Question FRNs are issued for a maturity period of ________ and are normally redeemed in one shot. Correct Answer 5-7 years Your Answer

5-7 years

Multiple Choice Multiple Answer Question Which factors affect the exchange rates in short time period? Correct Answer Demand and supply, interest rates, inflation, , Sentiment, Speculation Your Answer Demand and supply, interest rates, inflation, , Sentiment, Speculation , GDP,Balance of trade and payment,Unemployment Select The Blank Question ________ is the acid test for events to be included in country risk. Correct Answer Control by Government' Your Answer

Control by Government'

Multiple Choice Single Answer Question How many types of documents are required in the International trade? Correct Answer Six Your Answer

Six

Multiple Choice Multiple Answer Question What are the main causes of fluctuations in Exchange rates?

Correct Answer Government regulatios, fiscal policies, political instabilities , Market demand and supply, sentiments Your Answer Government regulatios, fiscal policies, political instabilities , Market demand and supply, sentiments , The purchasing power of currency changes every second Multiple Choice Multiple Answer Question Which two countries were included to convert the group of five into the group of seven? Correct Answer Canada , Italy Your Answer

Canada , Italy

Multiple Choice Single Answer Question What do you call a 'decreasing principal swap'? Correct Answer Amortizing swap Your Answer

Amortizing swap

Multiple Choice Single Answer Question Who are the members of the Special committee on Indian Direct Investments abroad, for normal route? Correct Answer Representatives of Ministry of finance, Ministry of Economic affairs, Ministry of Commerce and RBI Your Answer Representatives of Ministry of finance, Ministry of Economic affairs, Ministry of Commerce and RBI Multiple Choice Single Answer Question What are the functions of Export Import Bank of India? Correct Answer Finance functions and development functions Your Answer

Finance functions and development functions

LIST OF ATTEMPTED QUESTIONS AND ANSWERS True/False Question

Services of a Paying bank are taken when currencies of exporters country or the importers country is not involved in the transaction. Correct Answer False Your Answer

False

Select The Blank Question To make the export more competitive and remunerative, the Indian Govt extends ________ facility. Correct Answer Cash Incentives Your Answer

Line of credit

Multiple Choice Multiple Answer Question For issuing a Documentary credit The applicant is required to provide to the issuing bank details about Correct Answer The purchase-sales contract, the mode of payment, the tenor of payment , The documents required, the names of authorities who would issue the documents Your Answer The purchase-sales contract, the mode of payment, the tenor of payment , The documents required, the names of authorities who would issue the documents , The track record of the buyer and seller Multiple Choice Multiple Answer Question Which of these are not considered as good securities by the banker? Correct Answer House Airway bill , Postal Receipt Your Answer

House Airway bill , Postal Receipt , On board bill of exchange

Multiple Choice Single Answer Question What is the statutory requirement about receivables of exports from India? Correct Answer The export receivables must be repatriated within six months of the date of shipment. Your Answer The export receivables must be repatriated within six months of the date of shipment. Multiple Choice Single Answer Question What do firms face while evaluating risks in cross border investment? Correct Answer The issues are different, unique to each project, and solutions are varied Your Answer

The issues are different, unique to each project, and solutions are varied

True/False Question

An Indian importing goods from USA pays dollars in India.

Correct Answer False Your Answer

True

Multiple Choice Multiple Answer Question Name the services provided by 'Reuter 3000' in forex market:Correct Answer Quotation of forex rates, interest rates , Electronic broking by matching the rates Your Answer Quotation of forex rates, interest rates , Electronic broking by matching the rates , Calculate and quote rates for transactions Match The Following Question

Correct Answer

Your Answer

16 principles for good environmental conduct

Business Charter of sustainable development

Business Charter of sustainable development

World Chambers Congress

World Federation of Chambers

Doha

ICC Conference in 2004

Marrakesh

Marrakesh

13 types of trade contracts

INCOTERM

World Federation of Chambers

Multiple Choice Single Answer Question What is the cause of Volatility in the exchange market? Correct Answer Interest rates are changing, Interest rates affect and are affected by exchange rates Your Answer Interest rates are changing, Interest rates affect and are affected by exchange rates Multiple Choice Single Answer Question How many Countries participated in the third survey on Foreign Exchange, coordinated by BIS in 1992? Correct Answer 26 Your Answer

26

True/False Question

Cash flows generated from aforeign project may replace revenueproducing imports to the host country Correct Answer False Your Answer

True

Multiple Choice Multiple Answer Question Internationally only those currencies are in demand Correct Answer Which are convertible , Which have liquidity confirmed in Euro market operations

Your Answer

Which are convertible , Which have liquidity confirmed in Euro market operations , which have greater gold parity

True/False Question

Out of 15 member countries in European Union 12 countries commiteed themselves for a single currency. Correct Answer False Your Answer

False

Multiple Choice Multiple Answer Question Major investors in NIF are :Correct Answer Commercial Banks , Non banking Finance companies , Insurance Companies Your Answer Non banking Finance companies , Commercial Banks Select The Blank Question The major industrial power whose economy was not affected after the II world war was ________. Correct Answer USA Your Answer

Germany

Multiple Choice Multiple Answer Question What are the main causes of fluctuations in Exchange rates? Correct Answer Government regulatios, fiscal policies, political instabilities , Market demand and supply, sentiments Your Answer Government regulatios, fiscal policies, political instabilities , Market demand and supply, sentiments , The purchasing power of currency changes every second Multiple Choice Single Answer Question The severemost problem faced by the Fixed exchange rate system was Correct Answer Inability of US to change gold for dollars Your Answer

Measurement of world liquidity

Multiple Choice Multiple Answer Question What schemes have been floated by RBI for facilitating Inter bank transactions? Correct Answer Clearing house for settlement and clearance for the operations of Ads , Set up a market intelligence cell to study and monitor developments in Forex Market Your Answer Clearing house for settlement and clearance for the operations of Ads , Set up a market intelligence cell to study and monitor developments in Forex Market , Purchase excess foreign curency from Ads at higher rates Select The Blank

Question

Over the years ________has been the most widely accepted currency in international capital market. Correct Answer American Dollar Your Answer

Euro Dollar

Match The Following Question

Correct Answer

Your Answer

Departure

EXW

EXW

Main carriage paid

CFR

FAS

Main carriage unpaid

FAS

FAS

Arrival

DES

DES

True/False Question

The American Dollar market comprises of the domestic segment of USD the offshore segment of USD. Correct Answer True Your Answer

True

True/False Question

Tranche' is a managed issue of securities in smaller denominations.

Correct Answer True Your Answer

True

Multiple Choice Single Answer Question Which is the most actively traded currency in the international market today? Correct Answer US Dollar Your Answer

EURO

True/False Question

Prior approval of RBI is needed for payment of all imports.

Correct Answer False Your Answer

True

Select The Blank Question Mostly, countries in the Middle east require ________ invoice, for facilitating import duties .

Correct Answer Legalised Your Answer

Customs

Select The Blank Question A ________ Bill of lading does not specify the terms and conditions of carriage. Correct Answer Short Form Your Answer

Short Form

True/False Question

Cause of increase in cost of public issues Underwriting fees , selling commission Correct Answer True Your Answer

True

Multiple Choice Single Answer Question How many types of documents are required in the International trade? Correct Answer Six Your Answer

Six

Multiple Choice Single Answer Question If Investment is out of EEFC accounts then :Correct Answer Exemption is granted from the criteria of Core activity stipulation under automatic route Your Answer Exemption is granted from the criteria of Core activity stipulation under automatic route Select The Blank Question When a credit is made to available to additional beneficiary wholly or partly it is called as ________. Correct Answer Transferable credit Your Answer

Transferable credit

Select The Blank Question India is a founder member of ________. Correct Answer IMF Your Answer

IMF

True/False Question

Bid rate is bidders buying rate and offered rate is bidders selling rate.

Correct Answer True Your Answer

True

Select The Blank Question When interest rate changes by 0.05 per cent then it has changed ________ basis points. Correct Answer 50 Your Answer

5

Select The Blank Question Partnership firm not eligible for Overseas Direct Investment needs to apply to RBI Approval in form ________. Correct Answer ODI Your Answer

ODA

Multiple Choice Multiple Answer Question With reference to ECB, fund raising activity includes :Correct Answer Floating rate notes, bonds, debentures , Syndicate loans, issue of shares Your Answer

Floating rate notes, bonds, debentures , Syndicate loans, issue of shares

Select The Blank Question Rupee was liberalised on ________ account in 1993, and on current account in 1994. Correct Answer Trade Your Answer

Capital

Select The Blank Question FRNs are issued for a maturity period of ________ and are normally redeemed in one shot. Correct Answer 5-7 years Your Answer

Up to 1 year

Multiple Choice Multiple Answer Question What are the objectives of International Chamber of Commerce? Correct Answer Champion the global economy as a force of economic growth , Creating of job and prosperity Your Answer Champion the global economy as a force of economic growth , Creating of job and prosperity Multiple Choice Multiple Answer Question Which contracts are used for transportation by road?

Correct Answer FCA , CPT Your Answer

FCA , CPT

Multiple Choice Single Answer Question Financial management is complex process for Correct Answer Companies which have international trade Your Answer

Companies which have international trade

True/False Question

The volume of trading of Derivatives in the world trade market is growing but still only 5% of the total. Correct Answer False Your Answer

True

Multiple Choice Multiple Answer Question What are the functions of International Finance Corporation? Correct Answer Help private enterprise by financing projects , Provide technical assistance to private enterprise Your Answer Help private enterprise by financing projects , Provide technical assistance to private enterprise , Help to maintain the exchange rate stability Multiple Choice Multiple Answer Question What does a speculator accept while dealing with an option? Correct Answer The exercise or strike price, the amount , The expiration date and time, the method of payment Your Answer The exercise or strike price, the amount , The expiration date and time, the method of payment , The Market report of the exchange which trades the option Select The Blank Question The ________is a Floating rate index for swaps. Correct Answer US Commercial paper rate Your Answer

US Commercial paper rate

Multiple Choice Single Answer Question What is the role of ICC towards WTO? Correct Answer To provide world business recommendations to WTO Your Answer

To provide world business recommendations to WTO

LIST OF ATTEMPTED QUESTIONS AND ANSWERS True/False Question

The control of enviromental aspect Life side analysis.

Correct Answer True Your Answer

False

True/False Question

Enviroment indudes natural resources.

Correct Answer True Your Answer

False

Select The Blank Question TQM emphasises on ________. Correct Answer Small improvement Your Answer

Continuous improvement

Multiple Choice Multiple Answer Question How TQM helps in reducing cost? Correct Answer By reducing external failure cost , By reducing internal failure Your Answer

By reducing external failure cost , By reducing internal failure

True/False Question

The main business process objective is to serve the customer in excellent manner. Correct Answer True Your Answer

True

Select The Blank Question The Design & Development Review to be carried ________ Correct Answer To evaluate ability of product to meet requirements Your Answer

To evaluate ability of product to meet requirements

Select The Blank Question Circle represent ________ while plotting flow chart.

Correct Answer Beginning or end of operation Your Answer

Inspection

Select The Blank Question Rectangle represent ________ While plotting flow chart. Correct Answer Steps in Activity Your Answer

Steps in Activity

True/False Question

ISO - 9000 -2000 this revision of ISO is currently in use.

Correct Answer True Your Answer

False

Multiple Choice Single Answer Question In TQM philosophy T stand for :Correct Answer Total Your Answer

Trust

True/False Question

Prevention of pollution is prevention of cause of pollution.

Correct Answer True Your Answer

True

Multiple Choice Multiple Answer Question Control chart is :Correct Answer Process monitoring tool , Process control tool Your Answer

Process control tool

Multiple Choice Single Answer Question In quality Assurance primary concern is:Correct Answer Co-ordination Your Answer

Control

Multiple Choice Single Answer

Question

Evolution of quality concept. Current era is

Correct Answer Continual Improvement Your Answer

Continual Improvement

True/False Question

Machine down time is waste.

Correct Answer True Your Answer

True

Multiple Choice Single Answer Question Strategic quality management means :Correct Answer Strategic Improvement Your Answer

Control

Multiple Choice Single Answer Question Enviromental objective means :Correct Answer Over all environmental goal Your Answer

Environmental target

Multiple Choice Multiple Answer Question Tqm focus on :Correct Answer Employee , Customer Your Answer

Management

Multiple Choice Multiple Answer Question Common elements of winners are :Correct Answer Senior management was actively involved , Control of overall process , Focus on customer Your Answer Senior management was actively involved Match The Following Question The management representative is responsible far

Correct Answer

Your Answer

Establishment & implementation of QMS

Establishment & implementation of QMS

The management review conducted

As & when required

Documentation

In ISO the focus on customer

Enhancing customer satisfaction

Supplier satisfaction

Responsibility for quality objective

Top management

Top management

Match The Following Question

Correct Answer

Your Answer

Dr. Deming believes

To prevent defect

To prevent defect

Ishikawa development

Cause & effect diagram

Histogram

Type of variation is due to Common causes

Common causes

Crosby's objective of quality

Quality & productivity work together

Histogram

Multiple Choice Multiple Answer Question The benefite of ISO 14001 are applicable to :Correct Answer The organisation which adopts it , Adjacent property owner & enviroment Your Answer The organisation which adopts it , Preservation of Nature Multiple Choice Single Answer Question Control chart shows :Correct Answer Performance of process Your Answer

Performance of product

Multiple Choice Multiple Answer Question The mean is :Correct Answer Center of process , Target of process Your Answer

Target of process

Select The Blank Question ________organisations are not eligible for MBNQA. Correct Answer Software Your Answer

Health care

Multiple Choice Single Answer Question TQM is aiming :Correct Answer Long term success Your Answer

Long term success

Multiple Choice Multiple Answer Question When cpk is less than one, :Correct Answer Process is not capable , Process is volatile Your Answer

Process is not capable

True/False Question

Leadership from top should be totally honest & transparent in dealing . Correct Answer True Your Answer

True

True/False Question

By Applying basic principle, the process improvement will be in organization's focuses on work process. Correct Answer True Your Answer

True

Multiple Choice Multiple Answer Question Corrective action means :Correct Answer Action taken on detecting NC , Action taken on observed NC Your Answer

Action taken on detecting NC , Action taken on defects

Multiple Choice Single Answer Question The process evaluation is to identify :Correct Answer Potential improvement in process Your Answer

Correctness of process

Select The Blank Question The cycle used in EMS is ________. Correct Answer Plan - Do - check - Act

Your Answer

Plan - Do - check - Act

True/False Question

Significant aspect means Perceived Risk.

Correct Answer True Your Answer

False

True/False Question

Enviromental impact is any change to the environment, whether adverse or beneficial. Correct Answer True Your Answer

False

Multiple Choice Multiple Answer Question Employee should be involved in :Correct Answer Decision making , Participation Your Answer

Participation

Select The Blank Question Quality approach is________ Correct Answer Potential failure prevention Your Answer

Inspection

Select The Blank Question The enviromental management system followed by organisation is ________. Correct Answer IS0-14001 Your Answer

IS0-14001

Multiple Choice Single Answer Question Process Quality planning means :Correct Answer Building quality into processes Your Answer

Building quality into processes

Select The Blank Question Tool is used in TQM implementation is ________.

Correct Answer P -D - C - A Your Answer

Quality circle

Multiple Choice Multiple Answer Question Current quality concept is :Correct Answer Continual improvement , Prevention of defect Your Answer

Continual improvement

Multiple Choice Single Answer Question In statistical quality control, primary concern is :Correct Answer Control Your Answer

Co-ordination

Multiple Choice Single Answer Question TQM approach is based on :Correct Answer Participation Your Answer

Participation

Multiple Choice Multiple Answer Question Ishikawa is the originator of :Correct Answer Fishbone Diagram , Cause effect Diagram Your Answer

Fishbone Diagram

Select The Blank Question ________ is calibration activity in company. Correct Answer Appraisal cost Your Answer

Prevention cost

Multiple Choice Multiple Answer Question Continual improvement is in :Correct Answer Enviromental objective , Audit Result , Corrective action Your Answer

Enviromental objective , Corrective action

Select The Blank

Question

Quality approach is________

Correct Answer Potential failure prevention Your Answer

Inspection

Multiple Choice Single Answer Question The employee must be :Correct Answer Aware about objectives Your Answer

Aware about objectives

Process capability is only calculated when it qualify Bell shape curve.

True False In control chart, X bar means :-

a) b)

Average of Reading Sum of Reading

c)

Mean of Reading

d)

Mode of Reading

In ISO - 90001Quality policy declare by management. Similarly what is specified in ISO - 14001?

a) Enviromrntal Policy

b)

Enviromentel plan

c)

Enviromentel objectives

d)

20.

Enviromental Regulations

Quality circle encourage :-

a)

Participation

b)

Tem work

c)

Groupism

d)

Individual Growth

Marks : 2 30.

The documents of external origin

The documents of external origin :-

a) b)

Should be controlled separately Should have

c) separate system d)

Require special control Should have documented procedure

Continious improvement is improvement of inviromental system.

True False

1. Pillar of TQM

1. Due to cut throat competition

2. Need for TQM

2. Employee

3. TQM focuses on

3. Continual improvement management

4. Quality approach

4. Change is technology 5. Productivity -Quality - cost -Delivery 6. Inspection

47.The difference of USL & LSL is range. True

False

Range is :a) b)

Difference of minimum & maximum Diff of highest & lowest

c) d)

Average of Reading Sum of Reading

LIST OF ATTEMPTED QUESTIONS AND ANSWERS Select The Blank Question The primary responsibility of human Resource is ________ Correct Answer To provide competent person Your Answer

To provide competent person

Multiple Choice Multiple Answer Question The inviromental policy must :Correct Answer Be Relevant to the activity , Comply with legislation , Commits to prevention of pollution Your Answer Commits to prevention of pollution , Welfare policy Select The Blank Question The management review to be conducted ________. Correct Answer At predetermined interval Your Answer

As & when required

Select The Blank Question Intangibility is ________. Correct Answer Service is intangible Your Answer

Service is intangible

Select The Blank Question Quality means________. Correct Answer Fitness for use Your Answer

Fitness for use

Multiple Choice Single Answer Question Record may be :-

Correct Answer Procedure to control its use. Your Answer

Retrievable.

Multiple Choice Single Answer Question Poor service is due to :Correct Answer Inadequate Resources Your Answer

Untrained people

Select The Blank Question The quality approach is ________ Correct Answer Potential failure prevention Your Answer

Potential failure prevention

Multiple Choice Single Answer Question What is ISO? Correct Answer International organization far standard Your Answer

International organization far standard

Multiple Choice Multiple Answer Question Common features of CMM is how to produce software product which are :Correct Answer Predictable , Consistence , Repeatable Your Answer

Repeatable

Multiple Choice Multiple Answer Question The records shall be maintained to provide :Correct Answer Evidence of conformity to requirement , Demonstrate effectiveness of system Your Answer Demonstrate effectiveness of system Multiple Choice Multiple Answer Question Dimensions of quality are :Correct Answer Performance , Feature , Reliability Your Answer

Performance , Reliability

Match The Following

Question

Correct Answer

Your Answer

Mission conveys message Company about

Mission statement

Business vision involve

Mission statement

Continues improvement

Business objective must include

Customer satisfaction

Short - term improvement

TQM emphasize on

Continues improvement

Customer satisfaction

Multiple Choice Single Answer Question The need for TQM is due to :Correct Answer Cut throat competition Your Answer

Cut throat competition

True/False Question

TQM & ISO both focuses on employee.

Correct Answer False Your Answer

True

Multiple Choice Multiple Answer Question Reliability of product means :Correct Answer Consistency of performance , Performance over period Your Answer

Consistency of performance

True/False Question

Histogram is used to compare data.

Correct Answer True Your Answer

True

Select The Blank Question Reputation means________. Correct Answer Past performance Your Answer

Past performance

Match The Following

Question

Correct Answer

Your Answer

Bureaucratic

Working together for excellence

Direction from Boss

Leadership from top

Provide consistent vision direction

Provide consistent vision direction

Excellence mean

Satisfy all customer need Satisfy all customer need

Team work mean

Unlimited thinking

Working together for excellence

Multiple Choice Multiple Answer Question While setting Goal ,following things to be consider. Correct Answer Marketability of product , Organization need Your Answer

Customer need , Marketability of product , Organization need

True/False Question

Quality is fitness for Purpose

Correct Answer True Your Answer

True

True/False Question

Product quality planning should address capability & qualification of equipment. Correct Answer True Your Answer

True

Multiple Choice Single Answer Question What is EMS? Correct Answer Enviromentel management system Your Answer

Enviromentel management system

True/False Question

Quality policy is defined by employee of organisation

Correct Answer False Your Answer

False

Multiple Choice Single Answer

Question

Calibration Activity in company is :-

Correct Answer Appraisal cost Your Answer

Appraisal cost

Multiple Choice Single Answer Question The evaluation & selection of vendor :Correct Answer Must have procedure for it Your Answer

To be decided by management

Multiple Choice Single Answer Question Quality objectives :Correct Answer Must be relevant to function within organization Your Answer

Must be relevant to function within organization

True/False Question

Periodic review of process to be taken to ensure consistency of process. Correct Answer True Your Answer

True

Select The Blank Question Working together for excellence is ________. Correct Answer Team work Your Answer

Team work

Multiple Choice Multiple Answer Question Cause & Effect diagram used to :Correct Answer Identify & organize possible causes of problem , Identify possible causes of problem & determining its relation Your Answer Identify possible causes of problem & determining its relation Multiple Choice Single Answer Question In inspection primary concern is :Correct Answer Detection of Defect Your Answer

Detection of Defect

Select The Blank Question Diamond represent ________ while plotting flow chart. Correct Answer Decision point Your Answer

Decision point

True/False Question

The ISO is based on Plan - do - check- Act.

Correct Answer True Your Answer

True

Multiple Choice Multiple Answer Question Cost of poor quality comprises :Correct Answer Cost of internal failure , Cost of external failure Your Answer

Cost of manufacturing

Select The Blank Question ________ is one of the major business processes. Correct Answer Customer servive & supply chain Your Answer

Customer servive & supply chain

Select The Blank Question Response is________. Correct Answer Speed Your Answer

Service

Select The Blank Question Knowing capability of any process is required to ________. Correct Answer Predict the behavior of process Your Answer True/False Question

Eliminate source of variation

When process improvements are institutionlized, the process is optimized level Correct Answer False

Your Answer

True

Multiple Choice Multiple Answer Question Process evaluation is to identify :Correct Answer Potential failure prevention , Correctness of product Your Answer

Potential failure prevention

Multiple Choice Multiple Answer Question Which of the following are depicted as the levels of the TQM? Correct Answer Culture , Management style Your Answer

Management style

Multiple Choice Multiple Answer Question Leadership style from top to bottom is :Correct Answer Totally honest & transparent , Providing consistent vision / direction Your Answer Totally honest & transparent Multiple Choice Multiple Answer Question In inspection primary concern is :Correct Answer Detection of Defect , Action on only detected defect Your Answer

Detection of Defect

Multiple Choice Multiple Answer Question In parato Analysis, we have to focus on problem which contribute :Correct Answer 80 % Of total problem , 80 % Of causes. Your Answer

80 % Of causes.

True/False Question

The European award uses the criteria of efficiency & effectiveness. Correct Answer True Your Answer

True

Select The Blank

Question

The quality approach is ________

Correct Answer Potential failure prevention Your Answer

Potential failure prevention

True/False Question

Physical facilities is tangible dimension.

Correct Answer True Your Answer

True

Multiple Choice Single Answer Question Which is not pillar of TQM Correct Answer Finance Your Answer

Finance

13.

TQM promotes

TQM-4 LIST OF ATTEMPTED QUESTIONS AND ANSWERS Multiple Choice Single Answer Question The management review is carried :Correct Answer To ensure effectiveness Of QMS Your Answer To ensure effectiveness Of QMS Multiple Choice Multiple Answer Question Cause & Effect diagram used to :Correct Answer Identify & organize possible causes of problem , Identify possible causes of problem & determining its relation Your Answer Identify & organize possible causes of problem , Identify possible causes of problem & determining its relation Select The Blank Question Circle represent ________ while plotting flow chart. Correct Answer Beginning or end of operation Your Answer Beginning or end of operation Multiple Choice Multiple Answer Question By Applying basic principle the process improvement will be in organization's :Correct Answer Focus on work process , Maintain self esteem of other Your Answer Focus on work process , Maintain self esteem of other True/False Question TQM approach is based on Participation. Correct Answer True Your Answer True Multiple Choice Single Answer Question In statistical quality control, primary concern is :Correct Answer Control Your Answer Control True/False Question Histogram is used to compare data. Correct Answer True Your Answer True Page 1

TQM-4 Multiple Choice Single Answer Question In control chart, X bar means :Correct Answer Average of Reading Your Answer Average of Reading Multiple Choice Single Answer Question If some of the activities are outsourced :Correct Answer Vendors are approached on predetermined criteria Your Answer Vendors are approached on predetermined criteria Multiple Choice Single Answer Question In inspection primary concern is :Correct Answer Detection of Defect Your Answer Detection of Defect True/False Question Quality policy is defined by employee of organisation Correct Answer False Your Answer False Multiple Choice Single Answer Question Evolution of quality concept. Current era is Correct Answer Continual Improvement Your Answer Continual Improvement Select The Blank Question The cycle used in EMS is ________. Correct Answer Plan - Do - check - Act Your Answer Plan - Do - check - Act True/False Question The process mapping means sequence & interaction of process. Correct Answer True Your Answer True Select The Blank Question TQM promotes________ Correct Answer Team work Your Answer Team work Page 2

TQM-4 Multiple Choice Multiple Answer Question Ishikawa is the originator of :Correct Answer Fishbone Diagram , Cause effect Diagram Your Answer Fishbone Diagram , Cause effect Diagram Multiple Choice Multiple Answer Question Factor in GAP 1 is :Correct Answer In sufficient market research , Luck of interaction between management & customer , Luck of communication Your Answer In sufficient market research , Luck of interaction between management & customer , Luck of communication Select The Blank Question ________ is calibration activity in company. Correct Answer Appraisal cost Your Answer Appraisal cost Multiple Choice Single Answer Question Enviromental impact is :Correct Answer Any change to envioment resulting from organization activity Your Answer Any change to envioment resulting from organization activity Multiple Choice Multiple Answer Question Reliability of product means :Correct Answer Consistency of performance , Performance over period Your Answer Consistency of performance , Performance over period Select The Blank Question The enviromental management system followed by organisation is ________. Correct Answer IS0-14001 Your Answer IS0-14001 Multiple Choice Multiple Answer Question Which of the following are depicted as the levels of the TQM? Correct Answer Culture , Management style Your Answer Culture , Management style Page 3

TQM-4 Multiple Choice Single Answer Question Strategic quality management means :Correct Answer Strategic Improvement Your Answer Strategic Improvement True/False Question Quality approach is potential failure prevention. Correct Answer True Your Answer True Multiple Choice Multiple Answer Question Current quality concept is :Correct Answer Continual improvement , Prevention of defect Your Answer Continual improvement , Prevention of defect Multiple Choice Single Answer Question Quality objectives :Correct Answer Must be relevant to function within organization Your Answer Must be relevant to function within organization Select The Blank Question Quality means________. Correct Answer Fitness for use Your Answer Fitness for use Select The Blank Question The quality approach is ________ Correct Answer Potential failure prevention Your Answer Potential failure prevention True/False Question Documents in company are controlled by Management Representatives. Correct Answer True Your Answer True Multiple Choice Single Answer Question ISO - 140001 is built upon which of the following? Correct Answer Plan - Do - check - Act Your Answer Plan - Do - check - Act Page 4

TQM-4 Multiple Choice Multiple Answer Question The inviromental policy must :Correct Answer Be Relevant to the activity , Comply with legislation , Commits to prevention of pollution Your Answer Be Relevant to the activity , Comply with legislation , Commits to prevention of pollution True/False Question Value is for which customer is paying. Correct Answer True Your Answer True Multiple Choice Single Answer Question Creativity means :Correct Answer Unlimited thinking Your Answer Unlimited thinking Multiple Choice Single Answer Question Control chart shows :Correct Answer Performance of process Your Answer Performance of process Multiple Choice Single Answer Question Tally chart is :Correct Answer Data collection tool Your Answer Data collection tool Match The Following Question Correct Answer The quality manual have interaction of process

Your Answer Level 3

The sequence of operation &

The process mapping The sequence of operation & interaction of process Schematic flow diagram The role of management in QMS resources

To provide resources

To provide

Activities outsourced Vendors are assessed based on predetermined criteria Vendors are assessed based on predetermined criteria Page 5

TQM-4 Multiple Choice Multiple Answer Question In inspection primary concern is :Correct Answer Detection of Defect , Action on only detected defect Your Answer Detection of Defect , Action on only detected defect Multiple Choice Single Answer Question The process evaluation is to identify :Correct Answer Potential improvement in process Your Answer Potential improvement in process Match The Following Question Correct Answer Dr. Deming believes Ishikawa development

Your Answer

To prevent defect

To prevent defect

Cause & effect diagram

Type of variation is due to Crosby's objective of quality

Common causes Histogram

Cause & effect diagram

Common causes

Histogram

Select The Blank Question Working together for excellence is ________. Correct Answer Team work Your Answer Team work Multiple Choice Multiple Answer Question Bar chart is :Correct Answer Frequency Distribution tool , Data analysis tool Your Answer Frequency Distribution tool , Data analysis tool , Process control tool Multiple Choice Multiple Answer Question Continual improvement is in :Correct Answer Enviromental objective , Audit Result , Corrective action Your Answer Enviromental objective , Audit Result , Corrective action Page 6

TQM-4 True/False Question The performance of process is in controlled condition then it is called process at managed level. Correct Answer False Your Answer True True/False Question Failure mode & effect Analysis is prevention tool. Correct Answer True Your Answer True Multiple Choice Multiple Answer Question Quality circle encourage :Correct Answer Participation , Tem work Your Answer Participation , Tem work Multiple Choice Single Answer Question Analysis is :Correct Answer Process monitoring tool Your Answer Data analysis tool Multiple Choice Single Answer Question In quality Assurance primary concern is:Correct Answer Co-ordination Your Answer Co-ordination

Page 7

Related Documents

3rd Sem All Pgdib
November 2019 20
3rd Sem Finance All Subjects
November 2019 15
3rd Sem
October 2019 50
3rd Sem
November 2019 40
Syllabus 3rd Sem
June 2020 15